Missed Questions

Pataasin ang iyong marka sa homework at exams ngayon gamit ang Quizwiz!

HEV

+HbsAb, -antiHBc, -antiHAV, -antiHBV & -antiHCV with hepatitis like symptoms indicates what?

Chimeric; Humanized; Tyrosine kinase; Leprosy; Vas deferens & Gartner's duct; Collecting duct onwards; Glomeruli to DCT; Elbow & hook of the hamate

-xi- indicates what? -zu-? -nib? Hyopigmented patches with anesthesia over them are what? What does the mesonephros become in boys & girls? Ureteric bud gives rise to what? Metanephric mesoderm? Two most common locations of ulnar nerve injury?

IgA nephropathy; Thin basement membrane disease; PSGN is weeks after, IgA is days after; Psoas & iliacus

17-year-old with hematuria following a flu like illness has what? Asymptomatic hematuria with no back story is what? PSGN versus IgA nephropathy timing? Two major flexor of the hip?

CJD because 2-months is very fast; chorea is somewhat voluntary and directed

2- month onset of rapidly progressive dementia, ataxia and startle myoclonus indicates what? How does chorea differ from myoclonus?

H. influ, Moraxella & strep pneum; Rash, arthralgias & conjunctivitis; Choline uptake in presyn; Acetylcholine synthesis; Prevent Ach from entering vesicle; PCNSL; B-cell EBV; ACTH, MSH & beta-endorphins

3 COPD upper respiratory pathogens? Zika signs (3)? Where does hemicholinium block? Bromoacetylcholine? Vesamicol? HIV patient treated for toxo w/ no improvement has what? Cell & virus commonly found? 3 products of POMC?

Albumin, transferrin & transthyretin; A unique ARP that rises in response to bacterial toxins and falls in viral infections; Dry=gradual w/ drussen deposits, wet=acute w/ gray macula & hemorrhage; Smoking cessation, VEGF inhibitors for wet; Muscarinic & prostaglandin agonists; Beta-blockers, carbonic anhydrase & alpha2 agonists; Ciliary epithelium not ciliary muscle

3 negative ARPs? What is procalcintonin? Timing & funduscope of dry versus wet AMD? Treatment? 2 drugs to increase aqueous outflow? Decreases inflow (3)? What produces aqueous humor?

Prostate, small cell, Hodgkin; Cadherins & laminins; Effects the processing of mRNA; Microsomal monoxygenase; Sarcoma, breast, brain, adrenal & leukemia; Tissue factor (thromboplastin) is released into maternal circulation; It has hypotension & cardio shock

3 osteoblastic bone mets? Detachment & adherence are via what proteins? Pathogenesis of beta-thal? What detox enzyme inadvertently produces carcinogens? 5 cancers of li-fraumeni? Placental abruption causes DIC via what mechanism? How does amniotic fluid embolism differ?

Congenital CMV infection; IV Ganciclovir or oral valganciclovir

3 week old with petechial rash & this lung biopsy has what? Treatment?

Normal conjugated bilirubin indicates a biliary system obstruction over a liver defect

3-week-old boy with jaundice, dark urine, hepatomegaly, clay-colored, acholic stools, and normal conjugated bilirubin indicates what?

LTB4, 5-HETE, C5a & IL-8; Giant cell arteritis; HTN; MCD because PSGN would not have proteinuria high enough for frosty urine and only causes periorbital edema; Activate macrophages release 1-alpha-hydroxylase

4 neutrophil chemotaxins? Woman w/ OA & HTN with headache & muscle pains has what? IgA nephropathy produces what systemically? Boy with leg edema, periorbital edema & frothy urine has what? What causes the hypercalcemia in sarcoidosis?

Congestion (red & boggy), red hepatization (red & firm), grey hepatization (grey & firm), restoration; G-CSF to treat neutropenia after chemo; Hemolytic anemia w/ thrombocytopenia; Decreases degradation of vWF; Aminocaproic acid; KRAS GTP-binding protein that regulates cell growth; Inactivating mutations

4 stages of lobar pneumonia? What is filgrastim? 2 of 5 symptoms necessary to diagnose TTP? Decreased ADAMTS13 does what? What is an anti-fibrinolytic medication used to stop post-op bleeding? What does EGFR bind & function? What type of mutations cause cancer in APC & p53?

Methotrexate, sulfsalazine, hydroxychloroquine, minocycline & TNFa inhibitors; NSAIDS or glucocorticoids as DMARDs take weeks to take effect; Endothelial & capillary injury leading to noncardiogenic pulmonary edema; Normal; Atelectasis can occur

5 DMARDS in RA? Immediate RA relief (2)? Mechanism of ARDS? What is the PCWP? How can this cause V/Q mismatch?

Lung, breast, colon, pancreas, ovary; Lung, prostate, colon, pancreas, bladder; Thyroid, uterine, melanoma; Improved quality of care prolonging disease duration (better screening methods would increase incidence too)

5 cancer deaths in woman? In men? 3 others that are high in incidences but not deaths? What can explain a rising prevalence with a stable incidence?

Glucocorticoids, aromatase inhibitors, P450 inducers, PPI's & GnRH agonists; 5-15; Hypoventilation or high altitude; Zone I; Cervical; Thoracic & lumbar; Intertrigo

5 meds associate with osteoporosis? What is a normal A-a gradient? What causes hypoxemia w/ a normal A-a gradient (2)? Zone effected by HBV? Part of spine effected in RA? OA (2)? What does Candida cause in skin folds?

Red neurons <24 hours, neutrophils 24-72, microglial 3-7days, reactive gliosis (liquefactive) 1-2 weeks, astrocyte glial scar; Glutamine; Arginine; Spastic paresis & choreoathetoid; Vitamin C; Cotton-wool spots & flame hemorrhages; Fibrinous; Optic disc cupping

5 stages of neuron ischemia? GABA comes from what amino acid? Urea? Arginase deficiency signs? False negative stool guaiac test is what? 2 eye signs of hypertension? Type of necrosis? Glaucoma sign?

Korsakoff syndrome because Alzheimer's does not have preserved longterm memory

50 year old man with alcoholism, difficulty with short-term memory but preserved longterm memory suggests what?

Inhibit APC, Methylate, Increase COX-2, activate KRAS, inactivate DCC, inactive p53; Eosinophilic/acidic; Dopamine is for central nausea (migraines), 5HT3 is for stomach nausea (bugs) & H1/M1 are vestibular (motion sickness); External iliac, common iliac, abdominal aorta, inferior mesenteric; GTP

6 steps of adenoma to carcinoma sequence? Councilman bodies are apoptotic bodies filled with what? Difference in dopamine & 5HT3, H1/M1 treatments of nausea? Diverticulitis w/ sigmoid bleeding, catheter starts at femoral artery and goes where? What does KRAS bind?

BRCA, APC, p53, Rb, Wt1, VHL; Melanoma & hairy cell; Pleomorphic xanthoastrocytoma; PI3K, PIP2, Akt, mTOR (gene transcription); PTEN; Renal DVT; Antithrombin III

6 tumor suppressor genes. Two cancers of BRAF? Brain tumor with reticulin deposits is what? List the order of PI3K pathway? What gene inhibits mTOR? Guy with nephrotic syndrome & new varicocele has what? What product was lost?

Streptococcus agalactiae (Group B Strep)

A 2-week-old w/ fever, difficulty breathing, and mottled appearance most likely is infected with what gram-positive coccus (Larger name)?

Premature ovarian failure; FSH>40

A 35yo woman with amenorrhea but normal prolactin and TSH likely has what? What lab is elevated?

Mayer-Rokitanksy-Kuster-Hauser; Mullerian ducts; No uterus & scant pubic hair; Complete is p57 negative & incomplete is p57 positive; Vaginal bleeding, hyperemesis gravidarum & theca-leutin cysts

A 46 XX girl with a rudimentary uterus likely has what disease? Aplasia of what structure causes this? What would AIS show? Complete versus incomplete moles have what immunohistochemistry? What three manifestations can moles cause?

Diverticulitis

A 73-year-old woman w/ abdominal pain, increasing constipation and blood in the stool most likely has what?

1/8 b/c 1/2 inherited*1/2 passed on*1/2 chance of boy; Indirect bilirubin; Increased fragility on acidified glycerol lysis & abnormal eosin-5-malemide test; APR that binds free hemoglobin; Enterococcus

A boy has hemophilia, chance that his sisters kid will have it? What lab indicates hemolytic anemia? 2 tests for hereditary spherocytosis? What is haptoglobin? What causes vertebral osteomyelitis after UTI?

Abdominal aorta; low bHCG; testes=para-aoritc, scrotum=superficial inguinal; PCD; Persistant bronchial dilation (bronchiectasis)

A boy has left sided testicular torsion, what is the origin of the artery involved? What does bHCG show in miscarriage? Difference in lymphatic drainage of the testes and scrotum? Sperm w/o tails indicates what? What is the pulmonary manifestation?

Ulceroglandular disease from Franciscella tularensis; Rabbit; severe=streptomycin (adults)/gentamicin (children), mild=ciprofloxacin or doxycycline

A boy with a painless ulcer, fever, chills and enlarged lymph nodes after a trip to the country side has what disease from what organism? Vector? Treatment (2 each for severe & mild)?

A normal reaction to the MMR vaccine that will go away on its own

A child with a truncal rash and no fever likely has what?

Flexion because short head of the biceps brachii attach there

A coracoid process avulsion fracture inhibits what movement? Why?

Necrotizing fasciitis of subcutaneous tissues and superficial fascia; Dermis and subcutaneous tissue

A cut with worsening pain, slightly erythematous skin that is tender to the touch and crepitus on palpation indicates what? How does cellulitis differ?

Pilocytic astocytoma

A cyst with a mural nodule and eosinophilic intracellular inclusions with a twisted appearance indicates what?

PCOS; Increased LH w/ decreased FSH

A female with heavy menses, increased facial hair and acanthosis nigricans has what? Labs?

Neuroblastoma; chemo and surgical resection; skin, bone, and posterior mediastinum; opsoclonus-myoclonus ataxia; adrenal carcinomas or adenomas

A firm, tender, pediatric adrenal mass indicates what? Treatment? Common sites of metastases? Weird rare sign? What tumors do you treat with hormone modulators?

B12 deficiency from Diphyllobothrium latum; Ancylostoma or Necator

A fisherman with dementia likely has what? What infection causes microcytic anemia?

Continuously b/c aortic root is always higher pressure; Phentolamine; Left subclavian; Repercussion injury allows it to leak out of damaged cell membranes

A fistula between the aortic root & RV would have blood flow when? Norepinephrine IV that suddenly extravasates should be treated with what drug? A stanford type B aortic dissection originates next to what artery? What causes elevated CK after clot removal?

Von Willebrand's disease b/c platelet abnormalities cause more superficial bleeding than something like hemophilia or Factor V Leiden

A girl with bruising, heavy menses & nosebleeds likely has what?

Prolactin because the others would have an inhibitory effects but decreased dopamine would increase prolactin

A lesion in the hypothalamus would most likely increase what hormone?

Posterior membranous portion; bulbous urethra; bHCG & inhibin A; Blood=1 week, urine=2 weeks

A male urethral injury after a pelvic fracture in an MVA is most likely located in what portion? What is injured in a saddle injury? What two amniotic fluid markers are elevated in down's syndrome? When is bHCG detectable in the blood? Urine?

Interosseous membrane

A man pushes a piano across a floor. What structure transmits the force from the radius to the ulna?

SIADH because increased ACTH would cause Cushing's syndrome and hyperaldosterone would cause hypernatremia not hyponatremia

A man with a small cell lung cancer & serum sodium of 120 likely has what?

Type I (IgE-mediated)

A man with urticaria and angioedema after taking aspirin for the 2nd time ever is what type of hypersensitivity reaction (name)?

Primary hyperparathyroidism because confusion and nausea indicates hypercalcemia

A mildly confused & nauseous woman with brown tumors in her finger bones most likely has what?

Palmaris longus because it is the most superficial

A minor laceration near the thenar eminence would injure what tendon? Why?

Neonatal tetanus; Vaccination of the pregnant woman

A neonate with difficulty feeding, not opening mouth, clenched hands and arched back indicates what? How can this be prevented?

21-hydroxylase deficiency (CAH)

A newborn with low cortisol, low aldosterone and ambiguous genitalia has what?

UPEC (uropathogenic E. coli); P pilus

A non-sexually active woman with a UTI is caused by what organism? Virulence factor?

Non-functioning pituitary adenoma because prolactinomas typically cause prolactin levels that are higher than 100 ng/mL with symptoms like galactorrhea

A non-symptomatic patient with a pituitary mass and a prolactin level of 45 has what?

Renin will be low! because of feedback; Myeloperoxidase; Bleach; Pick disease (frontotemporal dementia); Renin in right renal vein versus left renal vein;

A patient given fludrocortisone will have what renin levels in 5 days? What enzyme gives mucus its green color? What does it produce? Man with progressively strange behavior & has become rude & unkempt has what? Right renal artery stenosis lab finding to predict elevation of obstruction is what?

Bacillus cereus from soil that was on the saw

A patient has vision loss after a rusty saw shatters into his eye. What organism causes this?

Peritonsillar abscess; Away from the infected tonsil; Obsessive-compulsive disorder due to immune reaction that damages the basal ganglia (PANDAS=pediatric autoimmune neuropsychiatric disorder associated with strep)

A patient leaning forward, drooling with unilateral swelling & erythema of the posterior oral pharynx, & deviation of the uvula indicates what? Where does the deviation point? What psychiatric condition is a sequela of GAS?

Parametrial (tissues next to cervix) neoplastic invasion; No; Metaplasia

A patient w/ pap smear showing high grade squamous intraepithelial lesions & unilateral hydronephrosis indicates what? Do patients need to be screened for pregnancy or HPV before receiving the HPV vaccine? Transformation zone of the endocervix covered by squamous epithelium instead of simple columnar cells is an example of what?

Oral activated charcoal, N-acetylcysteine if ingestion occurred over 1 hour ago

A patient who ingested acetaminophen 45 minutes ago should be given what antidote?

Pituitary apoplexy, not empty sella syndrome because of the acuity and the hemorrhaging in the MRI

A patient with 3 hours of severe headache, this MRI and no history of pregnancy has what?

Insulinoma; Whipple's=hypoglycemic symptoms brought on by fasting, blood glucose <50mg/dL during syptoms & relief of symptoms with the administration of glucose

A patient with a blood glucose of 45 & elevated insulin, c-peptide and proinsulin most likely has what? Triad?

Loeffler's syndrome from Ascaris lumbricoides

A patient with dyspnea, pleuritic chest pain, rusty sputum & eosinophilia has what disease from which organism?

Spontaneous bacterial peritonitis from translocation of bacteria through the gut wall

A patient with hepatic encephalopathy and no evidence of GI bleed most likely has what?

Insulinoma in MEN1

A patient with hyperparathyroidism, surgical excision of a prolactinoma and a blood glucose of 54 has what?

Adrenal medulla because cortisol and aldosterone would not cause palpitations but catecholamines would

A patient with hypertension, diaphoresis and palpitations has a adrenal tumor in what zone of the adrenal gland?

Insulinoma or surreptitious use of sulfonylureas

A patient with low glucose & elevated insulin/C-peptide can have what two things?

Jarisch-Herxheimer reaction causing lysis of bacteria; ACEI because it will precipitate angioedema; W/ stratification, significance disappears in confounding but usually remains in one population in effect modification

A patient with positive VDRL is given penicillin and then presents with myalgia, fever & headaches, what happened? What drug should you not give someone with C1INH deficiency? What is the difference between confounding and effect modification?

Essential fructosuria which is asymptomatic and often found when they are being worked up for something else

A patient with stomach pain, absent bowel sounds, air pockets in her abdomen & increased reducing sugar in her urine likely has what?

Ethylene glycol because these are calcium oxalate crystals; Fomepizole & ethanol; Glycol acid buildup

A patient with these crystals in their urine likely has what toxicity? Treatment? How does this cause acute kidney injury?

Surgery for detorsion and bilateral orchiopexy because the other testicles probably has the same anatomy and is at risk of torsion as well; Torsion of the testicular appendage

A patients testicle has a horizontal lie, absent cremasteric reflex, and an enlarged, erythematous, and swollen scrotum, treatment? Small firm, blue mass at the top of the testicle indicates what?

Mitral valve proplapse=Crescendo systolic murmur with a midsystolic click

A person with Ehlers-Danlos syndrome likely has what heart sound?

Referral to surgeon for removal of parathyroid; Normal labs

A postmenopausal woman has a radial fracture, elevated serum calcium & elevated PTH. What is the best next step in the management of this patient? What would you see in osteoporosis?

Ospemifene or topical estrogen cream; Estrogen cream should not be used in patients with history breast cancer & ospemifene should not be used in clotting disorders

A postmenopausal woman with pain during sex (dyspareunia) should be given what medication (2)? Contraindications (1 for each)?

Hydatidiform mole because it is too early to be preeclampsia

A pregnant woman at 18-weeks w/ hypertension, bilateral edema & 3+ proteinuria indicates what?

Retinopathy of prematurity due to vascular proliferation

A premature baby given intensive neonatal care with high supplemental oxygen may develop what eye pathology by what mechanism?

Cross-sectional b/c prospective cohort would be over years and would look at incidence; Strength of a qualitative relationship

A study takes blood pressures every day for 7 days, averages them and then tests for a mutation, what type of design is this? When is correlation coefficient used?

Dilated & coiled endometrial glands w/ edematous stroma; Chorionic villi would be present; Apoptosis from stream cell metalloproteases

A woman w/ left adnexal mass, positive BHcg and vaginal bleeding will show what on curettage? What if this was a molar pregnancy? What process causes menses & due to what enzymes?

Secondary hyperparathyrodism due to chronic kidney disease; High PTH, low Ca2+ & high phosphate; Osteitis fibrosa cystica

A woman with altered mental status, a medical history of uncontrolled hypertension and type 2 diabetes mellitus & GFR of 30 mL/min likely has what? Labs for PTH, Ca2+ & phosphate? What bone pathology develops that is associated with both primary and secondary hyperparathyroidism?

Lipodystrophy is a side effect of highly-active antiretroviral therapy (HAART)

A woman with stick like extremities, and increase in abdominal fat but a decrease in facial fat is likely taking what drug?

Candida; T-cells in superficial infection (HIV) & neutrophils in hematogenous candidemia (chemotherapy)

A yeast with pseudohyphae & germ tube formation is what? What two immune system defense protect against this (populations)?

Enterobius vermicularis

A young girl has itchiness in her underwear at night, microscopy shows bean-shaped organisms, diagnosis?

Genitofemoral; Iliohypogastric; Obturator; Lateral femoral cutaneous (inguinal ligament)

Abdominal retractors can injure what nerve? Closure of C-section? Retroperitoneal lymph node removal? What nerve is compressed from hyperflexion of the thigh during birth (ligament)?

Acute=type 3, months out it's replaced by type I collagen; Its a mostly flat line at the bottom; atrial pressure that jumps up in systole; Huge LV & aortic pressure; LV pressure>aortic; Atrial pressure high in diastole

Acute MI granulation tissue versus months out scar tissue have what two collagen types? In a normal pressure/volume tracing, what does the left atrial pressure look like? What indicates mitral regurg? Aortic regurg? Aortic stenosis? Mitral stenosis?

Fibroids (leiomyomas); desmin stains muscle; cytokeratin; vimentin

African-American woman w/ long, painful periods & masses on the uterus has what? Stain used? Epithelial cell stain? Connective tissue cell stain?

Glucose, because erythrocytes don't have mitochondria

After an overnight fast, what is used by erythrocytes for energy and why?

Amniotic fluid embolism; Thromboplastin in the amniotic fluid; Fetal squamous cells in the maternal pulmonary arteries; Inhibit & AMH

After cesarean delivery, a patient has chest pain, difficulty breathing, goes into cariogenic shock & develops DIC, diagnosis? What triggers the DIC? Histology? What two things do Sertoli cells produce?

Aseptic meningits>pancreatitis (rare); Digital rectal exam to check for acute prostatitis;

After orchitis & parotitis, what is the next most common manifestation of mumps? In a patient with fever, chills, pain & hesitancy with urination & muscle aches, what is your next step in management?

Rabies encephalitis; elongated, intracytoplasmic inclusions (Negri bodies) in the hippocampus and cerebellum

Agitation, hypersalivation, autonomic instability, facial muscle spasticity, hydrophobia and aerophobia indicate what? What is found on histology?

Prurigo nodularis; Hyperkeratosis and epidermal hyperplasia

An HIV patient has this skin finding, what is the name and biopsy findings?

Dilated & coiled endometrial glands w/ edematous stroma; Prophase of meiosis I; Metaphase of meiosis II

An ectopic pregnancy with a thickened endometrial stripe would find what on uterine curettage? Primary oocytes awaiting ovulation are arrested in what phase? Secondary oocytes awaiting fertilization are arrested in what phase?

UTI progressing to E. coli bacteremia; LPS on gram negative bacteria

An elderly man with altered mental status, flushed skin & fever likely has what? Antigen?

Root cause analysis starts with interviewing staff members to determine why this is happening; Slow growth to allow collaterals to form; 1/ARR; 1/AR

An error keep occurring in the hospital, what should you do? What feature of an occluding plaque prevents ischemia? Equation for # need to treat? Equation for #needed to harm?

Wilms tumor because they may have WAGR syndrome

An infant with aniridia and genital malformations should also be screened for what?

Cystic fibrosis; pancreatic insufficiency

An infant with poor weight gain despite large food intake, bulky stools and frequent cough indicates what? Why would trypsin be low and fat high?

Endometrial hyperplasia; The vagina not the uterus

An obese postmenopausal woman who took HRT through menopause & now presents with painless vaginal bleeding has what? Where does DES cause adenocarcinoma?

Nonsterile gloves & gown & wash hands with soap and water as alcohol does not kill spores

An old person is hospitalized for an E. coli UTI and develops severe diarrhea, what PPE do you have to wear?

Antigenic shift also known as genetic reassortment

An orthomyxovirus originally transmitted from poultry but now able to transmit from human-to-human underwent what type of change?

Type I collagen has 2 alpha-1 chains & 1 alpha-2 chain, a mutated alpha-2 allele would cause 50% of the chains to be messed up while a mutated alpha-1 allele would cause 75% to be messed up

Analysis of type I collagen protein shows a 50% reduction in the amount of type I collagen, which protein is mutated?

Polyribosylribitol phosphate; Factor H; Trehalose dimycolate; Bosetan & ambrisentan; PTH & EPO production

Another name for Hib's capsule? What factor does this bind? What is found in the cell wall of mycobacteria? What two drugs inhibit endothelial cell proliferation in PAH? What two paraneoplastic syndromes are associated with renal cell carcinoma?

Chronic with low infectivity versus full recovery

Anti-HBe, IgG-HBc & HBsAg verus anti-HBs indicates what?

Anti-centromere versus anti-Scl70 (topoisomerase); PTT; Club (clara) cells; Eczema which looks like a sandpaper rash when young but lichenized when older; Adults only, rare in children

Antibodies in CREST versus diffuse scleroderma? What is paradoxically increased in antiphospholipid syndrome? What lung cells detoxify via P450? Kid who has itchy rash on his cheeks & trunk after eating certain foods has what? What age group is dermatitis herpetiformis in?

Immune-mediated paraneoplastic neuropathy (often with small cell carcinoma); asymmetric weakness & autonomic neuropathy

Antibodies to Hu antigen is associated with what disease? How does it present?

Antiphospholipid syndrome; Hypercoaguability; Systemic sclerosis; Limited chest wall expansion, sending aortitis & anterior uveitis; Scalene muscles; FF=GFR/RPF

Anticardiolipin antibodies are found in what? ?Main symptom? Antitopoisomerase? What three organ systems are effected in ankylosing spondylitis? What muscles contribute to thoracic outlet syndrome? Equation for filtration fraction?

Anti-beta2-glycoprotein & anticardiolipin; Bone marrow suppression; RNA-induced silencing complex; RAS, MYC, EGFR, HER2, ABL, BRAF; DNA repair gene

Antiphospholipid syndrome antibody (2)? Main side effect of TMP-SMX? What is RISC? List the 6 proto-oncogenes. What type of gene is BRCA?

Area postrema & nucleus tractus solitarius; Substance P; Motilin agonist for gastroparesis; Pain & appetite; RNA poly I; 45S pre-rRNA; 18S, 5.8S & 28S;

Area in 4th ventricle & medulla that trigger vomiting? Neurokinin antagonists prevent release of what? Other than antibiotic, what does erythromycin do? Two roles of neuropeptide Y? What enzyme only works in the nucleolus? What gene does this transcribe? 3 products?

Intracellular=cell-mediated response; Nonselective block beta2 mediated K+ uptake; Nausea, vomiting & diarrhea; Hypersensitivity pneumonitis (Farmer's lung)

Bacteria trigger a humoral response, what is the exception? How can beta blockers cause hyperkalemia? 2 side effects of colchicine? Inhalation of thermophilic actinomycetes causes what?

Columnar epithelium with goblet cells

Barrett's esophagus from heart burn has what type of cells?

Diffuse large B-cell lymphoma; Nonpolar & nonhydroxylated; Lysyl oxidase forms desmosine cross links; Coccidoides

Besides Burkitt's lymphoma, what else has a c-Myc mutation? 2 properties of elastin that differ from collagen? What enzyme and modification accounts for its elasticity? Spherules packed with endospores is what?

Denture wearers and DM; 6 week course of Zidovudine & formula instead of breast milk; Cesarean section over vaginal delivery

Besides HIV, what other populations is oral thrush found in (2)? An infant born to a HIV positive mother should receive what intervention (2)? Prophylactic measure?

Negative endocarditis

Besides Q fever, what can Coxiella burnetti also cause?

Annular pancreas; Chorionic villus sampling=weeks 10-14 of pregnancy, amniocentesis=after 14th week

Besides duodenal atresia, what else can cause vomiting in babies with Down's? When would you recommend amniocentesis versus chorionic villus sampling to diagnose in utero?

Shigella; Dehydration, bowel perf & toxic megacolon

Besides listeria, what other organisms uses an actin polymerization system? Most common complication of entamoeba histolytic infection (3)?

Basophils & tryptase; Who have your sexual partners been>what do you identify as (b/c then you're labeling him); Study subjects change behavior when they know that they're being studied; Choosing only hospitalized patients as subjects; Researcher belief in treatment actually effects the outcome (observer-expectancy bias)

Besides mast cells and histamine what other 2 things are elevated in anaphylaxis? A man divorced form his wife and hinting that he's gay should be asked what? What is the Hawthorne effect? Bearskin's bias? Pygmalion?

Clindamycin; candida vulvovaginitis

Besides metronidazole, what else can be used to treat bacterial vaginosis? What is fluconazole used for?

Nasal transepithelial potential difference; Clomiphene acts a SERM to prevent negative feedback; To minimize endometrial hyperplasia

Besides sweat chloride test, what other test can be used of CF? What drug is used to help fertility in PCOS & what is the mechanism? When would you administer progesterone?

Venous thromboembolism & hot flashes; Thiazide diuretic; Neprilysin inhibitor that increases ANP/BNP in heart failure patients

Besides the endometrial effects of tamoxifen, what other two side effects do tamoxifen & raloxifine cause? What kind of drug is chlorthalidone? What is sacubitril used for and what is its mechanism of action?

Prostaglandin release; Crohn's disease; Obstructive biliary processes & hypothyroidism; Sickle cell, analgesics, acute pyelo & diabetes (SAAD papa)

Besides their normal effects, what else do loop diuretics induce? Hyperoxaluria is associated with what? Besides hyperlipidemia, what two diseases are associated with xanthelasma's? Four causes of papillary necrosis (pneumonic)?

Heat production through uncoupling oxidative phosphorylation in the large amounts of mitochondria via the protein thermogenin

Brown adipose tissue makes up 5% of neonates weight. What is it's function & what process/ protein does it use to accomplish this?

Crytpococcus neoformans; Collapsed lung due to bronchial obstruction; Mediastinal deviation towards the effected side; Heparin not aspirin

Budding yeast with a thick capsule is what? Decreased breath sounds with infiltrate of entire lung is what? What else is seen on X-ray? Prevention of a DVT after surgery is with what drug?

Anaplasmosis or Ehrliciosis

Buffy coat examination showing intracytoplasmic vacuoles indicates what two possible diseases?

Prevent MAC; Thrombosis, pancytopenia & hemosiderosis; Basophilic inclusions in neuts (Dohle bodies); Enterocytes; Any net gain of ATP; Transcription activation

CD55/59 normal function? 3 manifestations of PNH? When LAP is high it's a leukemoid reaction, what is seen on blood smear? Dysfunctional cells in celiacs? When erythrocytes produce 2,3-BPG what do they sacrifice? Function of c-Myc?

Towards, away, away, towards; Trochlear; Hemiballismus due to sub thal nuclei; Optic tract; Germinal matrix; Radial!!!!; Sub thalamic or GPi; Neurons

CN V, X, XI, XII deviation? Only serve that exits dorsally? Lacunar stroke causes what movement disorder? Marcus Gunn pupils are a problem with what? In prematurity, intraventricular hemmorhag comes from where? Crutches are what injury? Deep brain stimulation of where is helpful for parkinsons? What does synaptophysin stain?

Conversion of L-Dopa to 3-O-methyldopa; increase the Km b/c it takes more substrate to reach ½ Vmax

COMT inhibitors prevent what? Competitive inhibitors change what component of enzyme functioning?

Herpes encephalitis; Frontal and temporal lobes

CSF with mononuclear leukocytes, elevated protein and normal glucose in a patient with fever and sever headache indicates what? What part of the brain is affected?

Tubular vacuolization; Because it causes metastatic calcification of normal tissues; Inosine monophosphate dehydrogenase (IMPDH) inhibitor; Omalizumab

Calcineurin inhibitors cause what finding in the kidney? Why can high phosphate be dangerous? Mechanism of action of mycophenolate motel? Monoclonal antibody against IgE?

RBF=RPF/(1-Hct); Cafe-au-lait, fibrous dysplasia & precocious puberty; GNAS; HTN & thrombosis; Antibiotics; Decreased GFR because low RPF will cause high oncotic pressure; FF is stable because GFR & RPF increase the same amount

Calculation for renal blood flow? What are the 3 characteristics of McCune-Albright? Gene effected? Two side effects of EPO? Treatment of joint effusion with high WBC? Tons of constriction of the efferent arteriole causes what? Constriction of afferent on filtration fraction?

SD/sqrt(n); Deficient phosphorylation of mannose to target from golgi to lysosome; PPO=more choices but also more money; UGA, UAA, UAG (u go away, u are away, u are gone); It's 1/TPR so take the reciprocal at the end

Calculation for standard error? What is the problem in I-cell disease? Downside of PPO versus HMO? What are the 3 stop codons (pneumonic)? What should you keep in mind when calculating TPR in parallel?

(Oral area*IV dose)/(IV area*oral dose); PPV increases with increased prevalence while NPV decreases; Methionine restriction; Cardiac & muscle problems; Loss of function in SPINK1 which normally produces a trypsin inhibitor or trypsin mutation so that it can't inactive itself; Erythro

Calculation of oral bioavailability? PPV & NPV associations with prevalence? Besides B6, how else do you treat homocystinuria? Carnitine deficiency has what two manifestations? What gene and function causes hereditary pancreatitis (2)? Campy treatment?

No, CT will often not show minor bleeds in the brain

Can a CT scan rule out a suspected subarachnoid hemorrhage?

Yes; Contraction; GBS even if they don't give you a history of infection; Lynch syndrome; Increased sympathetic nerve to the SA node;

Can people with epilepsy drive? Atelectasis after radiation causes what? Ascending paralysis is what? What is MLH1 implicated in? Tons of blood loss has what compensation first?

SGLT2 inhibitor; blocks reabsorption of glucose in the PCT; UTIs/yeast infections, hypotension

Canagliflozin is what type of drug? Mechanism of action? Side effects?

Plasma cell; Amyloid from immunoglobulin light chains; Inhibits inflammatory response; EML4-ALK tyrosine kinase in lung adenocarcinoma

Cell shown? Besides CRAB, what else is seen in multiple myeloma? What does TGF-B do? Target of crizotinib?

DNA is transcribed to RNA which is translated to protein; Alternative splicing; Latent period where it takes time for a symptom or benefit to show up

Central dogma of replication? A receptor is shorter and lacks a transmembrane domain, what happened? Less strokes in men who used antioxidants for >5 years versus men who took them for <5 years indicates what phenomenon?

Dysdiadichokinesia & intention tremor; Truncal ataxia; C7/8; Pure motor weakness of face, arms & legs; External=decreased movement, internal=increased; Autonomic ganglia; Gentifemoral; L1-L2;

Cerebellar hemisphere sign (2)? Vermis? Triceps reflex? Internal capsule signs (3)? Damage to external versus internal globus paldus? Horner's w/ arm involvement is a lesion where? Cremasteric reflex is what nerve? Roots?

Charcot-Bouchard causes intraparenchymal hemorrhage while hypertensive crisis causes lacunar infarcts; Strep viridans; CNIII; Shock=hyporeflexia then progresses to UMN symptoms; Bilateral pontine lesions

Charcot-Bouchard vs hypertensive arteriolar crisis? Solitary ring enhancing lesion after sinusitis is what? Uncal herniation damages what nerve? First manifestation of spinal injury? Pinpoint pupils are due to what?

They breakdown lipid membranes so target the viral envelope; No, Hepatits A is a picornavirus which are non-enveloped

Chloroform, diethyl ether and other organic solvents target what aspect of viruses? Does Hepatitis A have this component?

Choanal atresia will not have drooling; Inability to produce breast milk; Pair of alleles is more frequently inherited together than would be expected based on their individual allele frequencies; Ascending= bleeding/iron deficiency anemia, sigmoid=obstruction

Choanal atresia versus TEF sign? Lactose synthase deficiency would cause what? Linkage disequilibrium? Ascending colon versus sigmoid cancer presentation?

Huntington's disease

Chorea, mood abnormalities, impulsive behavior and agitation indicate what?

Bronchilitis obliterans; Vascular obliteration; Bevacizumab; Alemtuzumab CLL; PCT; Dyspnea, altered mental status & petechial rash; Upregulates beta adrenergic receptors

Chronic rejection in lung transplant manifests as what? Kidneys? Monoclonal antibody against VEGF? CD52 & use? Where is the majority of water reabsorbed regardless of hydration status? Triad of fat embolism? How does hyperthyroidism cause afib?

Adenosine deaminase; Purine; Mom with O blood has IgG anti-A & anti-B which can cross the placenta, type A & B blood are IgM so baby is fine; Deficiency in NADH reductase to convert Fe3+ back to Fe2+

Cladribine is resistant to degradation by what? What is it an analog of? What happen sin ABO disease? Congenital methemoglobulinemia is caused by what?

Lip=Maxillary prominence w/ inter maxillary segment, palate=palatine shelves; 5-6; Micrognathia causes tongue to interfere with palatal closure; Opioid antidiarrheal that slows motility; Combined w/ atropine; Th2

Cleft lip versus palate fusion failure? What week? What is Pierre-Robinson sequence? What is diphenoxylate? How is abuse discouraged? What cells are involved in ulcerative colitis?

Velocardiofacial syndrome; Line where the rectus abdominis dives posteriorly; Inferior epigastric arteries; Dopamine agonist in prolactinoma or GH secreting tumors; Shoulder pain from peritoneal irritation

Cleft palate, hypertelorism & TOF is what? What is the arcuate line?Transverse C-section can injure what? Cabergoline mechanism and use? Kehr sign?

Microcytic anemia; shortness of breath

Colon cancer presents with what hematologic manifestation? How might this manifest?

Vitamin B12 deficiency; Pernicious anemia; Hypersegmented neutrophils due to autoimmune attack

Conjunctival pallor, bilateral brisk patellar reflexes, slightly ataxic gait, and impaired position and vibration sense in the feet bilaterally are signs of what deficiency? What hematologic disorder causes this? What would you see on blood smear?

Choriocarcinoma

Continually elevated B-Hcg levels after removal of a hydatidiform mole indicate what?

Methionine; Vitamin B12; Mitral valve prolapse & valvular sclerosis, not calcification; Pulmonary hypertension

Conversion of homocysteine to what require tetrahydrofolate? Cofactor? What valvular pathology most commonly predisposes to infective endocarditis (2)? Enlarged coronary sinus indicates what?

Myocardial hibernation; Capitation (HMO); Both prokaryotes & eukaryotes

Coronary revascularization that improves contractility and ejection fraction is due to what phenomenon; Employers paying a monthly fee to cover heath care for all employees has what 2 names? What species are Okazaki fragments found in?

Mycobacterium avium complex (MAC); clarithromycin & ethambutol

Cough, fever, RUQ pain, & fatigue in a HIV with lymphadenopathy and hepatosplenomegaly indicate what infection? Treatment?

Bronchiolitis from RSV, pneumonia does not have wheezes

Cough/congestion followed by diffuse wheezes & crackles w/ increased work of breathing indicates what diagnosis & infection?

Moxi does not cover pseudomonas; ATP-gated; cyclic-nucleotide gated; Sulfatide; Reduction in ERV; In athletes

Coverage of moxi versus levo & cipro? CFTR channel is what kind? Photoreceptor/olfactory? What TB virulence factor allows for intracellular replication? What pulm function test indicates obesity related disease? When would FEV1 & FVC be increased?

One of the ANCA's as they are pauci-immune; Scaphoid & lunate; I-antigen; At FRC; -5; No the bronchial & thesbian (heart) veins dump some deoxygenated blood into the oxygenated pulmonary veins

Crescent glomerulonephritis with no immunoglobulin or complement deposition is what? What two hand bones are injured in a FOOSH? What antigen gives mycoplasma it's cold agglutination properties? When does airway pressure=0? What is intrapleural pressure at this point? Is all blood from the lungs oxygenated?

Uridine monophosphate synthase; Orotic aciduria because homocystinuria does not present with anemia; No, it is refractory to folate treatment

Crystal in the urine with a megaloblastic anemia indicate what enzyme deficiency? Diagnosis? Does folate help the anemia?

Leiomyoma; Glandular & stromal tissue; Chocolate cyst; Surgery (OCPs for those that don't)

Cycle irregularity since child birth, increased bleeding & a mass originating in the deep endometrium indicates what? Histology of the tissue in endometriosis? Chronic pelvic pain & right adnexal homogenous cystic mass indicate what? What is the recommended treatment for women who want to get pregnant?

Pyrimidine; Purine; Pyrimidine; PTT is prolonged in antiphospholipid; Drug that prevents water/K+ efflux from cells in sickle cell; Thrombin; Chronic hemolysis, not EPO deficiency; Hemophilia A b/c it increases factor VIII

Cytarabine is an analog of what? Fludarabine? Gemcitabine? Labs that distinguish antiphospholipid syndrome from Factor V Leiden? What does Gardos do? Treatment for hemophilia? Extramedullary hematopoesis in liver/spleen is caused by what? Desmopressin treats vW disease & what else?

Amatoxin; RNA poly II in mRNA synthesis; Ricin; rRNA & tRNA synthesis; DNA poly I

Death cap mushrooms contain what toxin? What machinery & process do they inhibit? What toxin inhibits rRNA? Function of RNA poly I & III? What is the only DNA poly that has 5' to 3' exonuclease activity?

Phenoxybenzamine b/c it is irreversible whereas phentolamine is reversible so it would increase Km; Inhibits inward Na+ channels; L1-2; L4; Metalloproteinases

Decreased Vmax in a vessel pretreated with a drug before the administration of norepi indicates use of what drug & why? Ranolazine mechanism? What vertebral level do renal veins enter? What level does the aorta bifurcate? What enzyme causes plaque rupture?

Thiamine (wet beri beri) not Vitamin B12; Migratory thrombophlebitis; Pancreatic, lung & colon adenocarcinomas; Cerebral venous thrombosis

Decreased light touch, vibration and DTR's with HF & peripheral edema indicates what vitamin deficiency? Linear erythema & tenderness indicates what? What 3 cancers are associated with this? Hyperthyroidism causes what hematology manifestation?

L5 radiculopathy

Decreased sensation on the dorsum of the foot indicates what?

Phenotypic mixing=2 viruses infect one cell resulting in genome of one virus with capsid of another; recombination=exchange of non-segmented, double-stranded DNA via crossing over of homologous sections

Define phenotypic mixing & recombination?

Primary=preventing a disease (health promotion), secondary=interrupting diseases before symptoms develop (community screenings), tertiary=treating a disease to prevent complications/progression; Bax; Cytochrome C; TNF/TNFR1

Define primary, secondary & tertiary intervention. Bcl-s is replaced with what in apoptosis? What is a caspase activating substance? Besides the FAS/95 ligand, what other binding induces the extrinsic apoptosis pathway?

Excessive anxiety/preoccupation w/ more than 1 unexplained symptom; Neurologic symptom after a stressor w/o any identifiable cause; Malingering has secondary gain; >1 month vs 3 days-1 month; >1 but < 1 month; Hypocretin & orexin; Modafinil

Define somatic symptom disorder. Conversion disorder? Malingering versus facticious? PSTD symptom length vs acute stress disorder? Brief psychotic disorder timeline? Low peptides in CSF of narcolepsy patients (2)? Treatment for narcolepsy?

Type 1; GVHD is usually in bone marrow transplants, not solid organs (exception=liver); GVHD=donor T-cells against MHCII, acute=recipient CD8+T-cells; Type 2; Type 2

Dermatographism is what type of hypersensitivity reaction? In an organ rejection after 10 days, what differentiates Graft-vs-host & acute rejection? Types of lymphocytes in each? Hyper acute rejection is what type of hypersensitivity? Guillain-Barre is what type of hypersensitivity?

Syphilis=single painless, dark field microscopy; Herpes=multiple painful, PCR or tzanck; chancroid=single painful, PCR or gram stain; Klebsiella=painless w/o lymphadenopathy, Donovan bodies on biopsy

Describe the ulcers & tests for syphilis, herpes, chancroid & klebsiella

Epidermodysplasia verruciformis; Autosomal recessive; HPV

Diagnosis? Inheritance pattern? Infectious agent?

Caspofungin=cell wall, amphotericin=cell membrane; CMV colonies; RNA poly & reverse transcriptase; inhibition of RNA-dependent RNA polymerase

Difference between caspofungin & amphotericin location of action? Erosions and ulceration of the colon in an HIV positive man indicate what? In addition to inhibiting DNA polymerase what else does foscarnet inhibit? Mechanism of action of Sofosbuvir?

Zero is same amount metabolized, first is same proportion metabolized; 3 bands=meiotic nondisjunction in meiosis I, 2 bands= II; Address them by their surname>ask if their first name is ok

Difference between first order and zero order kinetics? Downs Western that shows 3 bands and 2 bands indicates what? If your intern is calling a patient by their first name what should you do?

Permissive=one hormone allows another w/ different actions to exert maximal effect, additive=combined effect is the sum of individual effects, synergistic=combined effect exceeds the sum; Increases expression of alpha-1 receptors

Difference between permissive, synergistic & additive? How does cortisol increase effects of ang 2 & norepi?

Duodenal ulcers=decreased delta cells (somatostatin), gastric ulcers=decreased parietal cells; Enteropeptidase in the brush border; Taurine & glycine; Bluish-black gelatinous; White squamous cell debris

Difference in mechanism of action of H. pylori in stomach versus duodenum? What activates trypsin in the gut? Two amino acids involved in bile acid conjugation? Cholesterol granuloma in ear would look like what? Cholesteotoma?

Restrictive=increased, obstructive=decreased; Restrictive=low, obstructive=high; Transudate in alveoli; Heart failure cells; Pancoast tumor; Superior sulcus; Mediastinum; Pancoast

Difference in respiratory rate in restrictive vs obstructive disease? Tidal volume? Acute MI with dyspnea shows what in the lungs? Chronic heart failure? Guy w/ shoulder pain, hand muscle atrophy & hemoptysis has what? Location? Where are masses in SVC syndrome located? Which one more commonly causes Horners?

Superior rectal vein to inferior mesenteric vein; Inferior rectal to internal pudendal to internal iliac; Internal are somatic so cause no pain; Psuedomonas; YES; Left gastric

Drainage of internal hemorrhoids (2)? External (3)? Difference in presentation? Granulation tissue in the ear with intact tympanic membrane is what infection? Is pseudomonas motile? Ulcer in stomach will bleed from what artery?

Olanzapine; Fluoxetine; ADHD, narcolepsy & binge-eating disorder; Bulimia has compensatory behavior; Multiple motor & 1 vocal tick; Either a motor or vocal tick, not both; Mania; ASD restricted language is always, selective mutism is just in certain situations/places; SSRI;

Drug for anorexia? Bulimia? 3 indications for amphetamines? Binge-eating versus bulimia? Tourette definition? Chronic tic disorder? What can all antidepressant induce? Selective mutism vs. ASD? OCD treatment?

Decreases; Clopidogrel; Blocks ADP receptors on platlets; Skin ulcerations because PE's usually arise in deep veins not superficial ones

During exercise, what happens to the overall systemic vascular resistance? What should be used in patient allergic to aspirin? Mechanism of action? Most common complication of varicose veins?

Left gastric and inferior mesenteric

During removal of the pancreatic head, branches from what two vessels should be ligated?

Lung versus breast cancer EDGF receptor; Melanoma & RCC; Just a prolonged bleeding time from a qualitative platelet disorder from uremic toxins; HIV, CMV & toxoplasma; HBx protein inactivates p53; It's a RNA virus that lacks a reverse transcriptase;

ERBB1 verus ERBB2? What two cancers in IL-2 used in? Bleeding from IV site in renal failure causes what change to PT, PTT, bleeding time & platelets? Mono can also be caused by what (3)? Two proteins involved in HCC from HBV? Why doesn't HCV integrate into the host genome?

Autoimmune hepatitis; liver biopsy w/ portal mononuclear cell infiltrate that invades the limiting plate, anti-smooth muscle antibodies are not diagnostic

Elevated liver enzymes with no history of alcohol use, negative viral screen & elevated IgG indicates what? What is the gold standard for diagnosis?

Insulin resistance causes lipolysis leading to increased free fatty acid levels which in turn propagate insulin resistance by impairing glucose uptake & increasing hepatic gluconeogenesis

Elevated serum levels of what substance contributes to type 2DM?

Alcoholism causes thiamine depletion and administration of dextrose can use up the remaining thiamine in the alpha-ketoglutarate dehydrogenase reaction leading to Wernicke encephalopathy

Encephalopathy develops in an alcoholic after administration of dextrose, what reaction is effected & why?

Polymyositis; Anti-histidyl-tRNA synthetase; Autoimmune hepatitis; Glucocorticoids have highest anti-inflammatory properties; Von-Hippel-Lindau

Endomysial mononuclear infiltrate with patchy necrosis is what? What is another name anti-Jo-1? What are anti smooth muscle antibodies seen in? What drug prevents cellular reaction in asthma? Sporadic renal mass should make you think of what disease?

Vitamin C; Calcium oxalate crystals

Enveloped shaped crystals in the urine indicate what vitamin excess? Name of crystals?

Glycosylase, endonuclease, lyase, polymerase, ligase; Gallstones & alcohol; Nystatin (swish & swallow); Diffuse esophageal spasms due to impaired inhibitory neurotransmission; M (microfold) cells at the base of Peyer's patches; Lack of enteric stimulation decreases CKK release; GTP

Enzymes in BER (5)? 2 most common causes acute pancreatitis? Treatment of oral candidiasis? Large amplitude contractions of the esophagus are what? What cells does shigella bind? How does TPN cause gallstones? What energy source does Gq use?

Anaphylaxis, not septic shock

Eruption of an Echinococcus granulizes cyst causes what life-threatening reaction?

Slapped-cheek rash w/ sparing of the nasoabial folds followed by a lacy, reticular rash that spreads over trunk and extremities; Only infects children under 2 years of age; Red blood cells in the bone marrow

Erythema infectiosum has what progression? How does roseola differ? Where does this virus proliferate?

Left gastric; Superior rectal; Paraumbilical; Short gastric, left gastroepiploic & pancreatic; Celiac artery; Inhibits RNA poly & 5' mRNA cap & amps up immune response

Esophageal varices arise from blood shunting through what vein? Anorectal? Caput medusa? 3 splenic artery branches? Where does it arise from? Besides IMP & hypermutation what are 3 other mechanisms of ribavirin?

They both increase, but Q increases to a greater degree; Urine PAH * flow rate/plasma PAH; Radial=anterolateral displacement, median=anteromedial displacement; Brachial artery runs with the median nerve

Even though V/Q ratio decreases from apex to base, what happens to V & Q independently? RBF=RPF/(1-hematocrit) but how do you calculate RPF? FOOSH causes supracondylar humerus fractures, difference in radial versus median nerve injury? What artery can be injured?

Lumen obstruction not bacterial infection; 65; GI tract, liver, pancreas, lungs, bladder & urethra; Spleen, kidney, ureters, internal genitalia; Hepatocellular carcinoma; Rho-regulatory proteins; Duodenal ones are rarely malignant

Even though a hyperplastic lymphoid tissue can cause appendicitis, what is the inciting event? Medicare age cutoff? Endoderm derivatives (6)?Mesoderm (4)? Hemochromatosis is associated with what cancer? C diff toxin damages cell membranes via what? What ulcers do not require biopsy?

Contracture; Rupture of previously closed wound due to inadequate contracture; Ileus, intraventricular hemorrhage, hypocalcemia & hypoglycemia; Hyoptension & pulm hemorrhage

Excessive MMP & myofibroblasts cause what? What is wound dehiscence? Side-effect of terbutaline in neonates (4)? Surfactant use (2)?

Aplastic anemia, short stature, hypoplastic thumbs, hyper/hypopigmented patches; Absent in sickle cell, highest in trait; Spleno/hepatomegaly; Ruxolitinib

Fanconi anemia 4 symptoms? Sickle cell trait versus sickle cell HbA levels? Myelofibrosis shows pancytopenia and what else (2)? JAK2 inhibitor drug?

Hypothyroidism

Fatigue, cold intolerance, & constipation in a patient with vitiligo are indicative of what?

Primary carnitine deficiency; hypoketotic hypoglycemia; medium chain acyl-CoA dehydrogenase deficiency; Vitamin C

Fatigue, difficulty walking, S3 gallop & low carnitine indicate what deficiency? What is the main blood work indicator? Same blood work w/o cardiac symptoms indicates what? What vitamin is necessary for carnitine synthesis?

Lacunar ligament; Femoral artery/vein; Parietal cells; GH & TSH; Ileum/colon releases peptide YY which inhibits ECLs; Somatostatin & prostaglandins; 5 weeks; Developmental field defect (one malformation causes the same in adjacent tissues); Dysplasia

Femoral hernias are lateral to what? Medial? Pertechnetate scan labels what? What two pituitary hormones does somatostatin suppress? What down regulates gastrin after a meal? 2 hormones that also do? Prosencephalon cleavage occurs when? Holoprosencephaly is what type of defect? Osteogenesis imperfecta?

Hydrops fetalis; Parvovirus B19

Fetal autopsy indicating pleural effusion w/ secondary pulmonary hypoplasia & ascites inidicates what? If the mom had arthralgias during pregnancy, what infection is this?

West Nile; Good history b/c it's a clinical diagnose, there are no screening tools; snRNPs; Dynein; Muscle hypertonicity; Huntington=deacetylation, Fragile X=hypermethylation

Fever w/ rash & flaccid paralysis is what? In suspected tetanus, what is most important? Spinal muscular atrophy is due to what deficiency? Kinin does anterograde transport, what does retrograde? Uncorrected congenital hydrocephalus causes what? How do Huntington's & Fragile X?

Salmonella type can cause diarrhea OR constipation

Fever, abdominal pain, constipation & maculopapular lesions on chest & abdomen indicates what?

Visceral larva migrans from Toxicara canis; Enzyme-linked immunosorbent assay

Fever, abdominal pain, wheezing & eosinophilia in a boy who recently got a puppy indicates what infection? Test to confirm diagnosis?

5-alpha-reductase, interfering with the conversion of testosterone to dihydrotestosterone

Finasteride inhibits what enzyme and production of what product?

Pulmonary fibrosis; Expiratory flow rate due to radial traction on the airways; Morning headaches, causes pulm HTN & right heart failure; High in trachea, increases in bronchi then decreases onward due to extreme branching

Fine crackles, finger clubbing & diffuse reticular opacities indicate what? What is increased & why? Main clue to OSA & main sequela? How does airway resistance change?

6 percent

For a non-twin sibling, what is the lifetime risk of developing diabetes given that a sibling is diagnosed?

T=troponin to tropomyosin, I=troponin-tropomyosin to actin, C=Ca2+ binding site; High potassium conductance & low sodium conductance

Function of troponin I, troponin C & troponin T? What two ions are responsible for the resting membrane potential of the cell?

GERD=eos & resolution w/ PPI, allergen=tons of eos w no resolution w/ PPI; PMS2; Obstruction of duct w/ stones; Obstruction; Pancreatic cancer; Ubiquitination of IkB;

GERD vs allergen esophageal disease (2)? Random lynch syndrome gene? Inciting factor in cholecystitis? Pale stool & dark urine is what? Add enlarged gallbladder and you have what? Main step in activation of NFkB by TLRs?

Iron because gastric acid reduces Fe3+ to Fe2+ which is the absorbable state

Gastrectomy can lead to what deficiency? Why?

Zollinger-Ellison syndrome; PPI & Abx as number one cause is H. pylori; Encapsulated organisms; Ach, gastrin & histamine; H/K ATPase proton pump; Recruits eosinophils & basophils

Gastrin levels rising in response to secretin is what? Guy with duodenal PUD is put on what two drugs? C3 deficiency predisposes to what? Parietal cells secrete HCL in response to what (3)? Common pathway for these? C3a function?

6 months; Serotonin agonist; Keratin; Fixation of complement by preformed circulating antibodies; B/t 8-10 (midaxillary) & 10-12 (paravertebral); Any surgical history is lymph vessel obstruction; Abscess & scarring, not inflammatory cancer; Entamoeba histolytica; Water contaminated with human feces

Generalized anxiety disorder timeline? Triptan mechanism? Desmosome composition? Tetanus booster leading to an arthus reaction is mediated by what? Midscapular thoracocentesis rib? Hx of ovarian cancer now has edema, why? Firm, dimpled area of breast after mastitis is what? Organism that eats RBCs? Source of infection?

Primary infection; Reactivation

Gingivostomatitis from HSV is what stage of infection? Labialis?

HUS; Microthrombi; Urine Cl- >20; This cause is not responsive to isotonic saline; Arterial is normal, venous has decreased O2, increased CO2 & decreased pH

Girl with bloody diarrhea followed by hematuria, anemia & thrombocytopenia has what? What causes this? How due you determine metabolic alkalosis due to hyperaldosteronism? Why is this important? Arterial and venous blood changes at peak exercise?

Posterior subcapsular cataract; Cushing's syndrome

Glare when looking into headlights or the sun indicates what? What endocrine syndrome is this found in?

Intravascular devices

Gram-positive organisms are now the leading cause of nosocomial bloodstream infections due to increased use of what?

JAK/STAT non-tyrosine kinase receptors

Growth hormone, prolactin and EPO act on what receptors?

MS because they're both immunologic and inflammatory

Guillain-Barre syndrome is most similar to what other disease?

Oral cavity, it is not contributing to disease; Cancer (renal cell or urothelial); Glycogen & lipid accumulation; Vertebral, anal atresia, cardiac, tracheoesophageal fistula, renal & limbs

Guy with fever, night whats & hemoptysis has sputum with Candida, where did it come from? Painless hematuria in the elderly is what until proven otherwise? What causes cells to look clear in CCC? What is VACTERL?

Pulmonary actinomyoces; The stains make the sulfur granules purple; Atherogenesis; Nocardia

Guy with poor dentation & hemoptysis has what? What does lung biopsy look like? What deleterious effect does Ang 2 have? Pulmonary symptoms with ring abscess in the brain and branching culture is what?

Cardiac tamponade from pericardial effusion; LV failure with pulsus alternans; Kussmaul's sign; Sotalol; Glossopharyngeal & vagus

Guy with viral infection and pulses paradoxus has what? What would you see if it was viral myocarditis? Chronic pericarditis? What beta blocker has class 3 properties? Afferent and efferent of baroreceptors?

Intestinal ischemia & renal artery stenosis; Fibromuscular dysplasia; Transverse carpal ligament; Mycolic acid; Gram-positive bacteria

Guy with weight loss & one atrophic kidney likely has what (2)? One atrophic kidney in a female of childbearing age is likely what? Cutting what structure fixes carpal tunnel syndrome? What does acid-fast specifically stain? Where is lipoteichoic acid found?

Stratified squamous epithelium; True vocal chords

HPV infects what type of cells? Besides the anus & vagina, where else are these cells found?

Adult T-cell lymphoma; Tropical spastic paraparesis; Breastfeeding, sexually or IV drug use; Lymphocytes with lobulated nuclei (Flower cells); Leukemia cutis; Rb, E2F

HTLV-1 most commonly causes what disease?Rare disease? How is it transmitted? Describe the pathologic cells seen (Name). What skin manifestation can this cause? Gene target (2)?

Atherosclerosis; Fenoldopam; D1 agonist; Increases renal perfusion; CD31; Osler-Weber-Rendu

HTN predisposes to aortic dissection, what predisposes to aneurysm? In a person in hypertensive crisis with AKI, what drug should be used? Mechanism of action? How does it differ from hydralazine? What tumor marker indicates endothelial cell origin? What cancer?

TIA; Aspirin; Ecological; Ecological fallacy

HTN, hypercholesterolemia & neurologic deficits that resolve in 20 min are what? In addition to HTN meds & a statin, what else would you add? If a study is using population data instead of individual data, what type of study is it? Making conclusions about individuals based on these studies is called what?

Thyroid lymphoma (B-cell neoplasm); Maxillary sinus; Collapsed w/o a lumen; MCV>100; PBC leading to Vit A deficiency;

Hashimoto's patient w/ enlarged thyroid has what? Orbital fracture leaks blood to where? What does the esophagus look like on CT? Pancreatitis w/ AST>>ALT, what else is seen in blood work? Woman w/ pruritus & night blindness has what?

HLA-DR5

Hashimoto's thyroiditis is associated with which HLA class?

Dengue hemorrhagic fever; Secondary with a different serotype; Aedes mosquito; Chikungunya; Much faster progression

Headache, retroorbital pain, join/muscle pain & hemorrhage indicate what? Primary or secondary infection? Vector? What other weird disease does this vector transmit? How does Ebola differ?

Q fever

Headache, retroorbital pain, myalgia, pneumonia, increased liver enzyme & thrombocytopenia indicate what?

Hemosiderin in the lungs; Lipofuscin; Anthracosis from smog inhalation; Adenosine & acetylcholine

Heart failure cells contain what and are located where? Yellow-brown cells in the heart contain what? Black pigment in the lungs is what? What two substance prolong phase 4 of the SA potential?

Portal vein thrombosis; Budd-Chiari; Zone 3; Bicarb; Ulcer, not malignancy; Lactose binding to pressor portein removes it from operator locus & increased cAMP binds CAP activator; Decreases cAMP levels

Hematemesis & splenomegaly with normal liver biopsy is what? Abnormal liver biopsy? What zone does acetaminophen effect? Secretin secretes what? New onset odynophagia w/ GERD history is what? Mechanism of lac operon (2)? What does glucose do to this system?

Renal cell carcinoma producing EPO; Cancer; CF because it is meconium ileus; Pneumonia; Enterocolitis

Hematuria, flank pain & increased hematocrit indicate what? Painless hematuria is usually what? Inspissated green fecal mass indicates what disease? Most common cause of mortality? What about in hirschsprungs?

3' to 5' exonuclease activity; DNA glycosylase; Sudan III stain tests the stool for fat; Affected ear in conductive, unaffected ear in sensorineural; Asparagine

Hep C lacks what? BER enzyme? Most sensitive screening for malabsorptive disorders? Weber test in sensorineural vs conductive? Chemo drug that decreases cell turnover targets what amino acid?

Classic galactosemia>fructose intolerance because of the milk component

Hepatomegaly, cataracts, vomiting & hypoglycemia after breastfeeding indicate what?

C1 inhibitor deficiency; Bradykinin; Loratadine is a newer gen antihistamine that doesn't cross the BBB and propagate cognitive dysfunction; Alpha 2; Preferential venous dilators, with minor arterial effects (nitroprusside does both)

Hereditary angioedema is caused by what? What molecule is increased? What do you use in an elderly patient with an allergic reaction? What receptor does epinephrine not act on? What do nitrates do?

Decreased 2,3-BPG; Burkitt's can present in immunocompromised; Indirect bilirubin from hemolysis, D-dimer is not elevated even though there are microthrombi; Osmotic fragility test; Sickle cell & hereditary spherocytosis

High O2 affinity in HbF is due to what? HIV patient with mass surrounding intestine w/ a high mitotic rate is what? Elevated lab in HUS? Acidified glycerol test is what? Parvovirus causes aplastic crisis in what two diseases?

Surreptitious use of diuretics

High bicarb with low potassium & normal sodium indicates what?

Hypokalemia due to cross reactions with mineralocorticoid receptors, U-wave on ultrasound

High dose glucocorticoid treatment causes what electrolyte & cardiac EKG manifestation?

Citrate; Calcium because it bind oxalate; Crohn's causes calcium to not be absorbed; Both reduced because RPF may initially increase with efferent arteriole constriction but eventually will decrease

High levels of what in the urine prevent stone formation? Eating more what helps prevent stones? What disease effects this? Ureteral obstruction causes what change in GFR & FF?

Type 1 (Type II can use aerobic); Postural maintenance; GTO's=in series for force of contraction via Ib, intrafusal=in parallel for muscle length via II & Ia; Tubulin; Tocilizumab

High myoglobin & mitochondria are what type of fibers? Main function? Difference between golgi tendon organs & intrafusal muscle fibers? What does colchicine bind? What is a monoclonal antibody against IL-6 effective in GCA?

Gout because myeloproliferative disorders have increased rate; Multinucleated giant cell with horseshoe shaped nuclei involved in granulomatous inflammation; Fibula; Obturator b/c it supplies the head proximal to the epiphyseal growth plate

History of a myeloproliferative disorder and a swollen knee indicates what? What is a Langhans cell? Where does the LCL connect to? What artery is clinically important in the hips of children but not in adults?

Pulmonary fibrosis; Parakeratotic strateum corneum w/ neutrophil clusters; Atelectasis; The lung; Bortezomib; Multiple myeloma & Waldenstroms; Coffin lid

Honeycomb lungs are what? What is a munro abscess? What causes a matched V/Q defect? At low PO2 what is the only organ that vasoconstricts? What monoclonal antibody is a proteasome inhibitor? Uses (2)? Shape of struvite stones?

EBV infects B-cells and causes them to proliferate continuously (immortalization); Their envelopes are acquired from the nuclear membrane instead of the plasma membrane

How can EBV cause B-cell lymphoma? What makes the envelope of herpesviruses unique from other enveloped viruses?

Pregnancy is possible with a donor oocyte, hormone replacement helps height

How can a Turner syndrome patient have children?

Lecithin sphingomyelin ratio in amniotic fluid

How can you determine the maturity of fetal lungs in utero?

Estrogen increase TBG levels, decreasing free T4 up regulating TSH and leading to overall increase in T4 pools

How do OCP's effect thyroid hormone levels?

(RIRI) Decreased renin, decreased insulin, increased renin, increased insulin; C3b & C1q; 4-5 half lives; Pseudomonas; G3b

How do a1, a2, b1 & b2 effect renin and insulin? Deposited substances in PSGN & MPGN? How long does it take to reach steady state? Man after chemo is neutropenic and presents with necrotic ulcers what does he have? What is another name for ceramide trihexoside?

Acetylation, phosphorylation or adenylation of the drug outside of the bacterium; Gram-negative rods but not anaerobes; Piperacillin-tazobactam>clindamycin

How do bacteria create resistance to gentamicin (3)? What is ciprofloxacin effective & not effective against? What drug should be used against a beta-lactamase producing bacteroides (2)?

Estrogen secretes cholesterol & progesterone causes gallbladder hypomotility; Arrhythmia; Granulation tissue with neovascularization; Wavy fiber, hypereosinophilia, neutrophilia; First 4 hours

How do estrogen & progesterone contribute to gallstone formation? SCA 12 days after MI is due to what? What does the tissue look like? What happens 1st, 2nd & 3rd? When does the tissue appear normal?

Increase dopamine & norepinephrine; Because of its long half life; Cardiac arrhythmia from Na+ channel blockade;

How do stimulants work? Why is methadone good for heroin withdrawal? Most common cause of death w/ TCA's and mechanism?

Metabolic acidosis=1.5 x HCO3+8(+-2); Metabolic alkalosis=PaCO2 should increase by .7 for every 1 rise in HCO3; Respiratory acidosis=HCO3 increases by 1 for every 10 rise in PCO2; Respiratory alkalosis=HCO3 decreases by 2 for 10 decrease in PCO2

How do you calculate appropriate compensation for metabolic acidosis? Metabolic alkalosis? Respiratory acidosis? Respiratory alkalosis?

Yeast cells inside of the macrophages in lung or bone biopsy; Hepatosplenomegaly; Intrapartum penicillin, not penicillin at 30 weeks gestation; CD8+ Tcells

How do you confirm histoplasma? What organ pathology does this cause? How can neonatal Group B Strep be prevented? Hypersensitivity pneumonitis has high what?

Pitt's usually has collapse of the vertebral body, spinal epidural abscess does not and usually mentions IV drug use

How do you differentiate Pott's disease from a spinal epidural abscess?

Exposure to saturated steam (121°C) for 15 minutes

How do you kill C. diff.?

Stable=methotrexate; Unstable=emergency surgery; Colposcopy; HPV DNA typing

How do you manage an ectopic pregnancy in a stable versus unstable patient? How do you follow a Pap smear with a low or high grade squamous intraepithelial lesion? Atypical squamous cells of undetermined significance?

Aspergillus will branch at acute angles with septa and Mucor or Rhizopus will branch at wide angles w/o septa

How do you tell the difference between the two organisms that can cause headaches, nasal discharge & paranasal sinus tenderness?

Vanc b/c so many of them are methicillin-resistant; Osteoid production by osteoblasts; Serums sickness; Decreased complement from antibody/complex deposition & neutropenia

How do you treat coagulase negative staff? What is wrong in OI? 7 days after administration of infliximab a man develops joint pain, pruritic rash & vasculitis, what is this? What can be seen in blood (2)?

Pharmacologic interventions to raise HDL does not improve outcomes, so lower LDL w/ statins; Niacin; 1C antiarrhythmics; False ST depression; Can mask ischemia because it lowers myocardial oxygen demand

How do you treat patients with low HDL? What drug increases HDL the most, but isn't used for that? Patient with QRS of 95 that goes to 125 with exercise is on what class of drugs? What can be seen on exercise test with digoxin? Verapamil & stress test?

Pertussis toxin inactivates Gi via ADP-ribosylation & loss of Gi-mediated AC inhibition leads to an overactive AC

How does Bordetella pertussis increase hydrolization of ATP to ADP?

Linear ulcers; Ganciclovir (guanine nucleotide analog)

How does CMV esophagitis present on endoscopy? What do you treat it with (description)?

Multifocal, higher grade, nonpolyp derived; Ballooning degeneration; Dark colored from increased conjugated bilirubin; -itant; KRAS; Beta-catenin; Meniere's; Increased endolymph in the inner ear; Acoustic neuroma; Preeclampsia w/ seizures; Diarrhea in AIDS patients

How does UC cancer differ from sporadic (3)? Hep A liver biopsy? Urine findings? Suffix of neurokinin inhibitors? Increase in size of adenoma is due to what? What does APC cause accumulation of? Episodic hearing loss w/ vertigo is always what? Etiology? Chronic? Eclampsia definition? Isospora belli causes what?

It is diffuse, noncystic and usually effects white matter

How does a fibrillary astocytoma differ from a pilocytic astrocytoma?

Alcohol increases acinar cell sensitivity to CCK stimulating trypsinogen production in the cell

How does alcohol cause chronic pancreatitis?

The combination of edema factor, lethal factor & protective antigen which on their own are non-toxic; Hyperacusis due to decreased tone of the stapes muscle; Involvement of the nose (Hutchinson's sign)

How does anthrax cause pulmonary edema (3)? What ear pathology does Ramsay Hunt syndrome cause & why? What distribution of this is associated with the highest risk of HZO (sign name)?

It causes vessel damage producing hemorrhagic infarcts and contrast-enhancing abscesses

How does aspergillosis present in the brain?

Congenital absence of the vas deferens & azoospermia

How does cystic fibrosis cause infertility in men (2)?

Blocks conversion of 11-deoxycortisol to cortisol

How does metyrapone block cortisol synthesis?

Retained phosphorus binds calcium; TPP-HUS have normal PT & PTT; CPPD, gout is usually toe; Monosodium urate; Factor VII

How does renal failure cause hypocalcemia? Difference between lab values in DIC & TPP-HUS? Swollen knee joint is more often what? Crystal name in gout? What Vitamin K dependent pro coagulation factor has the shortest half life?

Diphtheria toxoid induces IgG production against the B portion of the AB toxin

How does the vaccine for diphtheria work?

X-linked recessive

How is hemophilia A inherited?

Tay-Sachs=hearing loss & lack of hepatosplenomegaly

How would you diagnose Tay-Sachs over Niemann-Pick disease?

ALS because fasiculations are LMN and hyperreflexia is UMN

Hyperreflexia, fasiculations and muscle atrophy indicates what?

Acyl-CoA dehydrogenase deficiency or carnitine deficiency

Hypoketotic hypoglycemia indicates what?

Systemic mastocytosis (clonal mast cell proliferation); KIT tyrosine kinase; Thickening of the muscularis mucosa; Anywhere from mouth to anus; Granulation tissue; In acute pancreatitis; Glycogen-rich cuboidal epithelium; Columnar mutinous epithelium; Papillary projections

Hypotension, flushing & pruritic itchy rash with mast cells on biopsy is what? What is mutated? How can Crohn's cause obstruction? Where does Crohn's effect? Pancreatic pseudocyst is lined with what? When do these form? Cells seen in serous pancreatic neoplasm? Mucinous? Adenocarcinoma?

Low RPF & GFR w/ high FF b/c GFR is less low due to renin rescue; Glucorticoids as NSAIDs & colchicine are contraindicated in elderly; Actin cytoskeleton

Hypovolemia causes what GFR, FF & RPF? Elderly patient with gout flare should have what drug? Wiskott-Aldrich is due to a dysfunction in what?

A P450 enhancer, not inhibitor b/c it is metabolizing the warfarin too quickly; Metronidazole & TMP-SMX; Cytoplasmic tyrosine kinase, not rinse tyrosine kinase which is the one for insulin; Strep pneumo, H influ or any other encapsulated species due to asplenia; GI bleed

If INR is subtherapeutic what else is the patient taking? What two abx inhibit warfarin? What type of receptor is JAK2? Sickle cell patient suddenly dies, organism? Most common cause of microcytic hypochromic anemia?

Tell me more about your concerns>I won't; PAD; Vasodialates & decreases platelet activation via inhibition of PDE; The impairment is bilateral

If a patient does not want to know their diagnosis, what should you say? Use of cilostazol? Mechanism (2)? Ace inhibitors can worsen renal function on what condition?

This is Parvovirus B19 so her fetus is at risk for hydrops fetalis; Taxoplasma gondii; Rubella

If a pregnant woman comes in with this son, what is her fetus at risk for? What causes intracranial calcifications? Patent ductus arteriosus?

The most open ended question possible; Length time=mortality of disease, lead time=screening methods; Mean +/- z-score*SE; Retroperitoneal space because it comes out of there under the inguinal ligament

If a step question asks about initiating an interview what's the answer? Difference in length-time & lead-time bias? How do you calculate CI? If the cardiac femoral is punctured superior to the inguinal ligament during cath, where would the blood pool?

Flow is proportional to r^4 so root 4 of 1/16=1/2=50%; Resistance is inversely proportional to r^4; Selection bias; Chronic diarrhea and failure to thrive

If flow is decreased by a factor of 16, what percentage has the lumen decreased? How would resistance be calculated? Attrition bias is a form of what? What are two key words for SCID?

Gluconeogenesis

If fructose 2,6 bisphosphate levels are increased, what is prevented?

Low C-peptide, insulin could be high or normal so isn't useful

If someone was purposefully administering insulin when they didn't need it, which test would determine this?

Brachiocephalic; SVC; Subclavian vein; AR, MR & ASD; HCM & AS; holosytolic murmur at left sternal border

If the right side of the face and right arm are engorged, what vein is occluded? What if both sides were? Just arm? What 3 murmurs does handgrip increase? What 2 does it decrease? What sounds does a VSD make?

High; aldosterone except where increased blood volume causes increased renal blood flow & release of atrial natriuretic peptide; hypernatremia, hypokalemia & metabolic acidoses

In Conn syndrome, what are bicarb levels? Why can sodium be normal? What is seen in diarrhea (bicarb, potassium sodium)?

Respiratory failure due to a metabolic acidosis with a superimposed respiratory acidosis; PaCO2=(1.5 * HCO3-)+8[+/-2]

In DKA, what is the main cause of death & why? What is winters formula?

K capsule, heat-labile enterotoxins & lipid A component of LPS

In E. coli, what antigens cause meningitis, diarrhea & sepsis?

CREB binding protein

In Huntington's disease, glutamine causes dislocation of which protein from the nucleus?

Rickettsia rickettsii; Coxsackie A causes hand foot and mouth disease in younger patients & tertiary syphilis

In a 25yo who has wrist and ankle rash from camping, what is the most likely organism? What are the other two disease with hand and foot rash?

Through the coronary sinus; Wash hands with alcohol, topical mupirocin leads to candida colonization; They produce IL-6

In a biventricular pacemaker, where does the second lead go? How do you reduce central line infections? Why can you get fever an weight loss from a myxoma?

4, exposed w/ disease, exposed without disease & non-exposed for both; Preventable adverse event b/c malpractice is a legal determination; Unexpected death or serious injury within a hospital; Respiratory alkalosis

In a case-control study, how many study populations do you have? If a physician messes up, what type of error is it? What is a sentinel event? What metabolic derangement does increased altitude cause?

Intussusception due to viral lymphoid hyperplasia; knees curled to chest

In a child with a recent gastroenteritis who now presents with intestinal obstruction, what is highest on your differential? What position does the patient like to be in to relieve pain?

Smoking cessation; aspirin therapy & diet/exercise

In a diabetic patient, what is the single greatest life style modification that decreases mortality? What are the 2nd and 3rd most effective?

Abdominal ultrasound to look for cysts of tumors because this is peripheral precocious puberty

In a girl with precocious puberty and a negative LH response in a leuprolide stimulation test, what is the next best step in management?

Hydrocortisone because it decreases ACTH-->decrease in androgens; switch to OCP's when older

In a girl with symptoms of 21-hydroxylase deficiency, what is the pharmacologic treatment? How does this change as she ages?

Chlamydophila pneumoniae; Hoarseness; Formation of atherosclerotic plaques; Gram negative

In a man in a nursing home what is the most common cause of pneumonia? Most common physical exam finding? What other pathology is it associated with? Gram stain?

SSRI or gabapentin

In a menopausal woman with a history of breast cancer, what is the best treatment option (2)?

Sulfonamides displace warfarin from albumin so you should reduce warfarin dosing to avoid uncontrollable bleeding

In a patient on warfarin, what should you be cautious of with administration of a sulfonamide?

Amylase, this is suppurative parotitis from Staph aureus; dehydration & anticholinergics; elevated gamma-glutamyltranspeptidase

In a patient who had endotracheal intubation followed by jaw pain with fever and pre auricular swelling, what serum marker will confirm diagnosis? Two other risk factors besides intubation? If alkaline phosphatase is elevated, what indicates liver pathology over bone pathology?

Toxoplasma=immunocompromised & usually multiple lesions, Taenia solium=immunocompetent & single cyst

In a patient who present with seizures and a cyst is found, what differentiates Toxoplasma from Taenia solium?

Topical emollient until further testing is completed

In a patient whose CC is dry skin but also has signs of hypothyroidism, what treatment should be initiated?

Subacute combined degeneration due to Vitamin B12 deficiency; vacuolar myelopathy

In a patient with Celiac disease and a sudden neuropathy, what is highest on the differential? What disease presents identically?

Acquired immunodeficiency syndrome (HIV)

In a patient with Cryptosporidium who presents with severe diarrhea, what is your most likely diagnosis?

Restriction along ventricular linings is diagnostic of CMV encephalitis

In a patient with HIV, what is the most likely diagnosis?

Lack of jaundice as pancreatic adenocarcinoma has painless jaundice; yes b/c MEN1 can have Zollinger-Ellison syndrome

In a patient with MEN1 and epigastric pain what would indicate that they do not have pancreatic adenocarcinoma? Can MEN1 present with gastrinomas?

This is a statin so myopathy; TSH because myopathy is a common presentation of hypothyroidism

In a patient with a drug-eluting stent after a recent MI, what is the most common side effect of the drug? What else should be checked in patients whit this symptom?

Free T4 to confirm hypothyroidism, then anti-thyroid peroxidase antibody

In a patient with an elevated TSH, which test should be ordered next?

Salt retention because it's regulated by aldosterone which is not in the pituitary and won't be influenced by mass effect, not free water absorption because that's mediated by ADH

In a patient with galactorrhea, vision changes & loss of libido, what is preserved?

Fructose because of inhibited glycolysis; not F-1-P because F-1-P would be elevated inside of the cells, not in the serum

In a patient with hereditary fructose intolerance, what compound would be elevated in the serum?

Thrombopoietin (TPO), made in the liver and responsible for stimulating platelet development

In a patient with jaundice and icterus, reduction of which hematopoietic factors would be most concerning?

Avoidance of alcohol because it directly damages heart muscle and raises blood pressure, the other interventions would be appropriate if there was evidence of atherosclerosis

In a patient with no evidence of atherosclerosis, which of the following should you recommend: aerobic exercise program, avoidance of alcohol, ingestion of more vegetables and decrease in red meat intake, isometric/weight-training exercise program or weight loss?

Painful lymphadenitis instead of nontender & rubbery which indicates syphilis; Griseofulvin; Flucytosine; Caspofungin; Nystatin

In a patient with painless ulcer & inguinal lymphadenopathy, what points towards Lymphogranuloma venereum? What antifungal inhibits mitosis? DNA/RNA synthesis? Cell wall inhibition? Ergosterol inhibition?

Both eyes will dilate (they're actually still constricting just to a lesser degree so it looks like both are dilating)

In a patient with suspected MS and complaints of right eye vision loss, what will happen in a swinging flashlight test of the right eye?

Induration ≥5 mm; Early diagnosis

In a person w/ known recent contact to someone with active TB, how big does the PPD induration need to be to be positive? What is the most important prognostic factor for Pott's disease?

Elevated in systole & diastole; Tranthyretin; Dystrophin; Increases SVR to shunt blood back to the right via the VSD

In a pressure volume tracing, what does mitral stenosis look like? Hereditary cardiac amyloidosis is in what gene? Hereditary dilated cardiomyopathy? What does squatting do in tetrology of fallot?

Neutrophilic versus lymphocytic infiltrate; BRAF V600E; Vemarufenib; RPF b/c it is a substance that is only filtered & secreted, not reabsorbed; Antibody-dependent cell-mediated cytotoxicity via MBP; IL-5; Late-phase type I hypersensitivity cytokine release

In acute graft rejection can what would the humoral response look like? What mutation is seen in melanoma? Monoclonal Ab inhibitor? What does substance PAH estimate? How do eosinophils kill things? What substance activates eosinophils? Besides parasites, what else do eos help with?

Gastric=acute, pneumonitis with respiratory distress, no fever, no abscess; Bacterial=chronic, pneumonia, fever, abscess

In an alcoholic with foul smelling sputum, what is the difference in presentation between aspiration of gastric contents & oropharyngeal bacteria?

This is mitral valve stenosis caused by Strep pyogenes due to untreated strep throat from childhood

In an elderly person who immigrated to the US and now has a diastolic murmur with an opening snap, what organism is responsible?

Decreased due to consumption to create serotonin

In carcinoid syndrome, are tryptophan levels increased or decreased?

BUN decreases and PTT increases

In chronic liver disease, what happens to BUN? PTT?

Hexokinase converts fructose to F6P, bypassing fructokinase; UDP-galactose-4-epimerase

In essential fructosuria, what enzyme causes patients to be asymptomatic? What replenishes the UDP used in galactose metabolism?

ATP from glycolysis of glucose

In insulin release, what causes closer of the K+ channel?

Involvement of the suspensory (Cooper) ligament; Upper outer quadrant; Inflammatory breast carcinoma

In invasive breast carcinoma, what causes skin dimpling? Where is the primary mass located? Dimpling caused by blocked lymphatics is caused by what malignancy?

Anisocytosis; Poikilocytosis

In iron deficiency anemia, what fancy word is used to describe the cells? How about in hemolytic anemia?

Adhesion to collagen, evasion of opsonization & multinucleate giant cells

In osteomyelitis, what is the mechanism of S. aureus, Salmonella & mycobacterial?

Their occupation as this could be anthrax

In patient with a black eschar ulcer, what would be most helpful in knowing?

Absolute=increased total RBC mass, relative=normal RBC mass; Dehydration & excessive diuresis; Contralateral hip; Superior lateral quadrant

In polycythemia, what differentiates an absolute form a relative erythrocytosis? What causes relative (2)? Where is the hip drop in tredelenburg gait? Where you give hip injections?

Thiazide diuretics; Stones, bones, moans, groans

In someone being treated for hypertension, diabetes and coronary artery disease, what drug that they are taking could cause hypercalcemia? Symptoms pneumonic?

Increase dosing because pregnancy increases the metabolic demand for thyroid hormones, it does not cause hyperthyroidism in people with hypothyroidism; increases it

In someone on levothyroxine who is now pregnant, how should you change their medication & why? What does estrogen due to T4?

Fibrin/platelet aggregates; Subendothelial collagen & GAGs; Lysozyme; Neutrophil specific granules; Cardiac & smooth muscle have calcium induced calcium release while muscle sarcoplasm calcium release is technically-coupled & independent of extracellular calcium

In subacute endocarditis, what do the bacteria latch onto (2)? Exposure of what two things in a plaque causes platelet aggregation? What enzyme is in tears & mucous? Where else is it found? Why doesn't verapamil effect muscles?

Volume overload, metabolic acidosis & hyperkalemia; Hypokalemia & dehydration; Egg-shell calcifications & birefringent particles; Noncaseating granulomas

In the maintenance stage of ATN, what can happen (3)? Recovery phase (2)? Two buzzwords associated with silicosis? Histology of hypersensitivity pneumonitis?

Glutathione antioxidant mechanism, cholesterol synthesis & fatty acid synthesis; 2 ATP molecules, no NADPH

In the pentose phosphate shunt, G6P is converted to ribulose-5-phosphate. The substance generated in this conversion is used in what three processes? What does the urea cycle use?

Impaired optic nerve axoplasmic flow; Physostigmine; GABAb spinal cord muscle relaxant; Acetylcholine; Basal nucleus of Meynert & hippocampus; Nucleus ceruleus; Inferior gluteal; Urinary incontinence; Presynaptic decrease of Ca2+ influx & postsynaptic K+ efflux

Increased ICP causes what in the eye? Atropine reversal? What is baclofen? Decreased neurotransmitter in alzheimer's? Two locations? Brain location of panic attacks? Difficulty rising from a seat is what nerve injury? Besides leg stuff, what else is seen in ACA stroke? 2 actions of opioids on the spinal cord?

Fabry's disease; alpha-galactosidase A

Increased ceramide trihexoside level, history of heart failure, painful stocking-glove neuropathy & blue-red spots on his thighs and abdomen indicates what? What is the deficiency?

Glucose-6-phosphatase because it converts glycogen to glucose & fructose to glucose

Increased hepatic glycogen w/ normal structure and no increase in serum glucose concentration after oral administration of fructose indicate what deficiency? Why?

Nephrogenic diabetes insipidus from lithium; Give desmopressin which shouldn't do anything unless its central diabetes insipidus; Furosemide

Increased thirst and urination with a high serum osmolality and bi-polarism indicates what? How can you test this? What would it be with low serum osmolality?

Peak serum concentration

Increasing a drugs bioavailability increases which other property?

delta-ala synthase; Delta-ala dehydrogenase & ferrochelatase; Inhibits pyridoxine phosphokinase; Fibrosis & atrophy due to repeated infarction, congestion is in liver cirrhosis; MCV>110 due to impaired DNA syn; IgG

Inherited sideroblastic anemia & Vit B6 deficiency effect what enzyme? Lead poisoning (2)? How does isoniazid cause B6 deficiency? Spleen histology in sickle cell? What defines megaloblastic? What class if RHOgam?

Epsilon; Two alpha & two delta; Increased CO2 in periphery->decreased pH & right shifts curve; Decreased CO2 lungs left shifts curve; PD-1; CTLA-4; APL version of AML; Arthritis, nail pitting & uveitis; Macrolides

Initial fetal globin? HbA2 strucutre? Bohr effect? Haldane effect? What ligand on T cells inhibits CD8+ response to cancer cells? What uses the same mechanism but binds B7? What cancer is treated with vitamin A? 3 psoriasis complications? Antibiotic that causes pyloric stenosis?

Diarrhea, not constipation; Vancomycin, linezolid & daptomycin; Optic neuritis, thrombocytopenia, serotonin syndrome; Myopathy; Selective

Intestinal graft rejection would cause what main symptom? What are the three common drugs used to treat MRSA? What are the 3 main side effects of linezolid? Side effect of daptomycin? What type of media is VPN agar?

Acidophilic=HSV, basophilic=CMV; One-sided nasal hemianopia; Bitemporal hemi; Homonymous hemi; Macular sparing; Substance P; Phenylketonuria; Ptosis, hypotension, hypoglycemia & hypothermia;

Intranuclear acidophilic vs basophilic inclusions? ICA aneurysm causes what sight manifestation?Acom? Anterior choroidal? PCA? Topical capsaicin decreases what? Pallor of substantial nigra in a 3 year old w/ seizures is what? 4 signs of dopamine hydroxylase deficiency?

Glyburide

Is glyburide or glipizide more likely to cause hypoglycemia?

TTP, HUS, ITP; Jaundice & splenomegaly; It looks like a myoglobin curve; HER2; BCL-2, estrogen & progesterone; Fibronectin, collagen & laminin; CLL

Isolated low platelets differential (3)? Two symptoms unique to hereditary spherocytosis? If hemoglobin is separated into monomers, what happens to the curve? Aggressive breast tumors are positive for what? Favorable prognosis (3)? Interns bind what (3)? Deletion 13q is what?

Aortic stenosis; Increased sympathetic tone; Transmural inflammation; Marfan's syndrome; Aortic regurg; Aortic valve closure

Isolated systolic hypertension is due to what? Combined diastolic & systolic? What is the pathogenesis of AAA? Cystic medial necrosis is seen in what? A cardiac cath has both aortic & LV pressures elevated, what is this? When is the murmur the loudest?

Pityriasis; Underlying malignancy or HIV infection

Itchy, rash that began on scalp/face & spread to chest and abdomen with painful palms and soles indicates what? What is it a sign of (2)?

Biliary atresia; Primary biliary cirrhosis; Pruritus & fatigue; Diffuse fibrosis w/ extrahepatic duct involvement too; Ballooning degeneration->apoptosis->shrinkage->Councilman bodies (eosinophilic);

Jaundice, dark urine & acholic stools in a newborn is what? Granulomatous destruction of the intrahepatic bile ducts is what? Two main signs? How would PSC differ? Viral hepatitis (4)?

C=oval, T=gracilis & cuneatus, L=just gracilis, S=tons of grey matter; Hypoxia-ischemia induced; Hippocampus; Perivenular inflammatory infiltrates; GABA; Upper is hyporeflexic, lower is hyperreflexic

Know the shapes at different levels. Bilateral wedge shaped brain infarcts are what? Cerebral ischemia damages what first? Pathology of MS? Decreased what in huntingtons? Reflexes in syringomyelia?

RccAUG (R=A or G); Elongation factor 2 requires GTP; Hypochromic, microcytic; (HALT) HbC, Asplenia, Liver, Thalasemmia; Biliary tract infections

Kozak consensus sequence in eukaryotes? What catalyzes translocation & what does it use? 2 characteristics of RBC's in beta thal? What causes target cells? Black stones indicate hemolysis, brown stones indicate what?

Inguinal; Popliteal & inguinal; Conjugated polysaccharide; Increased lymphatic drainage; HCM

Laceration on the medial foot drain to which lymph nodes? Lateral? What type of vaccine is Hib? In a patient with COPD, what mechanism is preventing them from having edema? A bifid carotid pulse is seen in what?

Mitral valve prolapse; Autosomal dominant; Tricuspid regurg from infective endocarditis

Late systolic murmur, heard loudest at the apical region & preceded by a click indicates what? How is this inherited? A woman who developed a new faint systolic murmur heard over the lower left sternal border w/ distended neck veins & hepatomegaly indicates what?

Mitochondrial inheritance; Anterior thigh feeling, thigh flexion & leg extension; Femoral; Increased LPL & decreased hepatic VLDL production; Fish oil; PCSK9 inhibitors

Leber hereditary optic neuropathy is inherited in what manner? Mass in your iliopsoas causes what nerve manifestations (3)? Nerve effected? What does increasing PPAR do in fibrates (2)? What else works this way? What reduces LDL receptor degradation?

Kaposi's sarcoma; Bacillary angiomatosis; Pyogenic granuloma

Lesion with increased spindle cells in the dermis that have invaded the collagen, forming slits with RBCs inside & many lymphocytes indicates what? What if it were neutrophils instead of lymphocytes?Neutrophils at the surface instead of scattered indicate what?

Multinucleate giant cells w/ intranuclear inclusions; Urge incontinence due to loss of inhibition of detrusor; Midline to your arm pit not your sternum; PAH via arterial thickening; Accentuated 2nd heart sound

Light microscopy of a shingles lesion would show what (2)? NPH leads to what type of incontinence? What does midaxillary mean? What pulmonary manifestation does CREST cause? Heart sound?

Trasversalis fascia, internal oblique, external oblique, external oblique aponeurosis, transversalis fascia

List the origins of internal spermatic fascia, cremasteric muscle, external spermatic fascia, superficial inguinal ring & deep inguinal ring.

0-4=dichorionic, diamniotic, 4-8=monochorionic, diamniotic, 8-12=monochorionic, monoamniotic & >13=conjoined; Proliferation of cytotrophoblasts & synctiotrophoblasts; Moles, choriocarcinomas have no villi

List the results of monozygotic twins when division occurs at 0-4 days, 4-8 days, 8-12 days & >13 days; A woman has vaginal bleeding and hemoptysis 9 weeks after delivery, what does histology show? What has hydropic chorionic villi?

Cholesterol thrombus; Glucocorticoids to mature type 2 pneumocytes; Dermal atrophy; Bowman's space because it's secreted in PCT

Livedo reticularis after stent placement indicates what? In PROM what should you give the baby to increase survival? Treatment of eczema with corticosteroids will show what on biopsy? Where is the concentration of PAH lowest in the kidney?

Abetalipoproteinemia; low triglycerides & cholesterol; no ApoB-100 or B-48; Retinitis pigmentosa

Loss of night vision, cerebellar ataxia & steatorrhea w/ star like RBC's indicate what? Lipid profile? Missing enzymes? What eye pathology develops?

Insulinoma; MEN1

Low blood glucose indicates what type of tumor? What is the major risk factor of this tumor?

Prominent germinal centers of B-cells; Fever & muscle aches (influenza-like); Yersinia=from food & no pus, shigella=fecal-oral & pus present

Lymph node biopsy in cat scratch disease would show what? What are the symptoms of pontiac fever (2)? In a child at daycare with bloody diarrhea what history differentiates shigella from yersinia (2)?

Beta & alpha globulin; Beta-2; Sarcoid is non-caseating, TB is caseating; Procollagen peptidase which cleaves N-terminal; Triple helix formation forms procollagen which is exocytosed for terminal cleavage to tropocollagen, spontaneous triple helix formation & lysyl oxidase cross linking

MHC 2 has what chains? MHC 1? Sarcoid versus TB? What enzyme and function are deficient in Ehlers-Danlos? In collagen synthesis what occurs extracellularly?

Weakness in transversalis fascia, not rectus abdominis; Metaplasia; All low; Anterograde transport towards plus end of microtubules; Megaloblastic anemia from inhibition of methionine synthase; Hyperkalemia; Sleep, interest, guilt, energy, concentration, appetite, psychomotor, suicide

Main cause of direct inguinal hernias? Longterm endotracheal tubes cause what? pH, PCO2 & HCO3 in aspirin poisoning? Kinesin function? Heme complication of nitrous oxide? Main succinylcholine side effect? SIGECAPS?

Agranulocytosis; Temporalis, masseter & med/lat pterygoid; Psuedostratifed columnar epithelial; Decreased motility; Physician makes decisions for patients instead of providing them with options; Waterhouse-Freidrich syndrome; Left renal; Kidney stone stuff, not leukocytes;

Main side effect of clozapine? 4 muscles of mastication? Smoker w/ bronchitis is what cells? Diabetic w/ diarrhea is by what mechanism? What is paternalism? Bilateral flank pain after meningitis is what? TIPS connects the portal vain to what? Woman with hydronephrosis will have what in the urine?

Chlamydia trachomatis because it does not gram stain

Man with dysuria, urethral discharge but only neutrophils & no organisms on culture indicates what?

Amlodipine/nifedipine b/c ACEI cause angioedema not peripheral edema; Loop diuretic; Transcription factors; CYP2D to endoxifen; LMWH

Man with hypertension goes a med & develops peripheral edema, what is the med? What is torsemide? What do hox genes code for? What is tamoxifen metabolized by and to? What is enoxaparin?

Anti-neutrophil (cANCA); I will not inform him if that is his preference (trying to speak to the son about influencing his fathers decision apparently doesn't respect their culture)

Man with increased creatine & non healing nasal ulcer has what antibody? A Mandarin elder doesn't want to know his diagnosis and it seems his son is deciding this for him, what do you say?

Hantavirus; Bunyavirus; SS-, circular, 3 segments

Man with pneumonia who owns rodents likely has what? Viral family? RNA structure?

Increased potassium conductance; Nucleolus; Inferoateral to the pubic tubercle; Korotkoff sounds; Frameshift because nonsense wouldn't be able to be seen in the cDNA b/c it reflects the mRNA transcript; No only missense mutations can

Mechanism of adenosine? Where is RNA made? Where is the great saphenous vein found? What sounds are heard in pulsus paradoxus? Gel electrophoresis on cDNA and the mutant is shorter than the wild type indicates what mutation? Can deletions be nonsense mutations?

Acetylcholine from preganglionic sympathetic fibers

Medulla chromaffin cells are activated by what?

Oxidation of iodine; Peripheral conversion of T4 to T3

Methimazole directly inhibits what process? Propranolol?

Eastern equine encephalitis; western equine encephalitis; mosquito

Middle-aged patient with fever, muscle aches, altered mental status, photophobia & seizures indicate what? What if this was in an elderly person or child? Vector for both?

Metformin=inhibits mitochondrial enzymes, GLP-1=G-protein adenyl cyclase

Molecularly, how do metformin and GLP-1 agonists exert their effects

Normocalcemia with hypercaluria; High protein diets; Desquamination of skin & diarrhea; Dystonia; Spasmodic torticollis; Blepharospasm; Writer's cramp

Most common blood and urine findings in kidney stones? What causes hyperurocosuria with normourisemia? Two main signs of GVHD? Sustained, involuntary muscle contractions is termed what? What is it called when it involves the neck muscles? Eyelids? Hand?

Haemophilus ducreyi; Ceftriaxone or azithromycin

Multiple painful penile ulcers with tender inguinal lymphadenopathy indicates what infection? Treatment (2)?

Intracellular calcium & epinephrine-induced cAMP increase; Epinephrine & glucagon increase cAMP; Liver=used in fasting state, muscles=for muscle contraction

Muscle phosphorylase kinase is activated via what two mechanisms? What about liver phosphorylase kinase? When is glycogen used in the liver versus the muscles?

BMPR2; Junctional are just in epidermis, compound extend into the dermis; CMV

Mutation for PAH? Difference between compound & junctional nevus? Pulm infection after lung transplant is what?

1=CN 4, 2= CN 3, 4=ICA, 5=pituitary, 6=CN 6, 7=CN V ophthalmic, 8=CN V maxillary; Antisocial disorder; G1 to S; Lacunar infarcts; Hypertension & diabetes; Rhomboids & levator scapulae; L4

Name 1, 2, 4, 5, 6, 7, 8; Criminal who threatens physician if they return him to jail has what? Rb gene prevents what phase? Cavities in deep brain filled with clear fluid are what? Two causes? Dorsal scapular nerve innervates what (2)? Landmark for lumbar puncture?

A=pineal body, L=olfactory, N=Infundibulum, O=mammary body, Q=CN 3, R=CN 5, S=CN 6, T=CN7, U=CN 8, VCN 12, W=CN 9, X=CN 10, Y=CN 11; Arch 3; Hypothermia & sedation; APP on 21, Presenilin 1 on 14 & Presenilin 2 on 1; ApoE4; ApoE2

Name A, L, N, O, Q, R, S, T, U, V, W, X, Y; Greater hyoid horn comes from where? 2 interventions to decrease metabolic demand of the brain? 3 proteins & genes that predispose to Alzheimer's? Apo protein? Protective Apo protein?

Acochordons; Increased estrogen; PLA2 (formation of AA); LTD4; Zileuton; 300, >300, <300, 100, 1000+

Name for skin tags? Spider angiomas are associated with what? Glucocorticoids inhibit what in the AA pathway? What does monteleukast inhibit? What blocks leukotriene formation? List the concentrations in the PCT, descending, ascending, DCT & collecting duct.

D antigen; +is hemolytic disease of the newborn, -is alpha thal; Kupffer cells; Heme oxygenate makes biliverdin; Biliverdin reductase makes bilirubin; Decrease because it's built up

Name of Rh antigen? Hydrops fetalis with + vs - coombs? Name for hepatic macrophages? What enzyme and product cause bruises to be green? Yellow? Vitamin B12 replacement would cause what change in methylmalonic acid?

1. Suprachiasmatic nucleus 2. Accessory optic nucleus 3. Lateral preoptic nucleus 4. Pretectal area 5. Supraoptic nucleus

Name the hypothalamic nuclei associated with: 1. Circadian rhythms, sleep-wake cycle, melatonin 2. Eye movements 3. Heat regulation 4. Pupillary light reflex 5. Water balance, production of ADH/oxytocin

Suprarenal, Aorta, Duodenum, Pancreas, Ureters, Colon, Kidney, Esophagus, Rectum (SAD PUCKER); Gallstone ileus; Cholecystoenteric fistula; Lesser omentum; Hepatogastric & hepatoduodenal; Fetal umbilical vein;

Name the retroperitoneal organs (pneumonic)? High-ptiched bowel sounds, abdominal distention and air in biliary tree is what? How did this occur? Gastric banding must pass through what structure? What two ligaments make this? Falciform/round ligament contains what?

Hypercalcuria; Hyperoxaluria; Hypocitraturia; Hyperuricosria; Vitamin B6; Vascular/interstitial fibrosis

Nephrolithiasis risk factors of hyperparathyroidism? Crohn disease? Distal renal tubular acidosis? Gout? Renal stone formation due to hyperoxaluria can be treated with what? Chronic transplant rejection has what macroscopic appearance?

Loss versus dispersion of Nissl & shrinking vs swelling of cell body; Norepi; Pons/lat floor of 4th ventricle; Accessory nerve; Trapezius over head things; Ipsilateral eye; Trigeminal

Neuron death versus regen (2)? The locus ceruleus produces what neurtransmitter? Where is this located? What nerve runs through the posterior triangle? What muscle/action is inhibited? Problem with your MLF will cause failure to adduct what eye? What nerve nuclei is at the level of the middle cerebellar peduncle?

Velocity=length/time; Increases length constant & decreases time constant; Mesocortical inhibition->flat affect, mesolimbic enhancement->hallucinations, nigrostriatal inhibition->dystonia, tuberoinfundibular inhibition->prolactinoma symptoms; Penicillamine;

Neuronal velocity equation? What does myelin do to these? Name the 4 dopaminergic pathways & associations. treatment of Wilsons disease?

Leukocytoclastic vasculitis; Hep B & C; PCL; Type III; Joints because uricemia; Neutrophils

Nonblanching palpable purpura on the legs after penicillin is what? What two diseases are this seen in? Excessive posterior displacement means that what ligament is injured? Hypersensitivity pneumonitis is what type? What body part develops pathology in Lesch-Nyhan? What cells are responsible for this?

Influenza; Otitis media

Nonspecific fever, cough & runny nose in an unvaccinated child should make you think of what? What is the most common complication?

IP3 in the vasculature & cAMP in the heart; decreased insulin secretion & decreased intestinal motility; Maxillary artery (middle meningeal artery was inured)

Norepinephrine causes increase of what two substance in what two tissues? Stimulation of alpha 2 receptors by IV norepi causes what two manifestations? Fracture to the pterion injures an artery that branches off of what?

Myoclonus; Femoral crease; Ischial spine; Diaphragmatic paralysis; Trapezius & SCM; C1; Cerebral amyloid angiopathy; AVM; CN VII

Nucleus ambiguos in medulla damage causes what? Where is a femoral nerve block administered? Pudendal nerve block? Sequela of interscalene brachial plexus block? CN XI innervates which two muscles? What spinal nerve only carries motor fibers? Spontaneous recurrent lobar hemorrhage in elderly is what? In child? Tumor in the parotid damages what nerve?

Gap junctions to exchange nutrients with the Haversian canal; Rapid hydrolysis by plasma esterases; Drug induced lupus; Peroneal nerve

Osteocytes are connected to each other via what junctions? Drugs in anesthesia have short duration do to what metabolism process? Anti-histone antibodies are specific to what? Fracture of the fibular head would injur what nerve?

Physician education on trailing zeros; Ultrasound followed by radionuclide biliary scan if needed; Secrete alkaline mucus; Pancreatic epithelial cells; Secretin; Decreases gastric H+ secretion; CKK; Somatostatin; GIP

Overdosing errors can be avoided how? Imaging for cholecystitis (2)? What do Brunner glands do? Where else do the secretions come from? What chemical stimulates this? What does this chemical decrease? What do I cells secrete? D cells? K cells?

In renal insufficiency use a V/Q scan; Chronic bronchitis; Stimulates histamine release; Vasoconstriction & increased capillary permeability

PE are usually seen with CT angiography, what is a contraindication and what else can be used? Nickel miner with thickened bronchial walls & enlarged mucous glands & squamous metaplasia has what? What does C3b do? What do leukotrienes C4, D4 & E4 do (3)?

Glycogen synthase, not phosphorylase; IL-1; Supinator canal; Impaired finger extension; Same but w/ absent triceps reflex; Hemagglutinin; Horner's, opthalmoplegia, decreased maxillary sensation; Spinothalamic & dorsal column; Trigeminal nerve; Contralateral weakness

PKA inhibits what? What does anikinra block? Radial nerve is injured where? Symptom? What if it's injured in the axilla (crutches)? Antibodies against measles target? Cavernous sinus syndrome symptoms (4)? Input to ventral posterolateral nucleus (2)? VPMN? Injury to pons causes what?

L4

Pain radiating to the medial calf and ankle indicates injury at which vertebral level?

Chlamydia; Keratoderma blennorhagicum; Sacroilitis; B27; Syphilis has a pustular rash not a vesicular

Painful urination followed by conjunctivitis, right knee pain & vesicular rash on palms & soles with negative gonococcal test is what? Name of the rash? What other symptom may appear? What HLA is it associated with? Why is this not syphilis?

Pancreatic cancer; CA-19-9

Painless jaundice indicates what? What non-specific serum markers are elevated?

ACTH deficiency from empty sella syndrome, not anemia from pituitary apoplexy

Pallor in an obese woman with hypopituitary signs is most likely due to what?

Ectopic pregnancy; Appendicitis you cannot feel a mass & ovarian torsion does not cause hemorrhage

Pallor, abdominal pain & lower right mass indicate what? How would appendicitis or ovarian torsion differ?

Digoxin; Arrhythmia; Increased calcium leads to delayed after-depolarizations; Liver; ACEI/ARBs, metoprolol, hydralazine/nitrites & spironolactone

Patient on multiple drugs for HF has nausea/vomiting & color vision changes, what drug are they on? What is the most serious complication? How does this come about? What organ is most resistant to ischemia from arterial thrombi? What 4 drugs reduce mortality in CHF?

Halothane induced hepatotoxicity; Elevated ALT & prolonged PT, hypoalbumin is a chronic marker, not acute; X-linked dominant; Hep B virus; Premature activation of trypsin!

Patient underwent cholecystectomy then has atrophied liver on autopsy, what? Lab findings of drug-induced or viral fulminant hepatitis (2)? Inheritance of Rett syndrome? Only DNA virus to replicate via reverse transcription? Cause of acute pancreatitis always?

Sudden decrease in LV preload; Decrease circulating LDL so increase LDL receptor density; Class 1C b/c only phase 0 is changed as phase 2 corresponds to increased QT; Class 1A & 3

Patient with heart failure & sudden afib has what change in hemodynamic function? Statins decrease & increase what? A drug that causes an increase in QRS w/ minimal effect on QT interval is what class? What two drug classes markedly prolong QT interval?

Coarctation of the aorta

Patients with Turners syndrome have what congenital cardiovascular defect?

Choledochal cyst; cysts that protrude into the duodenum

Patients younger than 10 years of age presenting w/ apparent biliary colic & dilation of the biliary bile duct may have what? What else may be found?

Haemophilus ducreyi; Klebsiella granulomatis

Penile ulcer with ragged borders and grey exudate indicates what? Extensive ulcerative lesions without lymphadenopathy & with intracytoplasmic inclusions indicates what?

Crigler-Najjar syndrome; UDP-glucuronosyl transferase; Bilirubin glucuronidation

Persistant jaundice, seizures and increased unconjugated bilirubin indicate what? What enzyme is absent? What is this enzyme involved in?

Osteomalacia; Osteopetrosis; Thermoregulation; Potter's syndrome; Vesicoureteral reflux; Nondepolarizing NMJ block (vecuronium); Depolarizing (succinylcholine); Paralysis for hours if they have a cholinesterase mutation

Persistence of osteoid matrix is seen in what? Persistence of primary spongiosa in the canal? Function of gloms bodies? Flattened nose & clubbed feet is what? Girl w/ recurrent UTI's, what is a risk factor for pyelonephritis? Phase 1 blockade that fades is what (example)? Phase 1 blockade that decreases with phase 2 fading is what (example)? Potential complication?

TNF-alpha, catecholamines, glucagon & glucocorticoids

Phosphorylation of insulin receptor and IRS lead to insulin resistance. What 4 mediators can induce this?

Legionella; Smokers (COPD); Urine antigen

Pneumonia with GI symptoms is most likely what? What else predisposes to this? Diagnostic tool?

Cushings syndrome, increased cortisol can cause hyperglycemic symptoms

Polyuria, polydipsia, hypertension and weight gain indicate what?

Syphilis; decrescendo diastolic murmur

Positive treponemal antibody absorption test indicates what? Aortic aneurysm caused by this has what characteristic sound?

EBV over Rubella because of the splenomegaly and absence of rash

Post auricular lymphadenopathy with a palpable spleen tip indicates what infection?

Gastroduodenal; Bile acids are not reabsorbed in the ileum->fat soluble vitamin deficiency (Vitamin K); Ulcerative colitis; Ammonia increases glutamine in astrocytes which makes them swell & prevents them form releasing glutamine to neurons to make glutamate

Posterior duodenal ulcer will bleed from which artery? Easy bleeding in Crohn's is due to what? Autoimmune hepatitis is associated with what GI disorder? How does ammonia cause asterixis?

Cryoglobulinemia

Postive HepB serology with fever, purpuric rash and diffuse joint pain indicates what?

Antibody-based assay for bacterially produced proteins because stool antigen is slow and labor intensive; TEN involves mucous membranes while scalded skin syndrome does not

Practically speaking, how do you test for C. diff? What differentiates staph scalded skin syndrome from toxic epidermal necrosis?

HIV encephalopathy; CD4 less than 200 and multinucleated giant cells

Progressive dementia with impaired memory, motor functioning and fecal/urinary incontinence indicates what? Whats labs and histology confirm this?

Onchocerca volvulus (river blindness); Loa loa=swelling of the skin and conjunctival infiltration, not skin hyperpigmentation; Onchocerca=ivermectin, Loa= diethylcarbamazine

Progressive vision loss with multiple well-circumcised black nodules in her chest and forearms indicates what infection? How does Loa Loa differ? Treatment for both?

Hypoparathyroidism due to hypomagnesemia from diarrhea

Prolonged diarrhea can cause what parathyroid changes?

Edward's syndrome; Choroid plexus cysts; Central apnea

Prominent occiput, a small jaw, low set ears, & clenched hands indicates what? What intracranial anomaly is associated with this? Most likely cause of death?

Cavernous nerves (parasympathetic from prostatic plexus) causing erectile dysfunction; Parasympathetics from pelvic splanchnic & inferior hypogastric plexus; Pudendal nerve; Genitofemoral from L1-L2

Prostatectomy can injury what nerve leading to what symptom? What Innervates detrusor muscle (2)? What nerve injury leads to fecal/urinary incontinence? What nerve injury causes absent cremasteric reflex & what spinal level injury?

L-selectin on neuts, E/P selectin on endothelial cells; PECAM-1; CD 18 integrins to ICAM-1 on endothelial cells; LFA-1 & Mac-1; Free radicals cause lipid peroxidation; vWF disease; Bernard-Soulier syndrome; Prolonged PTT w/o bleeding diathesis

Protein involved in rolling (2)? Transmigration? Adhesion (2)? Name of two CD 18 integrins? How does carbon tetrachloride cause fatty liver change? Poor platelet aggregation with ristocetin that corrects w/ normal plasma indicates what? Uncorrected? How does factor 12 deficiency manifest?

Primary Biliary Cirrhosis

Pruritis, yellow-tinged sclera & increased direct bilirubin indicated what?

Proximal tubules; Collecting ducts; Renal pelvis; Tumor cell induced osteolysis; Normal; Decreased due to renal failure; Increased b/c 1-alpha-hydroxylase is increased; Municipal drinking water systems

RCC arises from where in the kidney? Oncocytomas? Transitional cell? Hypercalcemia in multiple myeloma is caused by what? PTrP levels? 1,25 vit D levels? 1,25 Vit D levels in sarcoidosis? Mycobacterium kanasaii looks like TB but is acquired how?

Fusion of epithelial cells via F protein

RSV infection causes what typical finding using what protein?

Echinococcus granulosis; aspiration w/ US & albendazole

RUQ pain, hepatomegaly & a single, large calcified cyst in the liver indicates what infection? Treatment (2)?

IgG and fibrinogen

Rapid identification of Staph auereus causes clumping of latex beads coated with what?

Hartnup disease; neutral amino acids like tryptophan

Rash on sun exposure with vision changes and unsteady gait indicate what disease? What can be found in the urine?

Cholesterol 7alpha-hydroxylase; Cholesterol stones; Fibrates; Beta-glucuronidase; Upper glandular layer; Oxnytic (pale pink); Chief cells; Hyperosmolar food getting dumped into the small bowel (e.g. after gastrectomy)

Rate limiting step of bile acid synthesis? What would decreased activity cause? What med inhibits this? Enzyme in pigmented gallstones? Where are parietal cells located? Color? What is in the deep glandular layer? What is dumping syndrome?

Hep A; Necrotic pancreatitis; Dorsal, ventral does main duct; Decrease because liver isn't converting ammonia to urea; Increased nitrogen delivery to gut and blood leading to hepatic encephalopathy; Increased inhibitory, decreased excitatory

Raw oysters is what viral infection? Areas of white, chalky, fat necrosis of the mesentery are from what? Accessory pancreatic duct is from which portion? Cirrhosis causes what change in BUN? GI bleed causes what nitrogen manifestation? Neurotransmitter changes (2)?

Immune complex; Sterile int aspiration; Montelukast & ipratropium; Epinephrine stimulates B2 receptors causing an intracellular shift of potassium

Reactive arthritis is caused by what pathology? What paradoxical lab finding exists? What two receptor antagonists have benefit in asthma? How can a pheo cause hypokalemia?

Decreased drug delivery to the liver; Below=to systemic & above=to portal system; Methyltestosterone (he's taking steroids); EDGF inhibitors & lithium;

Rectal & sublingual have increased bioavailability over oral because why? The dentate line separates what lines of drainage? A professional athlete with acne has too much what? What two other drugs can cause this?

Continuous daily valacyclovir

Recurrence of genital herpes is mitigated by the use of what drug?

Tubular atrophy; Protein=4, alcohol=7, fat=9; No splenomegaly in aplastic anemia; First they're double negative, then double positive; P antigen

Recurrent UTI's and pathology in pic show what on histology? How many calories in 1 gram of protein/carb, fat & alcohol? MDS from aplastic anemia? Two steps before a Tcell is CD4 or CD8? What antigen does parvovirus bind?

Hereditary hemorrhagic telangiectasia (Osler-Weber-Rendu), not Marfan's or Ehler Danlos; AVM not berry aneurysm

Recurrent epistaxis, melena, seizures and cutaneous vascular lesions should make you think of what? What is the associated brain lesion?

Saxitoxin (works like tetrodotoxin); Ciguatoxin & batrachotoxin keep Na+ channel open; Depol=Na+, repol=K+; Amiodarone causes interstitial lung disease; Neovascularization; CN VII closes, CNIII opens; Hep B/C, hairy cell, HPV & Kaposi's

Red tide toxin? Eel & frog toxin name and action? Muscle action potential depol & repol? Guy with afib and now has inspiratory crackles is what med? Diabetic retinopathy fundoscope? What nerve closes & opens the eyelid? Indications for interferon-alpha (5)?

G6PD because the black pigment is unconjugated bilirubin from hemolyzed RBC's

Removal of small, black gallstones indicates what deficiency?

Osteodystrophy; Uremic toxins decrease peripheral conversion of T4 to T3; Radial traction helps maintain patency of airways; Bronchogenic carcinoma> mesothelioma

Renal disease causes what bone pathology? What causes functional hypothyroidism in renal disease? Why is TLC & RV increased in COPD? Most common cancer is asbestos?

Microvesicular steatosis; Sabin increases IgA b/c it's a live oral; Smooth muscle contraction of sphincter of Oddi; Glycoproteins; Inhibits pyruvate dehydrogenase & binds sulfhydral groups to inhibit cellular respiration; Insecticides; SS+RNA

Reye's syndrome shows what in the liver? Immune response differences in Sabin vs Salk? RUQ pain after morphine is what? What does PAS stain? Two mechanisms of arsenic? Common source? What material does HIV have?

CD54; Endothelial cell integrins; Reduction in amplitude of motor end plate potential; Progressive reduction of the Disrupts vesicle fusion; Valine, isoleucine, methionine, threonine; Musculocutaneous

Rhinovirus ligand? CMV? Myasthenia gravis causes what nerve manifestation? What is seen in a lung pressure diagram? Levetiracetam mechanism? Odd chain fatty acids an what 4 amino acids go through propionyl-CoA? Inability to flex elbow is what nerve?

Hypercalcemia from PTHrP; Cushing syndrome & SIADH; Galactorrhea

SCC of the lung causes what abnormality? Small cell lung cancer? Large cell lung cancer?

>2 symptoms, 1 being delusions, hallucinations & disorganized speech & > 6 months; >1 month but <6 months; Delusions/hallucinations w/o mood symptoms for > 2 weeks; Some attacks occur w/o trigger; Haloperidol; Inhibit the serotonin transporter; Mood reactivity, increased sleep & eating, leaden paralysis; Mirtaz & bupropion; Clozapine & olanzapine;

Schizophrenia definition? Schizophreniform? Schizoaffective? Panic disorder? What do you treat delirium with? How do SSRIs work? Atypical depression characteristics (3)? 2 drugs w/o sexual side effects? Two drugs w/ weight gain?

Increased lipoprotein synthesis; Parenchymal pressure atrophy; Glomerular permeability factor; Diaphragmatic & mediastinal

Secondary change in nephrotic syndrome? Kidneys in severe BPH show what? What cytokine is overproduced in MCD? Phrenic nerve transmits pain from which two pleura?

Acute mesenteric ischemia

Severe abdominal pain that is out of proportion to physical examination findings is indicative of what?

Peptic ulcer disease; gastroduodenal artery

Severe epigastric pain, episodes of vomiting blood and relief with antacids indicates what? What vessel is involved?

Cluster headache because they often present with autonomic symptoms

Severe pain behind the eye, tearing, nasal congestion, red eye, and reduced pupil size indicated what?

Tabes dorsalis

Shooting leg pains, difficulty ambulating, urinary incontinence and pupils unreactive to light but reactive to accommodation indicate what?

Hyperthyroidsim

Short-temperment, diarrhea, weight loss, weakness in the proximal muscles, atrial fibrillation & tachycardia indicate what?

Yes, colposcopy & cervical biopsy of any lesions suspicious of cervical intraepithelial neoplasia (CIN) 2

Should a colposcopy & cervical biopsy be performed on a pregnant patient?

Monoclonal expansion as hyperplasia is polyclonal; In the bone marrow, not a peripheral blood smear;

Sign of malignancy in enlarged lymph node? Ringed sideroblasts can be seen in lead poisoning where?

cGMP; BNP, ANP & EDRF (NO); Head bobbing & widened pulse pressure; Cell death in the setting of normal calcium levels; Increased serum calcium levels

Sildenafil decreases degradation of what? What three other hormones use this signaling pathway? Aortic regurg presents with what head manifestation and pulse pressure? What causes dystrophic calcification? Metastatic calcification?

FKBP which inhibits mTOR; Rituximab; Chloride as a production of CO2 entering RBC's; 20%; Inferior gluteal; Tracheal stenosis; CD8+ T cells

Sirolus inhibits IL-2 via what? Antibody against CD20? What ion is higher in venous blood than arterial? Normal filtration fraction? Gluteus maximus innervation? Lung flow-volume curve with a circular like curve is what? COPD has increased neutrophils, macrophages & what?

Vitamin K & folate; Short gastric veins or splenic vein thrombus; Premature babies w/ curvilinear areas of lucency on X-ray; Duodenal atresia; Splenic flexure & rectosigmoid junction

Small intestinal bacterial overgrowth increases what two vitamins? Esophageal varies are due to the left gastric vein, what causes fundal varices (2)? Necrotizing enterocolitis presents and who and what on imaging? Double bubble sign is what? Nonexclusive ischemia occurs where (2)?

Bloom syndrome

Small stature, infertility and sun-sensitive facial rash indicate what syndrome?

Otitis media, asthma & SIDS; Chronic; Commonly asymptomatic; Spongiosus; Stratum spinosum; Dyskeratosis; Stratum granulosum; Stratum corneum

Smoking during pregnancy causes what 3 things? Bronchiolitis obliterans is what type of rejection? Symptoms of acute rejection? Term associated with atopic dermatitis? Acanthosis? Squamous cell? Lichen planus? Hyperkeratosis?

Increased pH, PCO2 and bicarb

Someone who has been vomiting a lot will have what changes in pH, pCO2 and bicarb?

Alcohol withdrawal

Someone who undergoes an operation and has a seizure the subsequent morning with no history of seizures may be experiencing what?

Firm, small & brown; Itchy, rough rash; X-linked dominant; Rasburicase (urate oxidase); Sevelamer; 5HT3 antagonists, dopamine antagonists & neurokinin 1 antagonists

Spleen histology in sickle cell? Peau de'orange description? inheritance of vitD resistant rickets? Besides allopurinol what can be used for tumor lysis syndrome (enzyme)? Drug to manage hyperphosphatemia in kidney disease? 3 drugs classes for chemo vomiting?

Exfoliatin; IL-4, IL-5 & IL-10 Th2 mediatied; Hypoaldosteronism or aldosterone resistance; Primary biliary sclerosis; Rheumatoid arthritis

Staph scalded skin syndrome is caused by what exotoxin? Lepramatous leprosy shows what cytokine profile (3)? What two things cause type 4 renal tubular acidosis? Anti-Mitochondrial antibodies are in what? Anti-cyclic citrullinated?

CDK 4 hyperphosphorylates Rb to inactivate it, releasing E2F & allowing progression; Inhibits CDK; Cyanide by inducing methemoglobin (Fe3+); Cytochrome C in the mitochondria; HGPRT, xanthine oxidase & TPMT

Steps of Rb in G1->S? What does p27 do? Amyl nitrate administration increases hemoglobins affinity for what? What does cyanide inhibit? 6-mercaptopurine is activated by what & inhibited by what two things?

TCA & electron transport chain; Hep D needs the viral coat of Hep B to infect hepatocytes; D-xylose b/c it's a monosaccharide whose absorption is only decreased with intestinal overgrowth; Superior mesenteric though the marginal artery of Drummond; Nope

Succinate dehydrogenase is used in what two places? How does Hep B help Hep D replicate? After pancreatectomy, what substance will have normal levels? Perfusion of the bowel is maintained via which vessels when the inferior mesenteric artery is ligated (2)? Does Hep A have a carrier state?

Vertebral subluxation; Acute=flaccid, chronic=spastic; Endocarditis; Decrease gut flora so increase anticoagulation; NF1=neurofibromin, NF2=merlin; Microglial inflammatory nodules

Sudden quadriparesis after intubation in rheumatoid patient is what? Acute versus chronic spinal cord injury? Cerebral septic emboli are from what? Relation of antibiotics and warfarin? NF1 vs NF2 protein? Histopath of aids dementia?

Bicuspid aortic valve; Aortic valve disease; Parvus et tardus pulse & syncopal episodes

Systolic heart murmur at the second intercostal space indicates what? What this degenerate into? Signs (2)?

Ochronosis; Crohn's/UC, mesalamine; Inferior mesenteric & internal iliac; Inguinal; Hep B=ground glass, C=lymphoid aggregates w/ macrovesicular steatosis, E=patchy necrosis, A=Councilman bodies; Bilateral reccurent laryngeal nerve injury; Palatoglossus

Term for black seen in alkaptonuria? Sulfasalazine indications & similar drug? Lymph above dentate line goes where (2)? Below? Hep B, C, A & E liver biopsy? When does vocal cord paralysis occur? Only tongue muscle with CNX?

Mullerian inhibiting factor; External male genitalia; Internal male genitalia; Gram-variable anaerobic

The SRY gene on the Y chromosome causes the Sertoli cells to produce what? What does DHT do in embryogenesis? Testosterone? What is the gram stain & O2 requirements of Gardnerella?

Telencephalon gives rise to the striatum

The circled area in the image below derives from which embryologic structure?

Ends of the sarcomere where actin binds for stabilization; Myosin chain with heads; Actin does not overlap with myosin; Myosin does not overlap with actin; Psuedomonas; Different number of mitochondrial genomes in a cell lead to variable expressivity

The darker vertical line is what? A band? I band? H zone? Some in a swimming pool with ear pain is caused by what? What is heteroplasmy?

DNA repair

The inherited breast cancer mutations causes alterations in which cellular process?

Left renal or left phrenic vein

The left adrenal vein can drain into which two veins?

Frequency of firing of individual motor neurons

The nervous system increases the force of muscle contraction in a graded fashion by recruitment of active motor units and by increasing what?

It (C-peptide) is stored in secretory granules and exocytosed with insulin, not degraded in cell proteasomes; IL-1

The portion cleaved form the proinsulin molecules has what fate? Which protein is directly secreted w/o RER involvement?

Serotonin syndrome

The triad of altered mental status, autonomic instability & muscular rigidity indicates what?

Androgens; Estrodiol; Meningitis & pneumonia as the other diseases are caused by nontypeable H. influenza; Mefloquine or quinine; In vivax/ovale to kill the hypnozoites in the liver

Theca cells produce what? Granulosa cells produce what? What does the H. influenza vaccine mostly protect from (2)? What do you give someone w/ a chloriquine resistant malaria (2)? When is primaquine administered in conjunction with chloriquine?

Parvovirus B19, thymoma & lymphocytic leukemia; Follicular lymphoma has an indolent course; Complement mediated; M4/5=monocyte, M6=erythroid & M7=megakaryoblasts; M7; Xa inhibitor

Three things that cause pure red blood cell aplasia? Lymph node with fluctuating size over a year is what? PNH is what type of hemolysis? In AML state the precursors for M4/5, M6 & M7? Which one is associated with Down's? What is fondaparinux?

Decreased PTH leads to decreased stimulation of osteoblasts to produce RANKL and M-CSF (a cytokine that stimulates the differentiation of hematopoietic stem cells into osteoclasts that express the RANK receptor)

Thyroidectomy causes what changes in osteoblasts and osteoclasts?

I=single strand breaks, II=double strand; II; Irinotecan & topotecan; Compensatory erythrocytosis; Chesapeake & Kempsey Hgb; CD14; These are reticulocytes apparently with increased ribosomal RNA; Ferrochelatase

Topoisomerase I vs II? Which one does etoposide work through? What two inhibit topoisomerase I? If p50 is reduced, what would results? Two hemoglobins that would cause this? Marker for macrophages? Person treated for iron deficiency anemia with large, bluer blood cells, what causes the blue? Cutaneous photosensitivity in early childhood could be what heme deficiency?

Transmembrane ATP-dependent efflux pump; Renal tubular epithelial cells to eliminate foreign compounds; Bind GpIIb/IIIa; Inhibit IIa; Central regulator of iron homeostasis via insertion of ferroportins in liver & enterocytes;

Tumor cells resistant to chemo drugs through MDR1 is what protein? Where is this protein naturally located? Abciximab & tirofiban do what? Dabigatran/argatroban? What is hepcidin?

Superior mesenteric artery; Gastroduodenal; Prechordal mesoderm; Supravalvular stenosis; MHCII because they need acidic lysosome to cleave invariant chain & mix w/ endocytosed antigens; Endogenous antigens bind in RER;

Tumor in 3rd part of duodenum compresses what? 1st part? Phenotypic features of patau syndrome are due to defective fusion of what? William's syndrome heart defect? Defect in lysosomes would most effect what receptor in APCs? How does MHC I differ?

Amine dyes & smoking; Secondary hypertension; No because they arise from the Wolffian duct; Poles=compound papillae, middle=simple; Decreased lung compliance from distortion of tissue

Two risk factors for transitional cell carcinoma? Uncorrected VUR can cause what? Can females have posterior urethral valves? Difference cells at the poles of the kidney versus in the middle? How does left heart failure cause dyspnea?

Uric acid & cystine; Metabolic acidosis; Mycoplasma & chlamydia do not have cell walls; Bismuth; Anterior abdominal aponeurosis as the bladder is extraperitoneal

Two stones associated w/ low pH? Chronic diarrhea causes what pH derangement? A drug that inhibits glycosyltransferase would not effect which two bugs? 4th drug in quad therapy for H. pylori? A suprapubic tube can damage what?

Achlorhydria & hypokalemia; Rhabditiform larvae in stool, ivermectin; Toxic megacolon; Abd x-ray; By the ileocecal valve; Tenia coli=3 bands along colon that converge at appendix; Anterior tibial vessels & deep peroneal nerve;

Two symptoms unique to VIPoma? Diagnosis & treatment of strongyloides? Untreated UC w/ maroon colored stool is what? Next step? Where does intussusception usually occur? Landmark for appendix? Anterior compartment syndrome effects what structures (2)?

Chronic obstructive bronchitis because any chronic pathology would have allowed time for compensation; Varenicline partial nicotine agonist; Stimulate NK cells to destroy; Lysosomal enzyme release from macs or neuts

Uncompensated respiratory acidosis cannot be caused by what? What drug & MOA reduces nicotine withdrawal and attenuates rewarding effects of nicotine? What does IL-12 do? What causes abscess formation?

Unfractionated=antithrombin, Xa & thrombin, LMWH=antithrombin & Xa; PT reagent has heparin neutralizers; Tyrosine kinase; Pre=tamoxifen, post=aromatase inhibitor

Unfractionated heparin versus LMWH? Why is PT not increased w/ heparin? What type of receptor does Trastuzumab bind? Pre versus post menopausal estrogen breast cancer treatment?

V2=collecting ducts Gs, V1=vessels Gq; Decreases renal clearance by increasing absorption; Desmopressin does not act on V1; Out of the cell to lose + charge and bring it closer to -90mV

V2 versus V1 location & receptor? What does ADH do to urea? Difference between desmopressin & vasopressin? If membrane potential is -70mV & equilibrium for K is -90mV, what way will it move?

Peroxisomes; Zellweger & x-linked adrenoleukodystrophy; Nervous, adrenal glands, testes; Just contralateral paralysis; Medial=ASA contralateral paralysis w/ tongue, lateral=PICA ipsilateral face & horners w/ contralateral body sensation; Same as PICA but w/o horners & with decreased lacrimation/salivation; Locked in syndrome; D2 antagonism in nigrostriatal pathway; Mydriasis, ptosis & down & out

VLCFA can't undergo beta-oxidation, so what degrades them? 2 diseases associated w/ this? 3 organ systems effect by the later? Lenticulostriate symptoms? Medial vs lateral medullary symptoms & vessels? AICA? Basilar artery? What causes dystonic reaction after starting an antipsychotic? Pcom symptoms (3)?

Basilar & subclavian on left, brachiocephalic on right; Brachiocephalic; CABG of internal mammary artery

Vertebral arteries run between which two other arteries on the left and right? The inominate artery is another name for what? Coronary-steal syndrome is from what procedure?

Interact with MHC molecules on macrophages & nonspecific activation of T-cells; IL-1 from T cells & IL-2 & TNF-alpha form macrophages

Via what cells do superantigens exert their effects (2)? What does this release (3)?

Lichen sclerosus; Topical corticosteroids

Vulvar dystrophy in post-menopausal women is called what? Treatment?

Aversion to smoking; Involve a social worker during discharge; Hepatic excretion; Yersinia; Rapidly increasing tolerance to norepi in alpha agonists & NO in nitroglycerine; Inhibits nerve sodium channels

Weird sign of Hep A infection? Patient w/ impaired cognitive status, not taking pills & daughter is at work, what should you do? Excess copper is usually secreted how? ETEC & what increase cGMP? What is tachyphylaxis & what two drugs is it seen in? How does tetrodotoxin (puffer fish) work?

Ferrochelatase & ALA dehydratase; Antitoxin first, then penicillin; Nausea, headache, seizures, tachyarrhythmias; Benzodiazapines/barbiturates & beta-blockers; Coronary sinus

What 2 enzymes does lead inhibit? How do you treat diphtheria? Theophylline intoxication causes what (4)? What do you treat the seizures & arrhythmias with? What vessel has the lowest oxygen content?

Constipation & miosis; Target cells; Superficial inguinals; Glans penis drains to deep inguinal & calf to popliteal; Superior to external iliac & inferior to internal iliac; Inferior mesenteric

What 2 opioid effects do not display tolerance? Cells seen in beta thal? Anal canal below the dentate line drains to where? What two cutaneous areas do not drain there? Bladder drainage? Upper 3rd of the rectum?

Vasculature, bladder, eye; Vasculature, bronchi, uterus; Dobutamine is beta 1 selective while isoproterenol is a nonselective beta agonist; Binding activates phospholipase C which cleaves a molecule into DAG (directly activates PKC) & IP3 (activates PKC by increasing calcium release)

What 3 organs have alpha 1 receptors? What 3 have beta 2? Difference between dobutamine & isoproterenol? Explain Gq mechanism.

HCM, phosphodiesterase inhibitors & RV infarcts; WPW; Angiotensin 2 because ACE lives in the lungs and cleaves Ang 1 to Ang 2

What 3 populations are nitrates contraindicated in? Slight delta wave with STEMI is what? What is higher in pulmonary veins compared to pulmonary arteries & why?

Histamine, heparin & leukotrienes; Muscarinic agonist; Copper is lysl oxidase, Vitamin C is hydroxylase; Can precipitate pulmonary edema; Loop diuretic

What 3 things do basophil granules contain? What class is methacholine? Copper versus vitamin C enzymes? Why is mannitol contraindicated in CHF? What is bumetanide?

Glucose, naloxone & thiamine

What 3 things do you give a comatose patient who comes to the ER?

Activates macrophages, increase MHC expression & promotes TH1 formation; T cells; Purpura, intestinal hemorrhage & arthralgia; Staph can adhere to normal cells whereas Strep need a previous lesion

What 3 things does interferon-gamma do? What produces it? Three symptoms of HSP? Difference in Staph & Strep mechanisms of vegetation?

Vancomycin, opioids & radiocontrast; PKA; Multiple myeloma & Hodgkin's lymphoma; Hyaline cartilage; Frameshift; Multiple of 3 deletion

What 3 things induce IgE-independent mast cell degranulation? What messenger do they work through? What two heme/lymph cancers cause hypercalcemia? Biopsy of a hamartoma? What class of mutation causes Duchenne's? Becker's?

Fasting serum glucose > 126 mg/dl measured on two separate occasions, HbA1C > 6.5%, random blood glucose > 200 mg/dl on two occasions or random blood glucose > 200 with symptoms of hyperglycemia

What 4 criteria can diagnose diabetes?

Antihistamines, TCA's, alpha 1 blockers, centrally acting alpha 2 agonists & sedatives; Two clinicians independently confirm the site; Mitral valve hitting the interventricular septum; Wegner's & polyarteritis nodosa; Interview of the child on their own

What 5 drugs do you avoid in the elderly? How do you reduce wrong site surgery? Left ventricular outflow obstruction in HCM is due to what? Mononeuritis multiplex is associated with what (2)? Suspicion of child abuse should be followed by what?

Theophylline; Sildenafil, tadalafil; Roflumilast for COPD; Milrinone, cilostazol & dipyramidol; PDE5; urinary deoxypridinoline; Inferior parathyroid & thymus are both from pouch 3

What PDE inhibitor is nonselective? PDE5 (2)? PDE4 & use? PDE3 (3)? All increase cAMP, except which one increase cGMP? Most reliable marker of osteoclast activity? A person with myasthenia graves has a dysfunctional organ from the same place as what other organ?

Protein kinase A inhibits & protein phosphatase activates

What activates & what inhibits glycogen synthase?

Arginine; Maxillary artery, common carotid & internal carotid, subclavian arteries, pulmonary arteries & ductus arteriosus; Posterolateral is only supplied by PDA while anterolateral has LAD & LCX supply

What amino acid synthesize NO? What do the 1st, 3rd (2), 4th & 6th (2) aortic arches give rise to? Why is posterolateral papillary rupture more common than anterolateral?

Anti-double stranded DNA; Reduced C3/C4; Cystic renal dysplasia involves the renal pelvis; Eosinophils; Primase b/c uracil is only in RNA; DNA poly III

What antibody is in lupus? What else can be seen in labs? What differentiates ARPKD cystic renal dysplasia? Interstitial nephritis has what in the urine? Presence of uracil in DNA replication is what enzyme? Synthesis of daughter strands is done by what?

4th pharyngeal arch; Histamine; Increases preload; Ask the daughter to leave the room>contact adult protection services

What arch does a portion of the aortic arch come from? Red man syndrome is do to what molecule? What does leg raise do? Next steps in an elder who appears neglected and whose daughter is answering all of the questions?

Sarcoidosis, amyloidosis & hemochromatosis; Transthyretin; Retinol & thyroxine; Increase in size before regressing

What are 3 common causes of diastolic heart failure? Deposition of what amyloid protein causes infiltrative cardiomyopathy? What is it a carrier of? Most likely course of a strawberry hemangioma?

Profuse watery diarrhea, non-anion gap acidosis & hypokalemia

What are 3 main symptoms of a VIPoma?

DNA binding proteins; Calcium binding proteins; Neural crest & Langerhans; nitrates=large veins, nifedipine=arterioles; Epi & norepi

What are c-Fos & c-Jun? What are S-100 proteins? What two cells are they found in? What is the difference in size of vessels that nitrates and Nifedipine work on? What two molecules do precapillary sphincters respond to?

Dopamine, norepinephrine & epinephrine; dopamine & norepi=periphery & epi=adrenal medulla

What are catecholamines (3)? Where are these mainly produced?

Cadherins; Integrins; Acid-fast, 41 Celsius; Anemia & hepatosplenomegaly; Azithromycin; Microcalcifications with central areas of necrosis; Single file cells

What are desmosomes composed of? Hemidesmosomes? MAC has what stain and grows at what conditions? What differentiates this from disseminated TB (2)? What medication is given prophylactically for this? What does histopath of ductal carcinoma in situ show (2)? Histopath of lobular carcinoma?

Presents like MS with bilateral optic neuritis, spinal cord lesions spanning 3 contiguous segments & normal brain MRI; Devic disease; antibodies against aquaporin 4 protein

What are key features of neuromyelitis optica? What is another name for this disease? What does CSF show?

p-ANCA=myeloperoxidase, c-ANCA=proteinase 3; It falls duh!!!; Celecoxib; Positive=thymic cortical epi cells, negative=thymic medullary & dendritic cells; Pancytopenia! Hemolytic anemia

What are p-ANCA & c-ANCA against? What happens to PaO2 at altitude? What NSAID does not impair platelet aggregation? What cells do T cells interact with in positive & negative selection? Hematology manifestation of Gaucher's? Low haptoglobin is what?

Leukopenia, ESR close to 0 & proteinuria; Oral hairy=not precancerous, EBV; Oral=precancerous, tobacco/alcohol

What are the 3 lab findings of yellow fever? Difference between oral hairy leukoplakia & oral leukoplakia?

ASD, VSD or conotruncal abnormalities; Fixed split S2 with systolic ejection murmer

What are the 3 most common cardiac abnormalities in FASD? What does an ASD sound like?

Sedation, antimuscarinic, anti alpha adrenergic; Laxative; Adenoma not a carcinoma as hot nodules are benign & cold are malignant; Toxoplasma=abscess, cryptococcus=meningitis

What are the 3 side effects of antihistamines? What is decusate sodium? Hot nodule seen incidentally is probably what? Toxoplasma versus Cryptococcus in HIV patient (picture)?

Skin, eye, mouth=most common, disseminated=highest mortality & CNS=highest morbidity; Spiramycin

What are the 3 types of neonatal herpes infections & their population associations? Prophylactic treatment of toxoplasma?

Pneumocystis jirovecii prophylaxis & treatment for African sleeping sickness & leishmaniasis

What are the 3 uses for Pentamide?

Afferent=CN5, efferent=CN7; Afferent=CN9, efferent=CN10

What are the afferent and efferent components of the corneal blink reflex? Gag reflex?

1=CII, 2=B-100 & 3=E; 1 & 3; activates LCAT

What are the associations of familial dyslipidemias with apolipoproteins? Which are AR? What does A-I do?

CD8+ T lymphocytes known as Downey cells; CMV b/c syphilis does not have a rash

What are the atypical cells of EBV? Child with petechial rash, yellow skin, palpable spleen & liver & hearing loss has what infection?

Bitter almonds & garlic; hydroxocobalamin

What are the breath smells in cyanide and arsenic poisoning? Treatment for cyanide poisoning?

Epidemic=Rickettsia prowazekii, flying squirrels; Endemic=Rickettsia typhi, rat, mouse or cat; Superoxide dismutase, catalase & glutathione peroxidase

What are the causes & vectors for epidemic (1) versus endemic typhus (3)? List the 3 inherent antioxidants.

LAME= listeria, atypicals (mycoplasma, chlamydia), MRSA & enterococci; ampicillin

What are the cephalosporin resistant organisms? Treatment for Listeria?

Avoidance of aspartame and protein rich foods like meat, dairy products, grains and starchy vegetables

What are the dietary recommendations for PKU?

Maculopapular rash, snuffles, chorioretinitis, thrombocytopenia, & anemia; Gonorrhoeae=purulent discharge, Chlamydia=mucopurulent, Adenovirus=mucoserous, watery

What are the early sings of congenital syphilis (5)? Difference between Chlamydia, Gonorrhoeae & adenovirus neonatal conjunctivitis?

Constrict blood vessels, inhibit sympathetic, excitation of the heart, vasodilation; Beta 2; Gs=beta, Gq=alpha 1, Gi=alpha 2; Cardiac cells & juxtaglomerular apparatus

What are the effect of alpha 1, alpha 2, beta 1 & beta 2 receptors on the heart? Which one of these is not stimulated by norepinephrine? What has Gs, Gi & Gq? Where are B1 receptors found (2)?

Merocrine=exocytosis (sweat, saliva), apocrine=membrane-bound vesicles (mammary glands), holocrine=cell lysis (sebaceous glands); Abnormal lyonization (X-inactivation); Direct renin inhibitor; TCA

What are the functions of merocrine, apocrine & holocrine glands (examples)? How can a female express a x-linked recessive disease? What is aliskerin? Man with hallucinations, prolonged QT, dry mouth & facial flushing overdosed on what?

Disorganized glandular structures with perineural invasion

What are the histologic features of the most common primary pancreatic tumor?

Henderson-Paterson bodies

What are the intracytoplasmic inclusion bodies observed with a molluscum contagiosum?

Orthomyxo, rota, bunya & arena

What are the segmented viruses?

Small stature, recurrent infections & telangiectatic erythematous macules following a photo-distribution pattern; 15, DNA helicase

What are the symptoms of Bloom syndrome? Chromosome & protein effected?

Limb hypoplasia, chorioretinitis, & microcephaly; Loop of Meyer; Contralateral superior homonymous quadrantanopia

What are the symptoms of VZV congenital infection? HSV infects the temporal lobe which contains what visual pathway? What is the resulting visual disturbance?

It is benign but will cause increased CSF production leading to hydrocephalus

What are the symptoms of choroid plexus papilloma?

Parkinsons symptoms usually without tremor and with paresis and paralysis of vertical gaze and square-wave jerks

What are the symptoms of progressive supra nuclear palsy?

Acne, testicular atrophy & increased hematocrit

What are the three main findings of steroid abuse?

Na/Ca 3:1 exchanger on the membrane & SERCA ATPase in the sarcoplasmic reticulum; The allele of one gene effects phenotypic expression of alleles in another gene; Poly I=RNA primer removal, Poly III=fixing mismatched bases

What are the two channels that remove calcium from the cell? What does genetic epistasis mean? What does DNA poly I & DNA poly III use their exonuclease activity for?

Constipation & AV node block; Thiazide diuretic; Inarterial septum by the coronary sinus; Opening of the pulmonary veins; Between tricuspid valve & IVC opening

What are the two main side effects of CCB's? What is indapamide? Where is the AV node located? What other area is commonly ablated in afib? Where is ablation in atrial flutter?

Glycerol & fatty acids; hormone sensitive lipase; glycerol goes to G3P via glycerol kinase & fatty acids go to ketogenesis/TCA cycle via beta-oxidation (AcylCoA dehydrogenase)

What are the two products of triglyceride metabolism? Enzyme used to make these products? What are the fates and enzymes used in both?

Androgenetic alopecia 7 BPH

What are the two uses of finasteride?

Motor=Broca's but with intact repetition, sensory=Wernicke's but with intact repetition; lesions are near but not directly on the given areas

What are transcortical motor and transcortical sensory aphasias? Where are the lesions?

Renal to proximal portion & superior vesical to distal portion; Internal iliac artery; Gonadal; SMC of the afferent b/c JGA would be pumping out renin; Ureteropelvic junction; Vesicoureteral junction

What arteries supply blood to the ureter? In the true pelvis, what do the ureters cross anteriorly to? Posteriorly in the retroperitoneum? Clip applied to a pigs right renal artery would cause hyperplasia where? Fetal hydronephrosis is caused by obstruction where? Nonobstructive?

Deep brachial; Overactive=sudden urge with leakage while trying to reach the bathroom, underactive= continuous leakage; Urge=low, overflow=high; Multiple myeloma; Pseudomonas

What artery runs with the radial nerve? Symptoms of overactive versus underachieve detrusor? PVRU in overflow versus urge? Elder man who has back pain, fatigue & renal problems w/ eosinophilia has what? Patient with catheter UTI & gram negative lactose negative organism is what?

Occipital artery; Kiesselbach's plexus; Sphenopalatine, facial & ophthalmic; Primary biliary cirrhosis; Drug induced lupus & edema

What artery supplies the sternocleidomastoid? What plexus causes nosebleeds? What 3 arteries anastamose there? What are antimitochronnrial antibodies found in? Besides tachycardia, what two other side effects are seen with hydralazine?

Latissimus dorsi; Thoracodorsal; Trapezius accessory nerve; Endonuclease, NER; Ataxia telangectasia & Fanconi anemia; Catalase

What back muscle internally rotates the arm? Innervation? What muscle & nerve is injured in whiplash? What protein and function is inhibited in xeroderma pigmentosum? What two syndromes have NHEJ defects? Destruction of H2O2 is by what enzyme?

Necrotic hepatocytes with neutrophilic infiltrate and Mallory bodies

What biopsy findings are found in alcoholic hepatitis?

Bacterial abscess of cerebellum or temporal lobes; ring-enhancing lesions

What brain pathology is common following otitis media? What does it look like on CT?

Cheyne-stokes breathing; Neurologic dysfunction & CHF, not OSA; Would be constant height, not crescendo-decrescendo; Retainment of foreign body causing a granuloma

What breathing pattern in observed in the picture? Disease association (2)? How would OSA look? Kid with laceration and subsequent nodule has what?

Mesna binds acrolein to prevent it from being degraded by the cancer drugs and damaging the bladder

What can be given with chemotherapy agents to prevent gross hematuria side effect? How?

Subacute cerebellar degeneration; Anti-Yo/Anti-Hu; Corticospinal=motor in brain, spinothalamic=sensory in spine; Corticate=flexors & above the red nucleus, cerebrate=extensors & below the red nucleus; Memory loss;

What can small cell lung cancer & ovary/breast cancer cause? Antibodies (2)? Corticospinal vs spinothalamic? Decerebrate vs decorticate posturing? Irreversible symptom of Wernicke's?

Oligohydramnios causing pulmonary hypoplasia, limb deformities & flat facies; ACEI/ARBs; Contraindicated in the 3rd trimester due to premature PDA closure

What cause Potter's sequence & what are the three manifestations? What drugs cause this (2)? Can NSAIDs be used in pregnancy?

Combination of diabetic neuropathy and vasculopathy; autonomic neuropathy

What causes Charcot arthropathy? What does a DM patient with fainting spells likely have?

Viral DNA integration into the host genome

What causes HBV to develop into hepatocellular carcinoma?

JC virus

What causes PML?

Hypocalcemia

What causes Trosaeua's & Chovsteks signs?

Failed involution of the mammary ridge; Midclavicular=6-8th rib space, midaxillary=8-10 & paravertebral=10-12; Poorly perfused areas are vasoconstrictor but O2 opens these ups leading to an increase in physiologic dead space & decreased minute ventilation

What causes accessory nipples? Where should thoracentesis be performed midclavicular, midaxillary & paravertebral? Mechanism of O2 induced hypercapnia in COPD (2)?

Citrate anticoagulant chelates calcium; Magnesium is also chelated; Eos; 5 P's Pee (diuretics), Penicillin, Pain (NSAIDs), PPI's & rifamPin; BUN/Cr>20 b/c hypoperfusion w/ low urine Na; BUN/Cr<15 b/c BUN is not absorbed w/ high urine Na

What causes hypocalcemia after blood transfusion? What other enzyme is effected? Interstitial nephritis has high what? 5 causes? Prerenal azo BUN/Cr & FeNA? Intrinsic?

Primary hyperaldosteronism

What causes secondary hypertension as well as suppression of the renin-angiotensin system?

Decreased lagoon fibril production & increased cross-linking; Nonlinear decline in GFR w/ increase creatinine, slightly overestimates GFR; Hypoventilate them so that the hypercapnia causes vasoconstriction; Pyelonephritis & AKI

What causes wrinkles (2)? How are creatinine & GFR related? What can you do to cerebral edema patients to decrease their cerebral blood flow? What two diseases are WBC casts seen in?

Paranasal cells (fried-egg); S2-S4; Sacrospinal ligament; Ischial spine; internal pudendal & inferior gluteal artery

What cell predominates in a pap smear of a postmenopausal woman (appearance)? What are the nerve roots of the pudendal nerve? What ligament does this nerve run under? What bony prominence does the ligament attach to? What structures run medial to this nerve that you must avoid (2)?

T cells; Hot tub folliculitis; Spinal stenosis; Ligamentum flavum; Lewy bodies (alpha-synuclein eosinophilic inclusions)

What cells are involved in poison ivy? Papulpustular rash from Cancun is what? Parsthesias relieved by bending forward indicate what? What ligament hypertrophies? Besides loss of substantial nigra what other brain pathology is seen in Parkinsons?

Myeloblasts contain auer rods; Peroxidase; V, D, J diversity; CLL; Myelofibrosis & hairy cell leukemia;

What cells are pictured & what do they contain? Stain? What does TdT do? What leukemia do PDGF receptors play a role in? Two diseases w/ dry tap?

Ovary=simple cuboidal, fallopian tubes, uterus & cervix=simple columnar, ectocervix/vagina=stratified squamous; Unprotected sex as viable sperm can still remain distal to the ligated portion

What cells make up the ovary, fallopian tubes, uterus, ectocervix & vagina? A patient undergoing vasectomy should be told to avoid what in the next 3 months?

Kulchitsky cells; Peristalsis & nausea; (Vd x .7)/CL; Vd x plasma concentration; plasma concentration x CL; "I won't" telling her that you won't but urging her to tell her is coercive and damages patient trust

What cells synthesize serotonin in the GI? 2 functions of serotonin? What is the half-life equation? Loading dose? Maintenance dose? if a patient asks you not to tell her husband that the baby isn't his, what is your response?

Erythrocytes and endothelial cells in the blood brain barrier use GLUT-1

What cells use a high-affinity, hormone-independent transporter to uptake glucose?

Chromosome 13; AD=1 inherited and 1 acquired mutation, sporadic=2 acquired which is rare

What chromosome is often mutated in retinoblastoma? What is the difference in the autosomal dominant and sporadic versions?

Cerebellar hemisphere, not the vermis

What component of the motor system is injured in an intention tremor?

Neural tube defects; Nasal hypoplasia, stippled epiphysis; Microcephaly, hydrocephalus & thymic hypoplasia; Neural tube, cleft palate, microcephaly, finger nail hypoplasia

What congenital birth defect is valproate associated with? Warfarin (2)? Vitamin A (3)? Phenytoin (4)?

TNF-alpha; activated macrophages; IL-1 & IL-6

What cytokine mediates septic shock? What produces it? What two other cytokines induce systemic inflammatory responses?

MAC complex (C5b-C9)

What deficiency leads to impaired killing of Neisseria species?

Functional liver measures like bilirubin, PT & hypoalbuminemia; Vascular occlusion; Apple-peel sign; Psoriasis; True diverticulum from scarring & traction; Ursodeoxycholic acid;

What determines cirrhosis prognosis (3)? Duodenal atresia is failure to recanalize, what is jejunal/ileal atresia due to? Sign on imaging? What other skin pathology is acanthosis in? Mediastinal lymphadenitis can cause what? Nonsurgical treatment of gallstones?

Metformin

What diabetes medication is contraindicated in renal failure?

1st generation sulfonylureas

What diabetes medications cause a disulfiram like reaction with alcohol?

Normal serum lipase favors cholecystitis

What differentiates a presentation of cholecystitis from acute pancreatitis?

The presence of masculinization at puberty differentiates indicates 5-alpha reductase deficiency

What differentiates complete androgen insensitivity from 5-alpha reductase deficiency?

E. coli is indole positive while Enterobacter cloacae is indole negative

What distinguishes E. coli from Enterobacter cloacae?

Loop diuretics; hyponatremia & hypokalemia, hypercalcemia

What diuretic causes hyponatremia, hypocalcemia & hypokalemia? What do thiazides cause?

LH=testosterone from Leydig cells, FSH=inhibin B from Sertoli cells

What do LH & FSH respectively stimulate & release in the testes?

Kill infected cells that have decreased MHC1 expression by inducing apoptosis; Yes; Decreases the pH & increases osmolality; Increased breath hydrogen content; Decreased peristalsis amplitude & increased LES pressure; Scleroderma

What do NK cells do? Are they present in athymic patients? Lactase deficiency does what to the stool (2)? Other test? 2 signs of achalasia on manometry? What would decreased LES pressure be?

Carrier proteins for oxytocin and vasopressin

What do neurophysins do?

-zosin drug; beta-blocker; HCTZ; Endothelial cell dysfunction; Smooth muscle cells, not fibroblasts; Verapamil or diltiazam

What do you give someone w/ HTN & BPH? CAD & HTN? Essential HTN? What is the first cell type involved in CAD? What produces the fibrous cap? Patient with bradycardia & dizziness is mostly likely on one of what two drugs?

Antihyperlipidemic agents (Gemfibrozil)

What do you use to treat pancreatitis due to hypertriglyceridemia (example)?

Lymphocytes, plasma cells, macrophages and fibroblasts; fungal or Mycobacterium tuberculosis

What does CSF of chronic meningitis show and what is the causative agent?

Movement of chloride ions; L4; S1; Infraorbital does cheek, lip & gums; Zygomatic or temporalis; L2; Putamen; At the foramen cecum; Pcom; Precentral=motor, postcentral=sensory; Eyes to the right; Eyes to the left

What does GABA-A regulate? Patellar reflex level? Ankle? Orbital fractures cause what sensory deficits via what nerve? When would CN VII injury occur? Anesthesia on butt and thigh is what nerve? Wilson's effects what part of the brain? Where does posterior & anterior tongue divide? What artery causes CN III palsy? Precentral vs postcental gyrus? Left frontal eye field? Paramedian pontine?

Mononuclear infiltrate with lymphocytes & germinal centers; occasional multi-nucleated giant cells

What does Hashimoto's show on histology? deQuervain?

Cell membrane breakdown; Vibrio; Margination, rolling, activation, adhesion/crawling & transmigration;

What does LDH indicate? Raw oysters is what bacterial infection? 5 steps of leukocyte accumulation?

H. pylori infection

What does a duodenal ulcer most likely indicate?

Contralateral hemiballismus

What does a lesion in the subthalamic nucleus cause?

Primary syphilis; Penicillin interferes with cell wall peptidoglycans

What does a painless penile lesion indicate? What is the mechanism of the drug used to treat this?

Amorphous pale pink material within normal myocardium; Sqrt(1/40,000)=1/200=q, carrier frequency is 2q=1/100; IgG, C3b, CRP & mannose-binding lectin

What does cardiac amyloid look like? What is the probability of someone being a carrier for an autosomal recessive mutation if the population incidence is 1/40,000? What 4 molecules are opsonins?

Increases NADH/NAD+ ratio which causes lactic acidosis (pyruvate to lactate), hypoglycemia (oxaloacetate to malate) & ketoacidosis (TCA cycle doesn't want to make more NADH so acetyl-CoA gets shunted)

What does ethanol metabolism do to intermediate ratios and what 3 reactions does this effect?

Hyperplasia of the stratum spinosum

What does histology of acanthosis nigricans show?

Onion bulbing from continual remyelination and demyelination

What does histopathology of Charcot Marie Tooth disease show?

Nodules with surrounding hypoattenuation "halo signs"

What does imaging of aspergillosis usually show?

Increased true positives & decreased false negatives; Sensitivity * # patients with disease; (1-sensitivty)* # of patients with disease; HSV-1

What does increased sensitivity mean (2)? Calculation for TP? FN? Person infected with a virus that shows an initial flare with less severe cyclic pattern of relapse has what?

Balanced venous & arterial vasodilator; Coagulative; Preserved tissue architecture with anucleated cells; Aortic isthmus where ligament arteriosum attaches

What does nitroprusside do (2)? What kind of necrosis follows ischemic injury? What does histology show (2)? Traumatic aortic rupture most often occurs where?

Branchial cleft cyst between the mandible and sternocleidomastoid

What does persistence of the second branchial cleft cause?

Vasoconstriction & aldosterone activation; Increases end-diastolic sarcomere length; Compliance is an inherent property of the tissue (hypertrophy, amyloid etc. change this); Levodopa overdose

What does renin do (2)? What does infusion of saline do in hypovolemic shock? How does compliance differ? When are jerking movements seen in Parkinson's?

Intraparenchymal hemorrhage, not subarachnoid

What does rupture of a Charcot-Bouchard aneurysm cause?

TB because it impairs macrophage function; Kidney; Spleen; H2PO4 & NH3; Pyoderma gangrenous & erythema nodosum

What does silicosis predispose to? 12th rib fracture damages what? 9th-11th? With high H+ loss in the urine, which two molecules act as acid buffers? What two derm manifestations are associated with ulcerative colitis?

The neutral valine creates a hydrophobic pocket to interact w/ each other and sickle; Lysine is polar so there's no interaction; Right shift; Proline residues; Dendritic cells & T lymphocytes

What does the mutation in HbS do? Why is homozygous HbC disease so mild? HbS shifts the curve what direction? What distorts alpha helices? 2 cells in the paracortex of lymph nodes?

Atrophy of the caudate=Huntington's

What does this MRI show?

Raises RA pressure above left; PFO because even though VSD is more common, PFO is more common cause of embolism; Scoping of the bladder; Osler's=painful, immune complex, Janeway=painless, septic emboli; Roth spots; splinter hemorrhages

What does valsalva release do to the atria? A woman with an cryptogenic stroke likely has what heart defect? What is cystoscopy? What are the two differences in Osler's nodes & Janeway lesions? What has the same mechanism of Osler's? Janeway?

Kidney stones

What does vitamin C excess cause?

The need for ribose-5-phosphatefor nucleotide synthesis; The 2nd half is reversible so F6P & G3P get converted to R5P instead; Anemia=pancytopenia, crisis=just decreased reticulocytes; SS- circular; Red pulp hyperplasia as reticuloendothelial cells remove damaged RBCs

What drives the pentose phosphate shunt? If G6PD isn't working, what happens? Aplastic anemia vs aplastic crisis? Hep D characteristics? What causes splenomegaly in pyruvate kinase deficiency?

Dexamethasone

What drug can be given prophylactically to prevent post-operative nausea and vomiting?

Prednisone; Hemagglutinin antigen preventing viral entry; Neuraminidase preventing viral release; IgA bind pilli or other membrane proteins to prevent mucosal adherence; Staphylococcal protein A

What drug helps prevent radio-contrast allergic reaction? The killed influenza vaccine inhibits what protein and process? Oseltamivir? How does IgA inhibit Neisseria? What bug virulence factor impairs complement activation?

Dexamethasone>hydrocortisone, but high-dose hydrocortisone is still acceptable

What drug is used in an Addison crisis?

Ranolazine; Inotropic=increased contractility, chronotropic=increased parasympathetic tone; Platelets release PDGF which proliferates SMC

What drug treats angina refractory to other therapies? What inotropic & chronotropic effects does digoxin have? What provides proliferative stimuli for atherosclerotic plaques via what mechanism?

Carbamazepine, cyclophosphamide & SSRI's

What drugs can cause SIADH?

High triglyceride & low HDL with no change in LDL; waist circumference or was it-to-hip ratio

What dyslipidemic changes are seen in DM2? How is visceral fat estimated?

Maternal diabetes mellitus

What endocrine disorder causes polyhydramnios?

Diabetes

What endocrine disorder does PCOS increase the risk of in young women?

Megaesophagus and achalasia

What esophageal pathology does Trypanosoma cruzi cause (2)?

Decreases glucagon and insulin but insulin more so leading to hyperglycemia; Steatorrhea (secretin), hypochlorhydria (gastrin) & biliary stones (CKK)

What glucose manifestation do somatostatinomas cause? What other symptoms are seen and from suppression of what (3)?

Impaired reabsorption of bicarb; ARDS; Hyaline membrane deposition; Behcet syndrome; Uveitis; Chronic interstitial nephritis & ischemic papillary necrosis not ischemic tubular necrosis; Prolonged hypotension

What happens in type 2 renal tubular acidosis? Guys with pancreatitis and respiratory failure has what? Autopsy findings of lung? What is a vasculitis with genital & oral ulcers? 3rd symptom? Chronic NSAID use causes what two kidney manifestations? When does tubular necrosis occur?

It is increased; Vt * breaths/min; (Vt-deadspcae)* breaths/min; Amilrodie increases Ca2+ absorption, triamterene does not; Pulmonary edema;

What happens to FRC in obstructive conditions? Equation for minute ventilation? Alveolar ventilation? Triamterene vs amiloride effects on Ca2+? NO2 toxicity causes what?

Ampicillin causes a papular rash in mono patients

What happens to a patient treated for strep but who really has mono?

Fibrin narrows the capillary wall; Too few subjects to detect rare events; Ipsilateral limb ataxia

What happens to the kidney in HUS? Serious effects appearing after FDA approval is due to what? What does this image manifest as?

Coarctation of the aorta because it causes upper body hypertension; Decreased antibiotic entry

What heart defect is associated with a ruptured cerebral aneurysm? Besides the normal resistance to aminoglycosdies, how is Psuedemonas resistant?

Echocardiogram with contrast & agitated normal saline to observe if dilated pulmonary capillaries allow passage of bubbles into the left atrium

What imaging modality is used to diagnose hepatopulmonary syndrome?

SCID, ataxia-telangiectasia, Hyper-IgM & Wiskott-Aldrich; Internal carotid, ophthalmic, retinal; Nerve, artery, vein (NAV); Under the ribs; Inguinal ligament, adductor longus & sartorius

What immunodeficiencies are problems with both B & T-cell dysfunction (4)? From the neck to the retina, what vessels would a blood clot travel through? From lateral to medial, what is the order of vessels in the femoral triangle? Where is VAN? What borders this superior, medially & laterally?

Caudal regression syndrome=maternal diabetes

What increases the risk of this syndrome?

Herpes family viruses especially CMV or EBV and Campylobacter jejuni

What infections commonly lead to GBS?

Amantadine; Candidemia; Mucosal invasion via Peyer's patches as nontoxigenic (shiga toxin) strains still produce bloody diarrhea; Perineal body

What influenza drug is used in Parkinson's to increase dopamine? Patients with vascular catheters are at high risk for what infection? What is the main pathogenic mechanism of Shigella? In an episiotomy, what structure is most likely injured?

Glucagon/palmitoyl-CoA & insulin/citrate; Malonyl-CoA inhibits the carnitine acyltransferase shuttle

What inhibits (2) and activates (2) actyl-CoA carboxylase? What inhibits beta-oxidation & how?

Alpha & beta interferons bind to type I interferon receptors which leads to transcription of RNase L (endonuclease that degrades RNA) & protein kinase R (inhibits elongation factor-2); only active in the presence of double-stranded RNA

What interferons are generated in response to viral infections & what do they do? How are they selective for viruses?

Villous; Transverse duodenum; Recurrent C. diff infections; Inhibits RNA poly; Vibrio, norwalk virus & hep A; Betel nut chewing; Skin infection w/ gluteal draining tracts;

What intestinal adenoma is worse? SMA & aorta can compress the renal vein & what else? Fidaxomicin is used for what? Mechanism? Shellfish associations (3)? Esophageal SCC risk in Asians? What is intergluteal pilonidal disease?

Diabetic arthropathy; caused by the combination of diabetic neuropathy and vasculopathy

What is Charcot arthropathy? What causes this?

Substrate concentration to reach 1/2 Vmax; Glisson's capsule made of fibrin; Colipase displaces inhibitory bile-salts; alpha-amylase; PLA2; Angiodysplasia

What is Km? What is the capsule around the liver called & made of? Pancreatic enzyme that digest triglycerides & MOA? Carbs? Phospholipids? Hx of GI bleed with anemia & no colonoscopy findings is what?

PDE3 inhibitor so increases cAMP; Positive inotrope; Causes vasodilation so contraindicated in hypotension; Vagus nerve; (RR-1)/RR*100

What is Milrinone mechanism of action? When is it used? Who is it contraindicated in? What nerve is the afferent nerve for the aortic arch baroreceptors? Calculation for attributable risk percent?

Avulsion of tibial tuberosity due to overuse; Pes anserinus bursitis; Sartorius; Less likely to cause GI bleed, more likely to cause cardiovascular event; Only in inflamed tissues; Amiodarone

What is Osgood-Schlatter disease? Anteromedial knee pain in a runner is likely what? What muscle attaches there? Benefit & drawback of celecoxib? Where is COX-2 found? What QT-prolonging drug has the lowest risk of doing so?

Thrombocytopenia due to splenomegaly

What is a common hematologic manifestation of liver failure?

Subjects are randomly allocated to a sequence of 2 consecutive treatments with a washout period in between; Mature mRNA has a poly A tail; TAG; Muscle, bone & fat because it goes to high perfusion areas first and then redistributes

What is a crossover study? What molecule has a ton of A residues? What repeat do telomeres have? A highly lipophilic drug will go where last (3)?

Mandibular prognathism; Wilms tumor, macroglossia & organomegaly; octreotide

What is a facial deformity found in Angelman syndrome? What are the symptoms in Beckwith-Wiedemann syndrome (3)? Treatment?

Agranulocytosis

What is a hematologic effect of all antipsychotics?

A pheochromocytoma found outside of the adrenal medulla

What is a paraganglioma?

Myocardial infarction; chronic kidney disease & noncoronary atherosclerosis

What is a patient with type 2 DM most likely to die from? What other two diseases are at equal risk of this?

Crypt abscesses and ulcers

What is a physical feature of the intestines in IBD?

Rifampin

What is a prophylactic treatment for N. Meningitidis?

Enlargement of veins in scrotum; Aorta & superior mesenteric; 1C, 1A, 1B; Ischemia-induced arrhythmia

What is a varicocele? Entrapment of the left renal vein between what two arteries can cause this? Order the class I antiarrhythmics from strongest to weakest binding? When are IB most useful?

Infertility; Prostate cancer, endometriosis & precocious puberty; Dating error; Decreased estriol; 46XX because its from one 23X sperm duplicating in an empty ovum while 46XY would require two different sperm 23X & 23Y and an empty ovum

What is an indicate for pulsatile leuprolide? Continuous (3)? Most common cause of low second trimester AFP? What marker indicates fetal growth restriction? What is the most common karyotype of a complete mole?

Vitamin B1

What is another name for thiamine?

Lamination of the glomerular basement membrane

What is another term for the splitting of the basement membrane seen in Aport syndrome?

Hypoxic states (lung disease, R->L shunts) not anemia; Omeprazole; Retinal damage; Osteoprotegerin

What is associated with clubbing?What drug is associated with increased risk of hip fracture? Main side-effect of hydroxychloroquine? What molecule is Denosumab modeled after which blocks RANK & RANKL interaction?

It's the troponin of smooth muscle; JAG1; Disappearance of bile ducts; TOF; Increase; Decrease

What is calmodulin? Alagille syndrome is in what gene? What liver manifestation is seen? Heart manifestation? What do diuretics/ARBs/ACEI do to renin? Beta-blockers?

<70 mg/dL; Glyburide

What is considered hypoglycemic? What diabetes medication is most likely to cause this?

Presence of erythema migrans; western blot; topical antivirals

What is diagnostic of lyme disease? What if no rash is seen? How do you treat HSV keratitis?

Physiologic jaundice appears after the first 24 hours of life

What is difference in time of presentation between physiologic jaundice and Crigler-Najjar syndrome?

Arterialized ventricle w/ tricuspid valve malformation; Congenital torticollis; Neuro (tremors), GI (diarrhea/vomiting), autonomic (sweating/sneezing); Morphine or methadone;Sodium bicarbonate

What is ebsteins anomaly? Newborn with a firm left sided swelling that elicits pain when turned the opposite direction has what? What three signs are involved in neonatal abstinence syndrome (maternal opioid use)? Treatment (2)? What treats TCA overdose?

Many seborrheic keratoses; Axillary; Deltoid & teres minor; Coracoid process fracture; MS=spastic & cauda equina=flaccid

What is leser-trelat sign? Most common nerve injury in anterior shoulder dislocation? Two muscles involved? Shotgun/rifle shooting is associated with what? Cauda equina syndrome vs MS cause what bladder manifestations?

Probability of a binary outcome based on independent variables (developing gastric cancer from smoking, alcohol etc.); Right marginal artery

What is multiple logistic regression used for? If LAD is occluded what artery provides collateral circulation?

GTP; Succinyl-CoA to Succinate

What is necessary for oxaloacetate to synthesize PEP? Which TCA cycle reaction produces this?

Weight loss; pregnant or cystic fibrosis patients

What is often an initial presenting symptom of type 1 DM? When is an oral glucose tolerance test used?

Relief of dysuria in UTI's; Antimuscarinics (Tolterdine); Human placental lactogen; syncytiotrophoblasts

What is phenazopyridine used for? What class of drugs do you use to treat overactive bladder (specific example)? Increase in insulin sensitivity in maternal diabetes is due to what hormone? What secretes this hormone?

Single gene causes multiple effects in different tissues; A disease can be caused by multiple different genes; GU5' AG3'; Dopamine (D1 is the main controller of renal blood flow); low dose=B1, high dose=alpha 1

What is pleiotropy? Locus heterogeneity? What is the splice sites for sNRPs? A drug that increases renal blood flow and CO is what? What is the dose dependence of this drug?

Head of the pancreas, duodenum, proximal jejunum, gall bladder, common bile duct, and distal stomach

What is removed during a Whipple procedure?

Nodular glomerulosclerosis (Kimmelstiel-Wilson nodules)

What is shown here (two names)?

Ring fibers

What is shown on pathology of myotonic dystrophy?

It is the only DNA polymerase that possess 5' to 3' exonuclease activity so is the only one that can remove RNA primers & replace them with DNA

What is special about DNA polymerase I?

Hypotension in pregnant woman when lying down; Uterus obstructs IVC decreasing preload; Liver function; Nickel, Quarter/nickel, Quarter/dime, dime

What is supine hypotension syndrome? Mechanism? What must be checked before starting a statin? What are the cardiac pressure readings in RA, RV, pulm artery & LA?

Progesterone & glucocorticoid inhibitor; PGE1 analogue; P450 inhibitors; Menotropin mimics FSH producing a dominant follicle & hCG mimics LH to rupture the follicle

What is the action of mifepristone (2)? Misoprostol? What drug metabolism effect due protease inhibitors have? In menotropin, hCG combo therapy for infertility, what do each of them do?

Protamine sulfate; Communicating a choice, understanding info, appreciating consequences & rationale for decision (mild cognitive impairment does not indicate lack of decision making capacity); Epinephrine then diphenhydramine

What is the antidote for heparin? A patient refusing treatment who is oriented to person, place & time but not to month & date must meet what four requirements? Rash, dyspnea & hypotension after administration of ceftriaxone should be treated with what (2)?

Chlorhexidine, not used in neuro, ophtho or oto procedures due to its neurotoxicity; Disrupts cell membranes & coagulates cytosol; Formaldehyde cross-links DNA

What is the antiseptic of choice for surgical procedures & when is it contraindicated (? Mechanism of action (2)? What is the name and function of the antiseptic used to sterilize hospital instruments that can't be autoclaved?

Throat culture first because many viruses also cause this & Antistreptolysin O serologic titer confirms previous group A streptococcal infections only

What is the best diagnostic test for oral pharyngitis?

1/4 (1/2 chance she got it from mom * 1/2 chance she gives it to baby); Leukemoid is left shift (pic) w/ +LAP, CML is basophils w/ -LAP; Flexor digitorum profundus

What is the chance that a womans male baby will have Duchennes, given that her brother has it? Leukemoid reaction versus CML (2)? What tendon flexes the DIPs?

Increased succinylacetone & urine smelling of cabbage

What is the characteristic elevated lab and smell of hereditary tyrosinemia?

Hemorrhagic cystitis; Fast=e. coli, klebsiella, enterobacter, slow=citrobacter & serratia

What is the combination of dysuria, hematuria, and hemorrhage referred to as what? List the 3 gram negative rod fast lactose fermenters & 2 slow ones.

Alpha-ketoacid dehydrogenase; thiamine...lipoate, coenzyme A, FAD, NAD (Tender Love & Care For Nancy); pyruvate dehydrogenase; for hyperammonemia in urea cycle

What is the defect in maple syrup urine disease? What treatment should be initiated & what 4 other cofactors are required? Other enzyme that uses these cofactors? When is arginine administered?

Isolated hypertension before 20 weeks of gestation with the development of proteinuria or other signs/symptoms of preeclampsia/eclampsia after 20 weeks of gestation; HTM after 20 weeks gestation

What is the definition of chronic hypertension with superimposed preeclampsia? Gestational hypertension?

Pneumonia contracted at least 48 hours after being admitted into a healthcare setting; Enterobacter, Escherichia coli, Klebsiella & Salmonella

What is the definition of hospital-acquired pneumonia? What four organisms are in the Enterbacteriaceae family?

IgM antibodies develop after 2-3 weeks so absence of IgM antibodies early acute infection

What is the difference between acute infection & early acute infection with Hep B?

Central=high sodium, no change in urinie osmolality after deprivation but huge increase with desmopressin; nephrogenic=high sodium, no change & mild increase; primary polydipsia=low sodium, increased with water restriction but no change with desmopressin

What is the difference between central DI, nephrogenic DI & primary polydipsia in regards to serum sodium & water deprivation test results?

Co-transport occurs against a concentration gradient using the favorable gradient of another molecule; Carrier-mediated is down a concentration gradient with the help of transmembrane proteins

What is the difference between co-transport & carrier-mediated transport?

Oligohydramnios=too little amniotic fluid in the amniotic sac; polhydramnios=excess of amniotic fluid

What is the difference between oligohydramnios and polyhydramnios?

Oral candidiasis=Candida albicans & plaques can be scraped off, oral hairy leukoplakia=EBV & cannot be scraped off

What is the difference between oral candidiasis & oral hairy leukoplakia in organism and presentation?

Posterior pituitary is usually transient until the neurons regrow while hypothalamic is permanent

What is the difference between posterior pituitary & hypothalamic causes of central DI?

Triploidy=entire extra set of chromosomes (69); Trisomy=1 extra chromosome (47)

What is the difference between triploidy and trisomy?

PCP=diffuse, bilateral infiltrates; Aspergillosis=cavitary lesions OR consolidations

What is the difference in CXR between PCP & aspergillosis?

Neonates=mother ingesting cat feces or undercooked meat, immunocompromised=reactivation

What is the difference in acquired cause of Toxoplasma infection in neonates versus immunocompromised patients?

GAS skin infection=PSGN, GAS pharyngitis=ARF

What is the difference in clinical syndromes following GAS skin infection versus GAS pharyngitis?

Rotor=gallbladder can be visualized by oral cholecystography, Dubin-Johnson=biliary system cannot be visualized; Dubin-Johnson=black liver, Rotor=no black liver

What is the difference in imaging in Dubin-Johnson and Rotor Syndrome? Biopsy?

Pituitary apoplexy=hemorrhage & sheehan syndrome=ischemic necrosis

What is the difference in mechanism between pituitary apoplexy & sheehan syndrome?

Glycogen branching enzyme=amylo-1,4-1,6 transglucosidase & glycogen debranching=amylo-1,6 glucosidase; Anderson's disease presents with early cirrhosis

What is the difference in naming of glycogen branching enzyme versus glycogen debranching enzyme? A young patient with signs of liver failure& positive PAS stain likely has what disease?

High-flow=painless erection, cavernosal needle aspiration, low-flow=painful, injection of phenylephrine; Relief of pain when the scrotal contents are elevated, testicular torsion & epididymitis; Epididymitis has intact cremasteric reflex while testicular torsion does not

What is the difference in presentation & treatment of high-flow versus low-flow priapism? What is Prehn sign & what pathologies is it associated with (2)? How do you differentiate these two?

Pain that only occurs after a meal, asymptomatic & AST/ALT elevations

What is the difference in presentation between cholelithiasis, uncomplicated choledocholithiasis & complicated choledocholithiasis?

Sex hormones are decreased in 17alpha (lack of secondary sex characteristics, XY=ambiguous) and increased in 11beta (virilization)

What is the difference in presentation in sex characteristics between 17alpha and 11beta CAH?

MM=Primary amyloid lightchain protein; RA=secondary amyloid associated protein

What is the difference in proteins that cause multiple myeloma versus RA?

Pre=tamoxifen or other ER receptor antagonist, post=aromatase inhibitor because their aromatase levels are low enough for these drugs to have an effect

What is the difference in treatment of ER+ breast tumors in premenopausal and postmenopausal women?

Paget's=disorganized mosaic lamellar bone, Vit D deficiency=osteoid matrix accumulation around trabeculae, osteopetrosis=primary spongiosa in medullary cavity w/o mature trabeculae

What is the difference on histology between Paget's, Vitamin D deficiency & osteoporosis?

Vitiligo=complete absence of melanocytes and depigmentation, tinea versicolor=destruction of melanocytes and hypopigmentation; yeast clusters with short, curved, septate hyphae

What is the difference on microscopic imaging between vitiligo and tinea versicolor? What is the actual term for spaghetti and meatball appearance?

Purkinje, Atrial muscle, Ventricular muscle, AV node (Park at Ventura Ave); Vitamin B12

What is the fastest to slowest conduction velocity (pneumonic)? What vitamin deficiency can PPI's cause?

Bartholin's glands

What is the female equivalent of the males bulbourethral glands?

Ethosuximide because it targets the T-type calcium channels involved in these seizures

What is the first line treatment for absence seizures and why?

Meissner=light touch, pacinian=vibration & Ruffini=pressure

What is the function of Meissner, pacinian & Ruffini corpuscles respectively?

It binds the Fc portion of IgG, preventing complement activation

What is the function of Protein A in Staph aureus?

Scaffold protein that facilitates the degradation of β-catenin in the Wnt pathways; Lynch syndrome

What is the function of the APC protein? What disease has a mutated mismatch repair protein?

Overlapping cells with finely dispersed chromatin; intranuclear inclusions, grooves, psammoma bodies

What is the histologic description of Orphan-Annie nuclei? What else can be seen in papillary carcinoma (3)?

Laryngotracheobronchitis

What is the longer name for croup?

Storage and isosmotic concentration of bile made by the liver; release cholecystokinin (CCK) from I cells of the duodenum & jejunum

What is the main function of the gallbladder? What triggers release of bile?

Abacavir hypersensitivity reaction in HLA-B57 postive patients; Pyrimethamine; Indole positive species (Pastuerella); Eikenella

What is the major side effect of Abacavir? Trimethoprim & methotrexate have the same cellular target as what other drug? What bacteria produces a mouse-like order (classic example)? What organism is usually implicated in human bites?

5 rads

What is the maximum amount of radiation exposure in pregnancy?

Decreased adipose, decreases leptin which decreases pulsatile GnRH release

What is the mechanism behind amenorrhea in anorexic patients?

Inhibition of transpeptidases & other PBP; Carbapenem; HPV not herpes; Serpentine or medusa head

What is the mechanism of action of cephalosporins? What is the drug of choice in ESBL-producing organisms? An HIV positive man with rectal bleeding and hard mass in the anal canal most likely has what infection? What does anthrax look like on microscopy?

Calcimimetic that allosterically activates calcium sensing receptor in the parathyroid gland, decreasing PTH release

What is the mechanism of action of cinacalcet?

Impaired androgen-receptor (testosterone) interaction; Spironolactone & cyproterone; Anovulatory cycles normal in adolescents; presents similarly but in obese patients or people receiving estrogen only therapy

What is the mechanism of action of flutamide? What two other drugs do this as well? What is the cause of a young girl who does not get her period every month but has tremendous flow when she does w/ spotting in between periods? When does complex atypical hyperplasia of the endometrium occur?

GnRH receptor agonist, even though continuous administration acts like an antagonist

What is the mechanism of action of leuprolide

Antagonism of para-aminobenzoic acid (PABA)

What is the mechanism of action of sulfamethoxazole?

Inhibition of cGMP-specific phosphodiesterase for erectile dysfunction; TSH, glucagon & PTH

What is the mechanism of sildenafil and when is it used? What three hormone receptors use Protein Kinase A as a second messenger?

Altering normal composition of predominantly hydrophobic bile acids, thereby attenuating their propensity to destroy cell membranes & decreases bile cholesterol content

What is the mechanism of ursodiol?

Beta-carotene builds up in the blood due to lack of thyroxine which normally speeds the conversion of beta-carotene to retinol

What is the mechanism of yellowing skin in hypothyroidism?

Prevents lymphocytes/monocytes from gaining access to the central nervous system by binding integrins to prevent their adhesion to vascular endothelium

What is the method of action of Natalizumab?

Skin infection like cellulitis, myositis, and necrotizing fasciitis

What is the most common complication of VZV?

Complete atrioventricular canal defect (ASD, VSD & common AV valve); Holosystolic murmur of AV regurg w/ mid-diastolic rumble due to increased pulmonary venous return

What is the most common heart defect in Downs? What are the heart sounds (2)?

Impaired post-translational processing of the CFTR protein; Renal ammoniagenesis metabolizes glutamine to make ammonia and bicarb; Exercise, emphysema & Pulmonary fibrosis; Club cells

What is the most common molecule defect in CF? In ischemic colitis, what gets rid of the lactic acid through the urine? Normally equilibration of pO2 is perfusion limited, what 3 states is it diffusion limited? What cells can regenerate ciliated cells in the bronchioles?

Constipation from iron sulfate

What is the most common side-effect from treatment of iron deficiency anemia?

TSH levels because increased TSH occurs well before decreased T3/T4; secondary or tertiary hypothyroidism

What is the most sensitive test for hypothyroidism? What is the exception?

Lack of barrier contraception; Aromatase deficiency; Hirsutism because the placenta is trying to synthesize estrogen but the missing aromatase leads to androgen build up and diffusing back to the mom

What is the most significant risk factor in a woman with cervical intraepithelial neoplasia grade 3? A 46XX baby born with ambiguous genitalia w/ increased testosterone & androstenedione likely has what? How can the mother present & why?

AB toxin which cleaves SNARE proteins preventing Ach release

What is the name and mechanism of botulism toxin?

Ramsay Hunt syndrome; Ophthalmic fluorescein exam not Tzanck smear; EBV=single lesion & toxoplasma=multiple lesions

What is the name for facial paralysis, pain in the ear & loss of taste to the anterior 2/3 of the tongue? If the V1 trigeminal nerve is involved what is the next step? In an HIV patient, what separates EBV brain lesions from toxoplasma?

Atypical teratoid rhabdoid tumor

What is the name of the pediatric brain tumor with a mutated INI1 tumor suppressor gene on chromosome 22?

Probably acute bronchitis so albuterol and antitussives; Pneumonia would have a fever

What is the next best step in management for someone who has trouble breathing, fits of coughing up clear sputum & rales at the base of the lungs? Why is this not pneumonia?

CO2; IL-4; sodium-(Cl+bicarb); Poor liver perfusion which normally clears lactic acid & decreased oxidative phosphorylation due to poor oxygenation

What is the one thing that stimulates central chemoreceptors? What stimulates TH2 development? Calculation for anion gap? Two mechanisms of lactic acidosis in septic shock?

ADP-ribosylates Gs alpha protein, increasing adenylate cyclase which increases cAMP; same but activates cGMP

What is the pathogenic mechanism of Vibrio cholera? ETEC?

Gram negative corckscrew (syphilis); Doxycycline; Allergic desensitization therapy

What is the shape of the organism causing this rash following a penile lesion 2 months prior? Treatment for someone w/ a penicillin allergy? What if they're pregnant?

Exogenous corticosteroids are administered to prevent upregulation of ACTH (testosterone inhibitors won't work because it's upregulation of androgens that is the problem)

What is the treatment for CAH & why?

Succimer & dimercaprol

What is the treatment for mercury poisoning?

Upper right quadrant pain, fever, & jaundice; no jaundice

What is the triad of ascending cholangitis? How will cholecystitis differ?

Deer fly not the black fly

What is the vector for Loa Loa?

Sense of satiety; hyperphagia

What is the ventromedial nucleus of the hypothalamus responsible for? What does its destruction cause?

Kimmelstiel wilson nodules; Hematoxylin, eosin PAS; Usually no casts; TH2 cells; Thiazide diuretic; Proteinuria

What is this? What three key stains are associated with this? What does the urine look like? Response to schistosomiasis is governed by what? What is matolazone? What causes frothy urine?

It lacks a 3' to 5' exonuclease proofreading activity leading to high variation in antigenic HCV proteins

What is unique about HCV compared to the other hepatitis viruses?

Ceftriaxone and azithromycin; add doxycycline

What is used to treat Neisseria gonorrhoeae infection? Co-infection with Chlamydia trachomatis?

Acetic acid solution and topical ciprofloxacin

What is used to treat a nail infection caused by Psuedemonas (2)?

Dimercaprol; hemachromatosis

What is used to treat lead toxicity? What is deferoxamine used for?

ALT; amylase would not indicate a biliary cause of pancreatitis

What lab is important in diagnosing gallstone pancreatitis?

Serum thiocyanate levels; Nitroprusside is non-enzymatically transformed to cyanide to form cyanomethemoglobin

What laboratory test measures nitric oxide after administration of nitroprusside? Why?

Hepatoduodenal; Inferior vena cava; Abetalipoproteinemia; MTP gene; HIV & Parkinson's; Superior mesenteric; CN X

What ligament does the portal triad run through? Persistent bleeding after occlusion of this ligament indicates injury to what? Intestinal biopsy with foamy villi tips is what? Gene mutated? Seborrheic dermatitis associations (2)? Midgut malrotation volvulus is around what artery? In contrast to the gag reflex, the cough reflex is mediated by what single nerve?

Annular ligament; Ulnar collateral ligament; IgG4 PLA2R

What ligament is injured in nursemaid's elbow? What ligament is injured in pitchers? What antibodies are associated with idiopathic membranous nephropathy?

Non-selective beta blockers as they mask hypoglycemic symptoms, use B1-selective if necessary

What medication should be used with caution in diabetics & what is a better choice?

Naegleria fowleri; slower clinical course usually only in immunosuppressed patients but can cause keratitis in contact lens wearers

What microorganism is typically found in warm freshwater and can cause acute meningoencephalitis? How does Acanthamoeba differ?

Pubococcygeus

What muscle directly controls urine flow and orgasm in women?

Iliopsoas; Gluteus medius; tRNA; Inflammation converts arginine to it; Excess androgens; Finasteride to decrease DHT; Alopecia areata

What muscle is used to sit up without your hands? What muscle keeps the hip level during gait? A RNA molecule that contains chemically modified bases is what? Where does citrulline come from in RA? Pathogenesis of male pattern baldness? Treatment? What are glucocorticoid injections used for?

V maxillary; V mandibular; CN 9, 10, 11; Contract; M3 in endothelial cells produces NO which diffuses and activates rosin light chain phosphates & relaxation; Hypoglycemia, hyperammonemia, hyperketones; Cerebral blood flow;

What nerve passes through foramen rotundum? Foramen ovale? Jugular foramen (3)? Does detrusor contract or relax to pee? M3 receptor mechanisms in endothelial cells? Deficiency in methylmalonyl-CoA mutase has what symptoms (3)? What do anesthetics increase?

Fructose-1-phosphate b/c it enters after phosphofructokinase regulatory step; Increased gastrin; Overproduction of TGF-alpha; Glands so decreased gastrin secretion; Calcium binds to the extra lipids instead of oxalate; Involvement of the peritoneum; Obturator internus involvement; Psoas involvement

What non-glucose monnosaccharide is metabolized fastest & why? Cause of parietal cell hyperplasia? What causes gastric fold enlargement in Mentrier's disease? What is atrophied? Hyperolaxuria in crohn's is due to what? What causes appendicitis pain to migrate to RLQ? Pain on internal rotation? Hip extension?

Raphe nuclei; Locus ceruleus; Catecholamines & tyrosine, phenylalanine is actually built up; Dexamethasone; GABA, glycine, serotonin & dopamine

What nuclei has serotonergic neurons? Norepi fight/flight neurons are located where? BH4 deficiency causes low what (2)? Pneumococcal meningitis benefits from administration of what? 4 inhibitory neurotransmitters?

Burkholderia cepacia; Greater than 10% loss of birthweight or failure to recover lost weight by 10 days of age

What obscure organism infects the lungs of cystic fibrosis and CGD patients? What classes failure to thrive?

Streptococcus pyogenes (Group A Strep)

What organism causes most cases of necrotizing fasciitis (Larger name)?

Coffee-bean nuclei=Brenner tumor; Cowden's disease

What pathology and diagnosis is observed? What diagnosis is associated with PTEN mutations?

Desmoplasia; Tumor-induced proliferation of non-neoplastic fibrous connective tissue

What pathology is suggestive of adenocarcinoma in the breast? Definition?

Beta-oxidation of fatty acids

What pathway is mainly responsible for providing the substrate in DKA?

Schizont (replicated and about to burst); Fever as bursting of the blood cells causes this; Trophozoite

What phase of plasmodium is this? What symptom accompanies this? What phase looks like a ring?

Isotype switching & somatic hypermutation; Bone marrow; That there are falsely low levels of iron; Enterocytes increase intestinal transporter & hepatocytes upregulate hepcidin; Decreases nephrotoxicity of platinum-containing alkylating compounds

What processes occur in germinal centers (2)? Where does VDJ recombination occur? HFE protein mutations cause body to think what? What do enterocytes & hepatocytes do in response? What is amifostine used for?

Low oxygen levels so right shifted; Binds LPS & activates macrophage; VEGF & FGF; Increases mitosis; Somatomedin C; Vitamin B12 deficiency; Cystic fibrosis; Vinca alkaloid; Ileum & peripheral neuropathy; Pulmonary fibrosis & skin discoloration;

What promotes RBC sickling? CD14 function? Two factors that drive angiogenesis? What does EGF do? Another name for IGF-1? Neurologic dysfunction after folate is what? Phenylalanine deletion is what? What group is vincristine? 2 side effects? 2 of bleomycin?

Myotonin-protein kinase; Inhibition of intestinal CYP3A; Kinetic; Hypertrophic osteoarthropathy, derm/polymyositis & Trousseau's; GFR

What protein is altered in myotonic dystrophy? Grapefruit increasing cyclosporine is caused by what? Is this pharmacokinetic or dynamic? Adenocarcinoma can cause what three neoplastic syndromes? Inulin measures what?

Calcipotriene (Vitamin D analog that inhibits keratinocyte proliferation); Ustekinumab; IL-12; Deep peroneal nerve; pyrimidine dimers; Double strand breaks; Base deamination

What psoriasis drug activates a nuclear transcription factor (mechanism)? What monoclonal antibody is used? Target? Skin between the big toe is enervated by what? UV radiation causes what genetic mutation? Ionizing radiation? Chemical exposure?

Gianotti-Crosti; Hep B, EBV, enteroviruses & RSV

What rash like syndrome can present after viral infection? What 3 main viruses cause this (4)?

Norovirus usually causes sickness in adults too, not just children; ETEC

What separates the presentation of norovirus from rotavirus? What would the causative pathogen be if travel was involved?

Iron; The rectum (think of Hirschsprung's); Epinephrine metabolites; Acute outflow obstruction (e.g. Budd-Chiari) not chronic disease; Just mucus and sloughed cells, no leukocytes;

What should be supplemented at 4 months in breast fed infants? What portion of the colon has innervation last in development? What is not secreted in Dubin-Johnson's? Dilation of sinusoids is seen in what? What is seen in the stool in watery diarrhea?

B1 selective beta blocker; B2 blockade decreases insulin & causes hypoglycemia; After 60 seconds; 30 minutes; Prostacyclin (PGI2); Platelet activation & constriction

What should you administer to a patient with an acute MI? What effect do beta-blockers have on glucose (2)? In ischemia, how fast does contractility stop? For how long is damage reversible? What is the most potent inhibitor of platelet aggregation (2nd name)? What two things does TXA2 do?

Gq, Gi, Gq, Gs, Gi; Deviation of the infundibular septum; S3=systolic heart failure, S4=diastolic heart failure

What signaling pathways are associated with M1, M2, M3, D1 & D2 receptors? What embryologic event malfunction causes TOF? S3 versus S4 is seen in what?

Photoallergic reactions which are T-cell mediated; Chemically mediated=phototoxic

What skin reaction can sulfonylureas cause? How does this differ from chemically mediated reactions?

Microsporum; Obstructing tubular hyaline casts; HSV & mycoplasma (also drugs & malignancy); CD8+; Tissue inhibitors of metalloproteases (TIMPs); Colonizing; Necrotizing pneumonia & granulomas

What species causes tinea? Histology of multiple myeloma kidney disease? Two most common causes of erythema multiforme? Cells involved? Neutrophil elastase is inhibited by antitrypsin, what is macrophage elastase inhibited by? Aspergillosis ball in TB cavity is what type? What does invasive cause (2)?

Hypnozoite

What stage of the organisms life-cycle of Plasmodium ovale does primaquine interfere with?

P bodies store, regulate & turnover mRNA; DVT, estrogen-tumor, >35yo smoker, hypertriglyceridemia, liver disease, pregnancy

What step of mRNA processing occurs in the cytosol? 6 contraindications to OCPs?

Spindle afferents via Ia fibers; measures muscle force and causes relaxation via Ib fibers to an inhibitory interneuron on the alpha motor neuron

What structure initiates the mitotic stretch reflex? What is the function of the golgi tendon?

ACetyl-CoA inhibits pyruvate dehydrogenase & stimulates pyruvate carboxylase which converts pyruvate to oxloacetate

What substance directly stimulates hepatic conversion of pyruvate to glucose?

Creatinine, magnesium, and chloride

What substances are found at higher concentrations in the CSF than the blood?

Myoclonus; Contralateral subthalamic nucleus injury; Fc receptor on neuts & macs; Overflow; Overflow; Aggressive squamous cell carcinoma in previously traumatized area

What term describes the physical nature of hiccups? When does hemiballism occur? What is CD16? Diabetic autonomic neuropathy causes what incontinence type? Obstruction? What is a Marjolin's ulcer?

Rubber, dyes & leather; Nocardia, pseudomonas, listeria aspergillus, candida, e. coli, staph, serratia, B. cepacia, & h. pylori (Cats Need PLACESS to Belch Hairballs); Seizures & arrhythmia; Cytochrome P450 system

What three occupational exposures are associated with transitional cell bladder cancer? What are the catalase positive organisms (pneumonic)? Two symptoms of theophylline toxicity? What is it metabolized by?

Sodium nitrite=promotes methemoglobin formation, sodium thiosulfate=donates sulfur to convert to thiocyanate, hydroxocobalamin=cobalt moiety binds cyanide; Lactic acidosis & bright red blood

What three things can be given for cyanide toxicity & what are there mechanisms? What two hematologic manifestations are seen?

Arsenic exposure (Mee's lines); Homemade moonshine/wine or glass/woodworking; Bowen's disease (squamous cell carcinoma in situ)

What toxicity is associated with these lines? Two industry buzzwords for this toxicity? What associated disease are they at increased risk for (description)?

Cholera-like toxin; LPS on gram negative bacteria; EIEC versus EAEC

What toxin does ETEC use? What is endotoxin? What E coli invade epithelial cells versus cause intestinal cell adhesion?

Gastrin and histamine

What two chemicals induce gastric acid secretion?

16 & 4; 6; 11

What two chromosomes is the mutated gene for ADPKD located on? ARPKD? Wilm's Tumor?

Adenosine & dipyridamol (PDE inhibitor); Incidence is new cases in people at risk, prevalence is total cases in the whole population; Confounding not selection bias

What two drugs cause coronary steal syndrome (drug class)? Prevalence versus incidence? Stratifying a population by age, race, etc reduces what form of bias?

RAS & BRAF; p53 & Rb; MSH2; EGFR; PDGF

What two genes are signal transducer proteins? Cell cycle regulators (2)? DNA repair? Solid tumors can have upregulation of what gene? Glioblastomas up regulate what gene?

Sacropsinous & sacrotuberous; Sciatic, piriformis; Dehydrogenase & reductase; Succinate dehydrogenase; Vibrio vulnificus

What two ligaments form the greater sciatic foramen? What nerve & muscle go through here? What two general reactions are associated with niacin? What dehydrogenase reaction in the TCA used FAD instead? Cellulitis after swimming is what organism?

FRC & RV; Decreased DLCO & increased total lung capacity; Neutrophil elastase; Paget's is osteoclasts initially; Osteoclast/osteoblast mixed; Osteoblasts

What two parameters are increased in COPD? Two unique to emphysema? Alpha 1 antitrypsin works through what enzyme? Haphazard segments of lamellar bone with cement lines is due to increased activity of what cells initially? Stage 2? Stage 3?

M protein & N-acetyl-beta-D-glucosamine; Mitral regurgitation, Syndenham chorea & erythema marginatum; pyrogenic toxin; M protein

What two proteins are involved in rheumatic fever molecular mimicry? What cardiac, neurologic & skin findings are characteristic? What is the cause of scarlet fever? What factor inhibits phagocytosis & complement activation?

Coronavirus & Rhinovirus (Picornavirus); Rhinovirus does not spread to the lower respiratory tract to pneumonia or bronchitis

What two viruses cause the common cold (family of 1)? How do you differentiate them?

Hemangioblastomas; Agraphia, acalculia, finger agnosia & left-right disorientation; Angular gyrus; Rare artery of Percheron; Flexor digitorum superficialis & profundus; ALS; SOD1

What vascular manifestation in the cerebellum can VHL have? Gertsman syndrome 4 signs? Area effected? Bilateral thalamic stroke is due to what? What two muscles does the median nerve pass between? Spinal cord w/ cortical neuron loss is what? Gene implicated?

Coltivirus; Reovirus; Astrovirus; Tenofovir/emtricitabine

What virus causes Colorado Tick fever? Viral family? What causes acute gastroenteritis in an immunocompromised patient? Drug for HIV prophylaxis after needle stick?

Initially will increase because of the agonistic properties and then steadily decrease; Tamoxifen; Increased estrogen:androgen ratio w/ ductal epithelial hyperplasia

What will testosterone and DHT levels be after leuprolide administration? What can you give to combat gynecomastia induced by this medication? Physiological cause and histology of gynecomastia?

Radioactive iodine treatment destroys thyroid cells causing an outpour of antibodies

What will transiently worsen Grave's disease eye symptoms? How?

Achlorhydria due to PPI use, gastritis or VIPoma; Gastrinoma would increase stomach acid so increase the infectious dose; Vibrio vulnificus

What would make the infectious does of Vibrio cholera lower (3)? What would increase it? What are patients with severe liver disease at risk for?

Bronchiolitis from RSV; RV is increased, FVC is decreased & TLC is unchanged; Phosphatidylcholine; Hematogenous osteomyelitis of the metaphysis; Vertebral body

Wheezing in a kid with fever is what? Changes in TLC, RV & FVC with age? What is another name for lecithin? A boy begins limping after 4 days of fever, diagnosis and area effected? Most common area effected in adults?

2/3 because you know that she isn't showing disease signs so the homozygous recessive option can be crossed out; Promoters are right next to the start codon while enhancers can be anywhere

When calculating genetic probabilities what is the chance of a mother being a carrier if her sister has the recessive disease? Promoter vs. enhancer location?

Categorical data "goodness of fit" versus comparing two means; LA/LV thromboembolism; 5-14 days; >14 days

When do you use chip-squared versus t-test? Most common cause of a renal infarction? When does ventricular free wall rupture occur? True aneurysm?

Alveoli; Bronchioles; Terminal bronchioles; Bronchioles; alpha-1 antitrypsin deficiency; Avascular necrosis; Fibroproliferative disease of the palmar fascia

When does cilia dissappear in the airway? Cartilage? Goblet cells? Mucous glands? Man with decreased FEV1/FVC ratio and father had liver disease has what? Scaphoid fracture has what risk? What is dupuytren's contracture?

Only with LV dysfunction; Drop in systolic blood pressure during inspiration as the intraventicular septum bows into the LV (cardiac tamponade); Slow-rising, low amplitude pulse (aortic stenosis)

When is electric alternates seen in cardiac tamponade? What is pulses paradoxus (disease)? What is pulsus parvus et tardes (disease)?

Glycogenolysis is up to hour 18 of fasting while gluconeogenesis is after 18 hours

When is glycogenolysis versus gluconeogenesis active?

At the FRC as inhalation collapses interstitials and exhalation collapses extra-alveolar; Very bottom of loop of Henle; Cutaneous angiosarcoma's developing due to chronic lymphedema; Stimulates osteoblast maturation & increases calcium absorption

When is pulmonary vascular resistance at its lowest? In the absence of ADH, where is osmolarity the highest? What is Stewart-Treves syndrome? MOA of teriparatide (2)?

With onset of labor; Patients with respiratory conditions as it is only available in the inhalation form

When should Group B strep prophylaxis be administered? In what patient population is Zanamivir contraindicated in?

Aplastic anemia; Chloramphenicol, sulfonamides & carbamazepine; A=VIII X, B=IX X, C=XI AR; Deep-tissue bleeding like hemophilia; Collagen to Gp1b; Protects circulating factor VIII; Factors V & VIII

When you see pancytopenia what should you consider? 3 drugs that cause this? 3 hemophilia factors & inheritance? How does factor XIII deficiency present? What does vWF bind (2)? Other function? What do Protein C & S inactivate?

PCT=65% & loop of Henle=25%; Bicarb, amino acids & glucose; Loop diuretics; Staph, strep & Hib; Cervical lymphadenitis

Where & how much of K+ absorbed (2)? Order the three reabsorbed substances in the PCT from least to most. What is the most potent diuretic used for rapid symptom relief? 3 causes of secondary bacterial pneumonia? What does M. scrofulaceum cause?

Through the internal inguinal ring, lateral to the inferior epigastric artery

Where do indirect inguinal hernias protrude from?

Uvea (iris, ciliary body, choroid) because it is well oxygenated; Choroid b/c iris & ciliary body are in the anterior chamber

Where does TB like to infect in the eye? If examination showed no anterior chamber changes, where is the organism infecting?

Ang 2=PCT, aldosterone=DCT; TP/(TP+FP); Phase 3; PDA machine like murmur w/ inspiratory split; 1-P^# of events

Where does any 2 absorb sodium versus aldosterone? Calculation for PPV? What part of the action potential do class III most effect? Splitting of S2 in a child with a continuous murmur is most likely what? Probability of 1 event being different than the rest?

Near the branch point of the constant region; End of the constant region; J chain in IgA/IgM; Hemolysis or microcytosis; Macrocytosis; Fibrin from fibrinogen; None! PT, PTT and bleeding time are all normal; Transglutaminase

Where does complement bind? Fc receptor? What else binds here? What can cause falsely elevated HBA1c? Falsely lowered? Factor 1a is also called what? Factor XIII deficiency shows what labs? What type of enzyme is it?

Infundibulopelvic (suspensory) ligament; Ovarian artery, vein & nerve; Uterosacral ligaments; Prostatic venous plexus (lympathic spread to the skeletal system is very rare)

Where does ovarian torsion usaully occur? What does it contain? Tension loss of what ligament causes uterine prolapse? Prostate cancer that has metastasized to the spine travels through what?

Hypothalamus & Liver; Liver is the functional one

Where is IGF-1 produced (2)? Which one is systemic?

Superior temporal gyrus; Inferior frontal gyrus; Conduction aphasia with inability to repeat but preserved comprehension and output

Where is Wernicke's area located? Broca's? What does disruption of the arcuate fasciculus cause?

The CCA tail at the 3' end; The anticodon loop that minds mRNA; Binds the ribosome; Quantifies messenger RNA; DNA-binding proteins; Double stranded DNA

Where is an amino acid loaded onto a tRNA? What is across from this? What does the T loop do? What does RT-PCR do? What are southwestern blots used for? What is the probe used?

Nasal septum; Superior labial & anterior ethmoidal; Middle meatus; Duodenum; Vagal nerve; Hardend lense does not thicken upon ciliary muscle contraction so image is focused behind retina;

Where is the Kiesselbach plexus? What branch of the facial & ophthalmic artery are involved? Most common site of nasal polyps? What section of the intestine is involved in celiacs? Posterior external auditory canal is innervated by what? Presbyopia mechanism?

TNF-alpha "cachectin"

Which cytokine is thought to contribute to the cachexia seen in advanced cancer "name"?

Macrolides; Refer for prostate biopsy as finasteride keeps PSA low so any rise is worrisome; External male structures (penis, scrotum & prostate)

Which drug class is bacteriostatic at low doses and bactericidal at high doses? A man on finasteride has a PSA of 1.6 which rises to 2.4 ng/mL 12 months later, next best step? Finasteride is category X in pregnancy because it inhibits formation of what (3)?

Androgens, cortisol, aldosterone, fat soluble vitamins & thyroid hormone

Which hormones bind intracellular receptors (5)?

No special toxin just direct activation of the immune response resulting in increased intracellular cAMP

Which mechanism does Salmonella enteritidis use to exert it's effects?

Autoimmune hypothyroidism and celiacs disease

Which two autoimmune disorders are most commonly seen in patients with type 1 diabetes?

Berry aneurysms in the circle of Willis leading to subarachnoid hemorrhages

Which two brain manifestations does adult polycystic kidney disease present with?

RBC's because they lack mitochondria & liver cells because they lack the enzyme that converts acetoacetate to acetoacetyl-CoA; Free fatty acids because they cannot cross the blood-brain barrier

Which two cells cannot utilize ketones and why? What can't the brain use for energy?

Lithium & amiodarone

Which two non-thyroid related drugs commonly require thyroid monitoring?

Middle meningial artery in the temporal area; Foramen spinosum

Which vessel usually causes an epidural hematoma? Which landmark does this vessel pass through?

Demargination causes them to detach from the vessel walls

Why are neutrophil counts increased with corticosteroid administration?

Ileum damage causes bile salt wasting; Reduces risk; Sensorineural hearing loss; Autsomal dominant, FGFR3; 50% because parent must be heterozygous as homozygous is lethal; NOD2 in NF-kB; Chronic mesenteric ischemia, not gallstones; Gastrin & delta cells;

Why do Crohn's patients develop gallstones? Statins effect on gallstones? air>bone is what? Achondroplasia inheritance & gene? Chance of child being short if parent is? Gene & pathway in Crohn's? Postprandial epigastric pain w/ Hx of atherosclerosis is what? What two cells are in the antrum?

In addition to phenylalanine hydroxylase, it is also a cofactor for tyrosine hydroxylase to make DOPA

Why does dishydrobiopterin deficiency cause neurologic problems even after tyrosine supplementation?

B/c only RBC's are elevated; Neoplastic; Spondyloarthropathy; Prostate, breast, kidney, thyroid, lung (lead [PB] kettle); Direct thrombin inhibitor; Dihydrofolate polglutamate; Folinic acid; M3; M4 eosinophilic

Why is secondary polycythemia a misnomer? Spinal pain worse at night is what? Relieved with exercise? Malignancies that metastasize to bone (pneumonic)? What is hirudin? Methotrexate causes build up of what? Leucovorin is also called what? Vitamin A treats what version of APL? Chromosome 16 inversion causes what subtype?

High fist-pass metabolism; PDA w/ eisenmenger syndrome b/c coarctation would have to be really severe to cause this and they would not survive until 10; alpha 2 agonist; alpha 1 agonist; Do not allow them to leave the ER and reassess when they're sober

Why is the bioavailability of isosorbide denigrate so low? 10 year old with toe cyanosis/clubbing but no finger signs has what? What is clonidine? Phenylephrine? How do you handle a drunk patient who refuses treatment?

Recite the months backwards; 1st-gen antihistamine; Benzos; Wilson's disease; Dopamine agonists; To offset constipation of aluminum w/ diarrhea of magnesium; Milk-Alkali syndrome

Woman complaining of concentration problems should be asked to what? Chlorpheniramine is what? Contraindicated w/ what? Psych symptoms w/ liver is what? RLS treatment? Why would aluminum & magnesium hydroxide be combined? Risk of calcium carbonate antacids?

IBS b/c it's associated with psychological stress; Neuroleptic malignant syndrome, sepsis would not have muscle rigidity; Splenic sequestration; Anti-HBs; Valproic acid; Totally normal; P450 inhibitor because smx is a sulfonamide;

Woman w/ anxiety about her diarrhea /constipation has what? Rigidity, confusion & fever in a schizophrenic is what? Sickle patient with LUQ is always what? Recovery of Hep B indicator? What is divalproex? 10 month old who crawls, babbles & separation anxiety is what? Side effect of tmp-smx?

Ureter; Under; Because the other ureter is intact; Sampson's artery

Woman who underwent hysterectomy & now has back pain & fever probably has an injury to what? Does this go over or under the uterine vessels? Why can they void normally? What artery is the anastomoses between ovarian & uterine arteries?

Posterior subserosal uterine leiomyoma; Because we would not see uterine enlargement in a rectocele; Internal iliac artery

Woman with an enlarged uterus & constipation which is help by pushing on the posterior vaginal wall likely has what? Why is this not a rectocele? Ligation of what artery is indicated to control postpartum hemorrhage?

Metastatic breast cancer; Deeper than the basement membrane; Cystocele; Epistaxis, telangiectasias (pic) & AVMs; Heparin induced thrombocytopenia;

Woman with no history or other symptoms has two lytic lesions in her spine, what type of cancer? Where is subcutaneous tissue? Bulging into the anterior vagina on valsalva is what? 3 symptoms of osler-weber-rendu? Platelet fall after heparin is what?

Decreased intracellular calcium & myosin dephosphorylation; Attempt to contact any family to get consent to intubate (apparently intubation is nonimmigrant in this case, key word is MIGHT); Macrophages versus SMCs in plaques

cGMP mechanism (2)? Patient who might need intubation & his good friend says he doesn't want it, what do you do? Main cell in fatty streaks in young people?


Kaugnay na mga set ng pag-aaral

Chapter 54: Drugs Acting on the Upper Respiratory Tract

View Set

PrepU Chapter 37: Management of Patients with Musculoskeletal Trauma

View Set

BODY COMPOSITION/BODY MASS INDEX

View Set

Chapter 11. Poverty, Inequality, and the Welfare State

View Set

CompTIA 901 Unit 2.5 WiFi Standards

View Set

interpersonal communications test 2

View Set

EAPS 10500 | Units 10-12 (Moons, Tides, and Rings; Exoplanets; Hazards of Space Travel) | Study Guide (Exam 4)

View Set